You are on page 1of 59

REVIEWER ON CONSTITUTIONAL LAW II | CYNDI SATORRE-BICERA | 2nd Sem | S.Y.

2021-2022 1

Why does the Constitution devotes an entire Article for CLASSES OF RIGHTS
the Bill of Rights? And why was it placed in Article III
in the Constitution? 1. Natural Rights – those possessed by every citizen
without being granted by the State for they are given to
A: It is purposely placed there to set limitations and to man by God as human beings created in His image that he
ensure that the State will not be committing abuses in its may live a happy life. (inherent and inalienable). In PIL,
exercise of its inherent powers — police power, power of every child has the right to be born.
imminent domain, and power of taxation. 2. Constitutional Rights – conferred and protected by the
Constitution.
The 1987 Constitution is considered as a 3-in-1 3. Statutory Rights – provided by law, promulgated by the
Constitution. Primarily divided into 3, the provisions are: law-making body and consequently may be abolished by
the same body.
1. CONSTITUTION OF SOVEREIGNTY (supreme power
to govern) – Government of the people, for the people and CLASSIFICATION OF CONSTITUTIONAL RIGHTS
by the people.
1. Political Rights – the power to participate directly or
Article I (Philippine Territory) – this is the first indicia that indirectly in the establishment or administration of the
the Constitution is making known its desire to exercise government. – Ex. Right to citizenship, right to suffrage,
sovereignty over the metes and bounds of its territories right to information, right to form political parties, etc.
over the Philippine archipelago (even extend up to 12-nm
from the baseline stretch which we call the territorial sea). 2. Civil Rights – a law which secures private individuals
Simply put, sovereignty means the State has the power to for the purpose of securing enjoyment of their means of
ordain and enforce laws over the entirety of the Happiness. – Ex. Rights against involuntary servitude, right
Philippines. not to be imprisoned because of debts, property rights,
right to a healthful and balance ecology, right to education
Article II (Declaration of Principles and Policies) –
Section 1. “The Philippines is a democratic and republican 3. Social and Economic Rights – (ICESCR) – intended to
state. Sovereignty resides in the people and all government insure the well – being and economic security of an
authority emanates from them.” individual.

The people own the sovereignty thus we adhere to the 4. Rights of the Accused (Section 14, Art. XIII) – intended
principle of democracy (our freedom) and republicanism for the protection of a person accused of any crime.
(which impose certain limits on freedom). The rights of
other persons are the limitations of our own right. ‘sic utere GENERATIONS OF THE STUDY OF HUMAN RIGHTS
tuo ut alienum non laedas’ (use your own right, but do not 1st generation – Civil and Political Law (ICCPR)
violate the right of others). A right seizes to be one if in the 2nd generation – Social & Economic Rights (ICESCR)
exercise of it, you are trampling upon the rights of others. 3rd generation – Right to (Collective) Development

BUT, the Constitution is also regarded as a social contract The relationship between human rights and civil and
where people who wield or possess sovereignty under the political rights: Human rights are broader in scope as a
1987 Constitution surrender their sovereignty in favor of matter of fact civil and political rights are just two slices of
the government so that the Government now will be the the cake known as human rights law.
one to exercise the sovereign powers to avoid chaos from
people exercising each of their own sovereignty. POLICE POWER – "that inherent and plenary power in
the State which enables it to prohibit all that is hurtful
2. CONSTITUTION OF GOVERNMENT to the comfort, safety, and welfare of society.” In other
Art. VI – Power of the legislature to make/repeal laws. words, police power is vested upon any civilized
Art. VII - Executive Department executive these laws government for purposes of promoting the general welfare
Art. VIII – Judiciary merely interprets the law or common good. This is actually one of the essences of
Art. IX – Constitutional Commissions democracy and of course republicanism as espoused by
Art. X – Local Government (autonomy of all LGUs) Section 1, Article II.
Art. XI – Ombudsman, SB and Tanodbayan
Art. XIII – Commission on Human Rights
SCOPE OF THE POLICE POWER – insofar as the present
Constitution is concerned, the exercise of Police Power will
3. CONSTITUTION OF LIBERTY (Article III BOR)
justify the enactment of legal measures that would cover
three very important areas:
BILL OF RIGHTS 1. Public Health; 2. Public Safety, 3. Public Morals

Declaration and enumeration of a person’s rights and SECTION I, ARTICLE III


privileges which the Constitution is designed to
(Due process and Equal Protection Clause)
protect against violation by the government, or by
individuals or by groups of individuals.
Section 1. No person shall be deprived of life, liberty, or
property without due process of law, nor shall any person
Basis: According to Marcelo H. Del Pilar, perfection of
be denied the equal protection of the laws.
humanity is not possible without freedom for the individual.
Thus, the existence of social institutions and all political
1. Life – is the most important form of human rights. A
organizations and relationships are justified insofar as they
person who is breathing but is not accorded with all his
have for their primary aim the defense and protection of
human rights is good as dead.
freedom.
2. Liberty – everyone’s liberty cannot be taken away from

From the Discussions of Atty. Enrique Jr. Bonocan


University of Mindanao College of Law
REVIEWER ON CONSTITUTIONAL LAW II | CYNDI SATORRE-BICERA | 2nd Sem | S.Y. 2021-2022 2

him without due process. by the courts but by the administrative bodies which are
not always bound by the finer points of the judicial due
3. Property – one of the very important components of process as mandated by the Constitution. As explained in
comfortable living. Labor Rights – also partakes of a these two doctrines:
property right. An employee cannot be just terminated from
his employment without due process. Doctrine of Exhaustion of Administrative Remedies –
The mandate of the rules of procedures, there are certain
DUE PROCESS – implies the right of the person affected cases which though may be cognizable by the trial court
thereby to be present before the tribunal which pronounces but one cannot immediately file charges without going
judgment upon the question of life, liberty and property in through the administrative process.
its most comprehensive cases to be heard by testimony or
otherwise and to have the right of controverting by proof of Ex: If an LGU enacts a tax ordinance (questionable). The
any material fact. [Medenilla vs. Civil Service Commission]. Local Government Code says that before you can question
Due process of law is a proceeding which hears before it the validity of a Local Tax ordinance before the court, you
condemns, the punishment of the guilty only proceeds after have to bring it first to the attention of the SOJ → O.P. → CA
inquiry and renders judgment only after trial. [Dartmouth (if judgment is unfavorable).
College vs Woodward & Daniel Webster]
Doctrine of Primary Jurisdiction – there are issues that
Simply put, before the State condemns it has to give the need not be brought to the attention of the judicial courts
person a chance to be heard first; before it could proceed first because those issues might be dependent upon the
to punish the guilty, it has to conduct an inquiry, and it expertise of the appropriate administrative agency.
cannot render judgment if trial has not been conducted.
Ex: For valid monetary claims against the government, you
TWO ASPECTS OF DUE PROCESS — According to US need not go to court first, you have to first bring it to the
vs. Ling Su Fan (1908), due process of law is a process attention of the COA (Constitutional Law 1)
according to the law of the land. Due process of law" is not
that the law shall be according to the wishes of all the
The observance of fairness in the conduct of any
inhabitants of the state, but simply First, that there shall be
investigation or inquiry is at the very heart of the due
a law prescribed in harmony with the general powers of the
process. The essence of due process is to be heard
legislative department of the Government; Second, that
and as applied to administrative proceedings, this
this law shall be reasonable in its operation; Third, that it
means a fair and reasonable opportunity to explain
shall be enforced according to the regular methods of
one’s side or an opportunity to seek a reconsideration
procedure prescribed; and Fourth, that it shall be
of action or ruling of complaint.
applicable alike to all the citizens of the state or to all of a
class.
Administrative due process cannot be fully equated with
due process in its strict judicial sense, for in the former a
The first and second principles are known as the
formal or trial-type hearing is not always necessary, and
Substantive Due Process; and The third and fourth
technical rules of procedure are not strictly applied. [Peter
principles are known as Procedural Due Process.
Vivo vs. PAGCOR]
TWO COMPONENTS/ASPECTS OF DUE PROCESS
*RRACS – Revised Rules of Administrative Cases in the
Civil Service.
In the case of Alliance for The Family Foundation, vs.
Janette Garin, due process of law has two aspects:
Due process requirement entails the opportunity to be
substantive and procedural. Substantive due process
heard at a meaningful time and in a meaningful manner.
refers to the intrinsic validity of a law that interferes with the
This is satisfied if the parties are given the opportunity to
rights of a person to his life, liberty, or property. Procedural
explain their respective sides through position papers or
due process, on the other hand, means compliance with
pleadings. Or when a person is notified of the charge
the procedures or steps, even periods, prescribed by the
against him and given the opportunity to explain or defend
statute, in conformity with the standard of fair play and
himself.
without arbitrariness on the part of those who are called
upon to administer it. Stated otherwise in the case of
ANG TIBAY DOCTRINE (1940s) [Ang Tibay vs. CIR]
Secretary Of Justice vs. Lantion, it consists of the two
basic rights of notice and hearing, as well as the guarantee
CARDINAL RIGHTS in administrative proceedings:
of being heard by an impartial and competent tribunal.
(1) the right to a hearing, which includes the right of the
party interested or affected to present his own case and
Simply put, in substantive due process the law must be
submit evidence in support thereof.
valid, a person cannot be convicted of a crime based on an
(2) The tribunal must consider the evidence presented.
invalid law otherwise, the elementary maxim of nullum
(the right to hearing and the right to present of a party
crimen nulla poena sine lege applies. Procedural due
would be rendered meaningless if the tribunal will not give
process is procedural fairness.
any strain of consideration with regards these pieces of
Ex. Under the ROC, the trial cannot proceed without the
evidence.
arraignment of the accused. An accused tried and convicted in (3) The decision must have something to support itself.
absentia prior to his arraignment must be acquitted even if he (4) Not only must there be some evidence to support a
was guilty as hell because there is a blatant violation of the finding or conclusion, but the evidence must be
rules on criminal procedure. "substantial." – quantum of proof required.
(5) The decision must be rendered on the evidence
Due Process is not always judicial due process. There are presented at the hearing, or at least contained in the
other legal controversies that may have to be decided not record and disclosed to the parties affected.

From the Discussions of Atty. Enrique Jr. Bonocan


University of Mindanao College of Law
REVIEWER ON CONSTITUTIONAL LAW II | CYNDI SATORRE-BICERA | 2nd Sem | S.Y. 2021-2022 3

(6) The tribunal or body or any of these judges must act on


in sustaining ordinances based on the general
its or his own independent consideration of the law and
welfare clause.
facts of the controversy, and not simply accept the views of
a subordinate in arriving at a decision.
GR: Potestas non potest delegari
(7) The Court should, in all controversial questions, render
XPN: When congress delegates police power to LGUs.
its decision in such a manner that the parties to the
proceeding can know the various issues involved, and the
Congress enacted RA 7160 in 1991. It is a valid
reasons for the decision rendered.
delegation of police power.
Two-Fold purpose of Administrative Due Process
LIMITATION: it cannot go against a statute/national law.
1. Contributes to the accuracy and thus minimizes errors;
2. More intrinsically to the person with the subject of
deprivation, it gives him a sense of rational participation
and a decision that can affect his destiny and thus, Ynot v. IAC, G.R. No. 74457, March 20, 1987
enhances his dignity. (Fr. Bernas)
• 6 carabaos were transported in a pump boat from
Part of the due process clause is publication. Masbate to Iloilo and were confiscated by the police for
Publication cannot be dispensed with, without affecting the violation of EO 626-A. Petitioner Ynot brought an action
validity of the law. The 15-day period can be increased or for replevin and challenged the constitutionality of the
shortened, but in no case shall publication be dispensed EO. Trial court sustained the confiscation and did not rule
with. Therefore, there is no such thing as immediate on the constitutionality of the law on the ground that it
effectivity of the law (if the period is silent, follow the 15-D lacked authority to do so. Decision is affirmed by the IAC.
period (Tanada vs. Tuvera). Reason being that due
process which is a rule of fairness requires those who must HELD: EO No. 626-A was declared unconstitutional. The
obey a command must first know what to obey. Else, the measure struck at once and pounced upon the petitioner
maxim ignorantia legis non excusat. (Art. 3, NCC) is N/A without giving him a chance to be heard, thus denying
him the centuries old guarantee of elementary fair play –
due process.
ERMITA-MALATE HOTEL V CITY MAYOR OF MANILA,
G.R. No. L-24693, July 31, 1967
Bautista v. Juinio, G.R. No. L-50908, 1/31/1984
• Manila City Ordinance No. 4760 questioned for being
violative of the due process clause as it imposes too • Pres. Marcos signed an LOI that banned the use of
many restrictions and regulations on hotels/motels vehicles under the "H" and "EH" classifications of the
particularly Section 1: fees, filling out of form, inspection. LTC on certain days. Exempted are vehicles classified as
Service, Truck, DiPLomatic, Consular Corps, Tourist
HELD: NO, the petition is not covered by the mantle of Cars. MC No. 39, issued by respondent Minister of
protection associated with the due process guarantee. PWTC and respondent LTC, imposing the penalties "of
Police power measures being specifically aimed to fine, confiscation of vehicle and cancellation of license”.
safeguard public morals are immune from such Petitioners claim that the LOI is discriminatory and a
imputation of nullity resting purely on conjecture and denial of due process.
unsupported by anything of substance. Petitioner’s
claim must bow down to police power. To hold otherwise HELD: The laws operate equally and uniformly on all
would be to unduly restrict and narrow the scope of persons under similar circumstances or that all persons
police power which has been properly characterized as must be treated in the same manner, the conditions not
the most essential, insistent and the least limitable of being different, both in the privileges conferred and the
powers, extending as it does "to all the great public liabilities imposed. The government is not required by the
needs.” It would destroy the very purpose of the state if it Constitution to adhere to the policy of all or none. The
could be or allowed itself to be deprived of its LOI cannot be declared void on its face as behind it is the
competence to promote public health, public morals, presumption of validity. The necessity for evidence to
public safety and the general welfare. rebut such presumption is unavoidable as underlying
questions of facts may condition the constitutionality of
legislation. Absence of some factual foundation of record
Velasco v. Villegas, G.R. No. L-24153, 2/14/1983 for overthrowing the statute, The presumption of validity
• Manila City Ordinance No. 4964 prohibits barbershops must prevail.
to conduct massage business and in separate rooms.
Petitioners contend that it amounts to a deprivation of
property, of their means of livelihood without due process EQUAL PROTECTION CLAUSE – refers to the
of law. constitutional guarantee of the equality of a person before
the law. The equal protection clause confers upon a person
HELD: It was NOT UNCONSTITUTIONAL. The court the same rights and obligation under the law under the
explained that it was a police power measure. The same circumstances. There is more truism in legal equality
objectives behind its enactment are: rather than total equality. The equal protection clause does
not prohibit classification, for the government is not
(1) To impose payment of the license fee for engaging in required by the Constitution to adhere to the policy of all or
massage clinic business; none, it does not prevent the legislature from establishing
(2) in order to forestall possible immorality which might classes of individuals or objects upon which different rules
grow out of the construction of separate rooms for shall operate, so long as the classification is not
massage of customers." The Court has been most liberal unreasonable.

From the Discussions of Atty. Enrique Jr. Bonocan


University of Mindanao College of Law
REVIEWER ON CONSTITUTIONAL LAW II | CYNDI SATORRE-BICERA | 2nd Sem | S.Y. 2021-2022 4

EQUAL PROTECTION CONCEPTS:


one employee is a member of INC and they are not
allowed to join unions. There came a law which
• Legal equality. We have the equal number of rights as
exempted them from joining unions, likewise protected
well as the number of obligations under the law under the
them from being terminated because of either a close
same circumstances.
shop/union shop agreement. Religious freedom always
prevails over some contractual obligation. Sometimes,
• Does not prohibit classification. At times, the law may
the law may make some exemptions. According to
impose classification insofar as those who are to be
INC’s ecclesiastical doctrine, this is not allowed and we
benefited by law because it is required by circumstances.
have to respect that. “Guarantee of the equal protection
Ex. Law on Senior Citizen
clause is not a guarantee of equality in the application of
the laws upon all the citizens of the State.”
Classification – is the grouping of things in speculation or
practice because they agree with one another in certain
particulars. A law is not invalid because of simple inequality.
The very idea of classification is that of inequality, so it goes Villegas v. Hiu Chiung Tsai Pao Ho G.R. L-29646
without saying that the mere fact of inequality in no manner
determines the matter of constitutionality. All that is required • A City of Manila ordinance prohibits the employment of
of a valid classification is that it be reasonable. aliens unless they first secure a permit from the Mayor
and pay the fee of ₱500. Violation of which incurs
imprisonment and a fine. The alien respondent who is
PEOPLE V. CAYAT, G.R. No. L-45987, May 5, 1939 employed in Manila brought suit and obtained judgment
from the CFI, layering the ordinance null and void.
• Mr. Cayat, a Non-Christian, native of Baguio and was
sentenced by the Justice of the Peace (MTC) to pay a HELD: The ordinance involved is a tax measure and in
fine of ₱5 or suffer subsidiary imprisonment in case of imposing a flat fee of ₱500, it fails to consider substantial
insolvency for possessing a bottle of A-1-1 gin, an differences in situation among aliens and for that reason
intoxicating liquor contrary to Act No. 1639 which only violates the rule on uniformity of taxation. It also lays
allows native wines and liquors made by the tribes. Cayat down no guide for the granting or denial of permit and
invoked his constitutional protection on the equal therefore, permits the arbitrary exercise of discretion by
protection clause. He claimed that there was an invalid the City Mayor. The ordinance denies aliens’ due process
classification. Argues that the law appeared problematic and the equal protection clause. Requiring a permit
and must be declared unconstitutional. before a person can be employed is tantamount to
denying him the basic right to earn a livelihood, the due
HELD: It is an established principle of constitutional law process clause and the equal protection clause over
that the guarantee of the equal protection of the law is citizens and aliens.
not violated by legislation based on reasonable
classification.
VERA V. CUEVAS, G.R. NO. L-33693-94 5/31/1979
REQUISITES FOR VALID CLASSIFICATION
1. It must rest on substantial distinction which make real • Section 169 of the Tax Code required that all
differences; condensed skimmed milk in whatever form sold in the
2. It must be germane to the purpose of the law; Philippines shall be clearly and legibly marked on its
3. It must not be limited to existing conditions only. immediate containers with the words, "This milk is not
4. Apply equally to all members of the same class suitable for nourishment for infants less than one year of
age” or words of similar import. Private respondents
Act 1639 satisfies these requirements. The distinction is (engaged in the manufacture and sale of filled milk
unquestionably reasonable (history has it that their use of products) brought an action before the CFI of Manila for
intoxicating liquors resulted in lawlessness and crimes). a declaration of their rights. The CFI restrained the CIR
The Act is designed to promote peace and order in the and its agents from enforcing this requirement.
non-Christian tribes. Its ultimate purpose can be no other
than to unify the Filipino people with a view to a greater HELD: It was held that Section 169 of the Tax Code has
Philippines. Judgment AFFIRMED. been repealed by RA 344. At any rate, Section 169
applies only to skimmed milk and not to filled milk;
enforced only against manufacturers of filled milk but not
against manufacturers of skimmed milk. Thus, denying
CENTRAL BANK EMPLOYEES ASSOCIATION V. them the equal protection of the law.
BANGKO SENTRAL G.R. No. 148208 12/15/2004

• Unionism is encouraged by no less than the DUMLAO V. COMELEC, G.R. L-52245 1//22/1980
Constitution. The management and the union must
engage in CBA (may contain a union security clause): (1) • Section 4 of BP 52 provides (in part): “any retired
Close shop agreement – management undertakes to elective provincial city or municipal official … to which he
accept applicants who are not union members; seeks to be elected shall not be qualified to run for the
(2) Union shop agreement – management may accept same elective local office from which he has retired.”
applicants who are not union members but within a Dumlao (former governor of Nueva Vizcaya) and a
specific time as written in the CBA must become a union candidate for 1980 elections sued for prohibition to enjoin
member (terminated if not). enforcement of the law on the ground that it is contrary to
the equal protection and due process guarantee of the
Cited the landmark case of Victoria v. Elizarde (1974), Constitution.

From the Discussions of Atty. Enrique Jr. Bonocan


University of Mindanao College of Law
REVIEWER ON CONSTITUTIONAL LAW II | CYNDI SATORRE-BICERA | 2nd Sem | S.Y. 2021-2022 5

foreign-hires certain benefits not accorded to local-hires


HELD: No. The purpose of the law is to allow the i.e. housing, transportation, shipping costs, taxes, and
emergence of younger blood in local governments. The home leave travel allowance and they are paid 25%
classification in question being pursuant to that purpose, more. The School justifies the difference on two
it cannot be considered invalid. The tiredness of the "significant economic disadvantages": (a) the "dislocation
retiree for government work is present, and what is factor" and (b) limited tenure. International School
emphatically significant is that the retired employee has Alliance of Educators contested, claiming that the
already declared himself tired and unavailable for the point-of-hire classification employed by the School is
same government work. discriminatory to Filipinos and that the grant of higher
salaries to foreign-hires constitutes racial discrimination.

IGOT V. COMELEC, G.R. No. L-52245 1/22/1980 HELD: The point-of-hire classification employed by
respondent School to justify the distinction in the salary
• Romeo Igot filed a petition for Prohibition to enjoin rates of foreign-hires and local hires is an invalid
enforcement of Sec. 4(2), BP 52: “any person convicted classification and is violative of the equal protection
of subversion, insurrection, rebellion or others of similar clause. There is no reasonable distinction between the
offenses and the filing of charges for such crimes before services rendered by foreign-hires and local-hires. The
a civil or military tribunal after preliminary investigation practice of the School of according higher salaries to
shall be a prima facie evidence of such fact.” foreign-hires contravenes public policy and violates the
principle of equal work for equal pay. While the School
HELD: With respect to the presumption arising from the needs to attract foreign-hires, salaries should not be
filing in a court or military tribunal to such crimes after used as an enticement to the prejudice of local-hires. The
preliminary investigation, this provides a contravention of local-hires perform the same services as foreign-hires
the constitutional presumption of innocence as the and they ought to be paid the same salaries as the latter.
candidate is disqualified for public office on the ground The "dislocation factor" and the foreign-hires' limited
alone that charges have been filed against him before tenure also cannot serve as valid bases for the distinction
civil or military tribunal. Such a person is virtually placed in salary rates and those ‘economic disadvantage” are
in the same category as a person already convicted of a adequately compensated by certain benefits accorded to
crime. Although the presumption is rebuttable, time them which are not enjoyed by local-hires, i.e. housing,
constraints may prevent from offering contrary proof, it is transportation, shipping costs, taxes, etc.
best that evidence of disloyalty be passed upon by courts
rather than by an administrative body. – NULL and VOID.
DYCAICO V. SSS, G.R. No. 161357. 11/30/2005
NOTE: In both cases, Igot and Dumlao were not directly
• Bonifacio and Petitioner Elena Dycaico lived together
injured. Dumlao’s petitions are mere requests for an
as husband and wife without the benefit of marriage. In
advisory opinion from the court – theory of judicial review
1989, Bonifacio retired and began receiving his monthly
and in Igot, he has no locus standi (not been convicted or
SSS pension until he passed away in 1997. A few
charged with acts of disloyalty nor disqualified from being
months prior to his death, after retirement, Bonifacio
candidates for local elective positions; can’t sue as taxpayer
married Elena. Elena filed with the SSS an application for
since the issue is not about disbursement of public funds).
survivor's pension. Her application was denied on the
But because of the paramount public interest in the
ground that under Section 12- B(d) of RA 8282 or the
question of the proximity in the election, the Court resolved
Social Security Law, she could not be considered a
the issue on special disqualification.
primary beneficiary.
Q: Are aliens entitled to the equal protection clause?
HELD: The Court held that the denial was invalid and
GR: It applies to all persons, both citizens and aliens. The
declared the proviso on Section 12-B(d) of RA 8282 as
Constitution places the civil rights of aliens on equal footing
violative of the equal protection clause. The proviso
with those of the citizens.
which qualifies the term 'primary beneficiaries' results in
the classification of dependent spouses into two groups:
XPN: Statutes may validly limit exclusively to citizens the
(1) spouses married to SSS members before retirement;
enjoyment of rights or privileges connected with public
and (2) spouses married to SSS members AFTER
domain, the public works, or the natural resources of the
retirement. The petitioner belongs to the second group as
State.
such, she and those similarly situated do not qualify as
'primary beneficiaries' and are not entitled to the pension.
NOTE: The rights and interests of the State in these things
The Congressional concern of the possibility of
are not simply political but also proprietary in nature and so
relationships entered after retirement for the purpose of
citizens may lawfully be given preference over aliens in
obtaining benefits and Section 12-B(d) was apparently
their use or enjoyment.
intended to prevent such ‘sham marriages’ is concededly
valid. However, classifying dependent spouses and
INTERNATIONAL SCHOOL ALLIANCE V. determining their entitlement to survivor's pension based
QUISUMBING, G.R. No. 128845, 6/1/2000 on whether the marriage was contracted before or after
the retirement of the other spouse, regardless of the
• International School, Inc. established primarily for duration of the said marriage, bears no relation to the
dependents of foreign diplomatic personnel and other achievement of the policy objective of the law, i.e.,
temporary residents. The School pursuant to Sec. 2(c) of provide meaningful protection to members and their
PD 732 hires teachers, classifying them into two: (1) beneficiaries against the hazard of disability, sickness,
foreign-hires and (2) local-hires. The School grants maternity, old age, death and other contingencies

From the Discussions of Atty. Enrique Jr. Bonocan


University of Mindanao College of Law
REVIEWER ON CONSTITUTIONAL LAW II | CYNDI SATORRE-BICERA | 2nd Sem | S.Y. 2021-2022 6

seized.
resulting in loss of income or financial burden." The
proviso infringes the equal protection clause. Both are
GR: A person or his house including papers can neither be
still legal spouses.
searched nor seized without a valid search warrant or
warrant of arrest as the case may be.
GSIS V. MONTESCLAROS, G.R. No. 146494 REQUISITES FOR A VALID SEARCH WARRANT
1. There must be an existence of probable cause;
Same situation with Dycaico vs. SSS, only difference in 2. This probable cause must be personally determined by
law involved (GSIS Law). Both Social Security Laws the Judge by examination under oath or by affirmation of
contain constitutional infirmities. the complainant and his witnesses; and
3. The so-called particularity of description of the place to
be search or the things to be seized.
HIMAGAN VS PEOPLE GR No. 113811
Purpose of this provision: “The inviolability of the home
• Himagan, a PNP in Davao, was implicated in the killing is one of the most fundamental of all the individual rights
of Benjamin Machitar, Jr. and the attempted murder of declared and recognized in the political codes of civilized
Bernabe Machitar. After the information was filed with the nations.” – Fr. Bernas
RTC, it issued an Order suspending petitioner until the
termination of the case on the basis of Section 47, R.A. “A man’s home is his castle.” No one can enter into the
6975, otherwise known as DILG Act of 1990. home of another without the consent of its owner.
A privacy of a home, the place of abode, the place where
Himagan filed a motion to lift the order for his suspension a man with his family may dwell in peace and enjoy the
(11 months already), relying on Section 42 of P.D. 807 of companionship of his wife and children unmolested by
the Civil Service Decree. According to him, his anyone even by the king, has always been regarded by
suspension should be lifted because it was supposedly civilized nation as one of the most sacred personal right to
limited to 90 days. Motion was denied, thus he went to whom men are entitled.
the SC claiming that an imposition of preventive
suspension of over 90 days is contrary to the CSC Law PROBABLE CAUSE – defined generally as “such
and would be a violation of his constitutional right to reasons, supported by facts and circumstances, as will
equal protection of laws. warrant a cautious man in the belief that his action, and
the means taken in prosecuting it, are legally just and
HELD: It was not a violation of the equal protection proper. It is “such facts and circumstances antecedent to
clause. Section 47 of RA 6975 is not unconstitutional. the issuance of a warrant, that are in themselves sufficient
The reason why members of the PNP are treated to induce a cautious man to rely upon them and act in
differently from the other classes of persons charged pursuance thereof.”
criminally or administratively insofar as the application of
the rule on preventive suspension is concerned is that FOR PURPOSES OF THE ISSUANCE OF WOA – such
policemen carry weapons and the badge of the law which facts and circumstances which would lead a reasonably
can be used to harass or intimidate witnesses against discreet and prudent man to believe that an offense has
them. If a suspended policeman criminally charged with a been committed by the person sought to be arrested.
serious offense is reinstated to his post while his case is
pending, his victim and the witnesses against him are FOR PURPOSES OF THE ISSUANCE OF SW – such
obviously exposed to constant threat and thus easily facts and circumstances which would lead a reasonably
cowed to silence by the mere fact that the accused is in discreet and prudent man to believe that an offense has
uniform and armed. The equal protection clause exists to been committed and the objects sought in connection with
prevent undue favor or privilege. It is intended to the offense are in the place sought to be searched.
eliminate discrimination and oppression based on
inequality. Recognizing the existence of real differences HOW PROBABLE CAUSE IS ESTABLISHED – Sec. 2
among men, the equal protection clause does not (last part: to be personally determined by the Judge…)
demand absolute equality. It merely requires that all “The establishment of the existence of probable cause
persons shall be treated alike, under like circumstances presupposes the introduction of competent proof that the
and conditions both as to the privileges conferred and party against whom they sought performed particular acts
liabilities enforced. Equal Protection does not absolutely or committed specific omissions violating a given provision
forbid classifications so long as it meets the valid of our criminal law.” – Stonehill v. Diokno
requisites (as laid down in PP vs. Cayat).

Bache & Co. Vs. Hon. Judge Vivencio M. Ruiz, et al.


SECTION II, ARTICLE III
• Respondent Judge issued search warrant No. 2-M-70
relying only on the stenographic notes that were read to
Section 2. The right of the people to be secure in their him.
persons, houses, papers, and effects against unreasonable
searches and seizures of whatever nature and for any HELD: The warrant is invalid and the things seized were
purpose shall be inviolable, and no search warrant or declared to be the fruit of the poisonous tree and were
warrant of arrest shall issue except upon probable cause to ordered to be returned to its owner. Personal
be determined personally by the judge after examination examination by the judge of the complainant and his
under oath or affirmation of the complainant and the witnesses is necessary to enable him to determine the
witnesses he may produce, and particularly describing the existence or non-existence of a probable cause, pursuant
place to be searched and the persons or things to be to Art. III, Sec. 2 (last part) of the Constitution, and Sec.

From the Discussions of Atty. Enrique Jr. Bonocan


University of Mindanao College of Law
REVIEWER ON CONSTITUTIONAL LAW II | CYNDI SATORRE-BICERA | 2nd Sem | S.Y. 2021-2022 7

particularly described in the search warrant


3, Rule 126 of the RROC, both of which prohibit the
• to leave the officers of the law with no discretion
issuance of warrants except "upon probable cause." The
regarding what articles they shall seize, to the end that
determination of whether or not a probable cause exists
"unreasonable searches and seizures" may not be made,
calls for the exercise of judgment after a judicial appraisal
• that abuses may not be committed.
of facts and should not be allowed to be delegated in the
absence of any rule to the contrary. The participation of
respondent Judge in the proceedings which led to the People v. Veloso, G.R. No. L-23051, 10/20/1925
issuance of Search Warrant No. 2-M-70 was limited to (Validity of John Doe Warrants)
listening to the stenographer’s readings of her notes, to a
few words of warning against the commission of perjury, • A John Doe warrant was issued by the Judge and
and to administering the oath to the complainant and his Veloso was arrested. Velos questioned the legality of the
witness – this cannot be considered a personal warrant as it violates the constitutional requirement of
examination. If there was an examination at all, it was the particularity of description.
one conducted by the Deputy Clerk of Court.
HELD: The warrant is valid. General rule, a warrant for
the apprehension of an unnamed party is void, "except in
“Mere affidavits of the complainant and his witnesses are not
those cases where it contains a description personae
sufficient. The Judge has to take depositions in writing of the
complainant and the witnesses he may produce and to attach such as will enable the officer to identify the accused."
them to the record. Deposition is necessary in order that the The description must be sufficient to indicate clearly the
Judge may be able to properly determine the existence or proper person upon whom the warrant is to be served.
non-existence of the probable cause, to hold liable for perjury As the search warrant stated that John Doe had
the person giving it if it will be found later that his declarations gambling apparatus in his possession in the building
are false.” [Mata vs. Bayona] occupied by him at No. 124 Calle Arzobispo, City of
Manila, and as this John Doe was Jose Ma. Veloso, the
Q: Is it necessary that a Judge should personally manager of the club, the police could identify John Doe
determine P.C. before issuance of a valid WOA? as Jose Ma. Veloso without difficulty. A John Doe warrant
can be valid provided that it contains a description
personae.
Soliven vs. Makasiar G.R. No. 82585 11/14/1988

What the Constitution underscores is the exclusive and Pangandaman Vs. Casar, G.R. No. 71782, 4/14/1988
personal responsibility of the issuing judge to satisfy (Limitation of John Doe Warrants)
himself of the existence of probable cause. In satisfying
himself of the existence of probable cause for the • The issuance of John Doe Warrants is not unlimited.
issuance of a warrant of arrest, the judge is not required Insofar, as warrants issued against 50 "John Does" and
to personally examine the complainant and his not one of whom the witnesses to the complaint could or
witnesses. Following established doctrine and procedure, would identify, it is of the nature of a general warrant, one
he shall: of a class of writs long proscribed as unconstitutional and
once anathematized as "totally subversive of the liberty
(1) personally evaluate the report and the supporting of the subject." Clearly violative of the constitutional
documents submitted by the fiscal regarding the injunction that warrants of arrest should particularly
existence of probable cause and, on the basis thereof, describe the person or persons to be seized, the warrant
issue a warrant of arrest; or must, as regards its unidentified subjects, be voided.
(2) if on the basis thereof he finds no probable cause, he
may disregard the fiscal's report and require the
submission of supporting affidavits of witnesses to aid Specific description of property to be seized – the
him in arriving at a conclusion as to the existence of description is required to be specific only insofar as the
probable cause. circumstances will ordinarily allow. It is not required that
technical precision of description be given, particularly,
The judge may rely on the finding of the probable cause where by the nature of the goods to be seized, their
by the investigating prosecutor. description must be rather general, since the requirement
of a technical description would mean that no warrant
could be issued.
SEARCH WARRANT WARRANT OF ARREST
Stonehill vs. Diokno, G.R. No. L-19550, June 19, 1967
Personal determination Judge may not personally (Particularity of description)
by the judge. determine and may rely
upon the finding of the • 42 assailed warrants to seize and take possession of
probable cause by the personal property to wit: books of accounts, financial
investigating prosecutor records, vouchers, correspondence, receipts, ledgers,
journals, portfolios, credit journals, typewriters, and other
PARTICULARITY OF DESCRIPTION documents and/or papers showing all business
transactions including disbursements receipts, balance
Section 2 of Article III requires that the things, the place or sheets and P&L statements and Bobbins (cigarette
person to be searched or the things to be seized must be wrappers) for an alleged “violation of Central Bank,
specifically or particularly described in the warrant. tariffs and custom laws, the internal revenue code
and the RPC.”
Intent of the requirement [Uy Kheytin vs. Villareal]
• to limit the things to be seized to those, and only those,

From the Discussions of Atty. Enrique Jr. Bonocan


University of Mindanao College of Law
REVIEWER ON CONSTITUTIONAL LAW II | CYNDI SATORRE-BICERA | 2nd Sem | S.Y. 2021-2022 8

being arrested who might be armed with a concealed


HELD: The Court in this case said that two points must
weapon and (2) to prevent the person from destroying
be stressed in connection with this constitutional
evidence within his reach.
mandate, namely: (1) that no warrant shall issue but
upon probable cause, to be determined by the judge in
Section 13, Rule 126 of the Revised Rules of Court:
the manner set forth in said provision; and (2) that the
Person lawfully arrested may be searched for a dangerous
warrant shall particularly describe the things to be seized.
weapon or anything which may be used as proof of the
commission of a crime without a search warrant.
None of these requirements has been complied with in
the contested warrants. Moreso, no specific offense had
been alleged in said applications. The averments with Nolasco v. Cruz-Paño, G.R. No. L-69803, 10/8/1985
respect to the offense committed were abstract. As a (Search incidental to lawful arrest)
consequence, it was impossible for the judges who
issued the warrants to have found the existence of • Mila Aguilar-Roque was one of the accused in a
probable cause, for the same presupposes the rebellion case. She was, at the time, at large but had a
introduction of competent proof that the party against standing warrant (fugitive of justice). Around 9 AM of
whom it is sought has performed particular acts, or August 6, the police applied for a search warrant from the
committed specific omissions, violating a given provision respondent to be served particularly in the leased
of our criminal laws. As a matter of fact, the applications residence of Roque which was described as a suspected
involved in this case do not allege any specific acts underground house of CPP/NPA. The search warrant
performed by petitioners. It would be the legal heresy, of was then issued by the respondent judge under the
the highest order, to convict anybody of a "violation of pending criminal case for rebellion. It did not, however,
Central Bank Laws, Tariff and Customs Laws, Internal appear in the records that an application in writing was
Revenue Code and Revised Penal Code," without submitted by the Chief of CDG (Lt. Col Saldajeno), only
reference to any determinate provision of said laws. the deposition of Secret Agent Lapuz was submitted to
Thus, the warrants authorized the search for and seizure the court stating that to his knowledge, there were kept in
of records pertaining to all business transactions of the premises sought to be searched, some records,
petitioners, regardless of whether the transactions were documents and other papers of the CPP/NPA and the
legal or illegal. The warrants sanctioned the seizure of all National Democratic Front including support money from
records of the petitioners and the aforementioned foreign and local sources intended for rebellion. At
corporations, whatever their nature, thus openly around 11:30 Roque and Nolasco were arrested by the
contravening the explicit command of our Bill of Rights — Constabulary Security Group. At 12 noon, the CSG
that the things to be seized be particularly described — searched the premises and seized 428 documents and
as well as tending to defeat its major objective: the written materials, and additionally a portable typewriter,
elimination of general warrants. and 2 wooden boxes, making 431 items in all.

ISSUES: (1) WON the search is valid; (2) WON the


WARRANTLESS SEARCHES AND SEIZURES
seized documents and other property be returned to the
petitioner; (3) WON petitioners must be released.
GR: No person can be searched nor be arrested without a
valid warrant, either for search of his person, house,
HELD: (1) The search was NOT VALID. It is at once
dwelling, papers or effects or even the seizure of his
evident that the foregoing Search Warrant authorizes the
person or those objects named in the warrant after the
seizure of personal properties vaguely described and not
determination of the probable cause personally determined
particularized. It is an all embracing description which
by the judge and with the other constitutional requirement
includes everything conceivable regarding the
that the warrant must be specifically described objects and
Communist Party of the Philippines and the National
things to be seized.
Democratic Front. It does not specify what the subversive
books and instructions are; what the manuals not
XPN:
otherwise available to the public contain to make them
1. warrantless search incidental to lawful arrest;
subversive or to enable them to be used for the crime of
2. search of a moving vehicle; Carroll Doctrine
rebellion. There is absent a definite guideline to the
3. seizure of evidence in “plain view”;
searching team as to what items might be lawfully seized
4. custom searches;
thus giving the officers of the law discretion regarding
5. where there is waiver of rights; consented search
what articles they should seize as, in fact, taken also
6. exigent and emergency circumstances.
were a portable typewriter and 2 wooden boxes. It is thus
in the nature of a general warrant and infringes on the
1. SEARCHES INCIDENTAL TO A VALID ARREST
constitutional mandate requiring particular description of
– an official making an arrest may take from the person
the things to be seized. Search warrants of similar
arrested any money or property found upon his possession
description were considered null and void for being too
which was used in the commission of the crime or could be
general.
the fruit of the crime or which might be furnished by the
prisoners with the means of committing violence or
(2) No. Notwithstanding the irregular issuance of the
escaping or which may be used in evidence in the trial of
Search Warrant and although, ordinarily, the articles
the accused. These exceptions are applicable to instances
seized under an invalid search warrant should be
where there was a warrant for the arrest of a person
returned, they cannot be ordered returned. Some
sought to be taken into custody issued by the court. When
searches may be made without a warrant. Thus, Section
this warrant is served upon the person, the arresting officer
12, Rule 126, Rules of Court: A person charged with an
may search him even without a search warrant.
offense may be searched for dangerous weapons or
anything which may be used as proof of the commission
Two-Fold Purpose of this principle: (1) To protect the
of the offense.
arresting officer against physical harm from the person

From the Discussions of Atty. Enrique Jr. Bonocan


University of Mindanao College of Law
REVIEWER ON CONSTITUTIONAL LAW II | CYNDI SATORRE-BICERA | 2nd Sem | S.Y. 2021-2022 9

(3) NO. Considering that ROQUE has been charged with NOTE: Mere suspicion is never enough. But the conduct
Rebellion, which is a crime against public order; that the of the officers turned the mere suspicion into a probable
warrant for her arrest has not been served for a cause.
considerable period of time; that she was arrested within
the general vicinity of her dwelling; and that the search of
2.1. SEARCH OF A MOVING VEHICLE
her dwelling was made within a half hour of her arrest,
the search at her place not need a search warrant; this,
for possible effective results in the interest of public order. Valmonte vs. De Villa, G.R. No. 83988, 9/29/1989
Such being the case, the personalities seized may be
retained by CSG, for possible introduction as evidence in • NCRDC with the mission of conducting security
the Rebellion case, leaving it to ROQUE to object to their operations within its area of responsibility and peripheral
relevance and to ask the Special Military Commission to areas installed checkpoints in various parts of
return to her any and all irrelevant documents and Valenzuela, Metro Manila as part of its duty to maintain
articles. peace and order in the NCR. Petitioners alleged that
because of the installation of said checkpoints, the
residents of Valenzuela are worried of being harassed
2. SEARCHES FOR MOVING VEHICLES
and of their safety being placed at the arbitrary,
capricious and whimsical disposition and blanket
Papa v. Mago, G.R. No. L-27360, February 28, 1968 authority of the military manning the checkpoints,
considering that their cars and vehicles are being
• The counterintelligence unit of the Manila Police subjected to regular searches and check-ups, especially
Department, received reliable information that a certain at night or at dawn, without the benefit of a search
shipment of personal effects, allegedly misdeclared and warrant and/or court order in violation of the Constitution.
undervalued, would be released from the customs zone
of the port of Manila and loaded on two trucks. Petitioner HELD: General allegation without stating the details of
Ricardo Papa, Chief of Police and a duly deputized agent the incidents which amount to a violation of right against
of the BOC, conducted surveillance at gate No. 1. unlawful search and seizure, is not sufficient to enable
When the trucks left, elements of the counter-intelligence the Court to determine whether there was a violation. Not
unit went after the trucks and intercepted them. The load all searches and seizures are prohibited; those which are
of the two trucks consisting of nine bales of goods, and reasonable are not forbidden. A reasonable search is not
the two trucks, were seized. Upon investigation, a person to be determined by any fixed formula but is to be
claimed ownership of the goods and showed to the resolved according to the facts of each case. True, the
policemen a "Statement and Receipts of Duties Collected manning of checkpoints by the military is susceptible of
in Informal Entry No. 147-5501", issued by the BOC in abuse by the men in uniform, in the same manner that all
the name of a certain Bienvenido Naguit. Claiming to governmental power is susceptible of abuse. But, at the
have been prejudiced by the seizure and detention of the cost of occasional inconvenience, discomfort and even
two trucks and their cargo, Remedios Mago and Valentin irritation to the citizen, the checkpoints during these
B. Lanopa filed with the CFI a petition "for mandamus abnormal times, when conducted within reasonable
with RO/PI” alleging, among others, that the goods limits, are part of the price we pay for an orderly and a
seized by members of the Manila Police Department was peaceful society.
without a search warrant issued by a competent court.
Q: Does the law require prior announcement to the
HELD: The search and seizure was VALID. Section 2203
public before installing checkpoints?
of the Tariff and Customs Code authorizes persons
A: No, there is no need for checkpoints to be announced.
having police authority to enter, pass through or search
Not only would it be impractical, it would also forewarn
any land, inclosure, warehouse, store or building, not
those who intend to violate the ban. Even so, badges of
being a dwelling house; and also to inspect, search and
legitimacy of checkpoints may still be inferred from their
examine any vessel or aircraft and any trunk, package, or
fixed location and the regularized manner in which they are
envelope or any person on board, or to stop and search
operated. SC ruled that not all checkpoints are illegal.
and examine any vehicle, beast or person suspected of
Those which are warranted by the exigencies of public
holding or conveying any dutiable or prohibited article
order and are conducted in a way least intrusive to
introduced into the Philippines contrary to law, without
motorists are allowed. For, admittedly, routine checkpoints
mentioning the need of a search warrant in said cases.
do intrude, to a certain extent, on motorists’ right to "free
Simply put, there is an express statutory provision under
passage without interruption," but it cannot be denied that,
the law. To require the law enforcer to apply for a search
as a rule, it involves only a brief detention of travelers
warrant prior to the conduct of the search would be
during which the vehicle’s occupants are required to
impractical because of mobility and the vehicle can be
answer a brief question or two. For as long as the vehicle
quickly moved out of the locality or jurisdiction in which
is neither searched nor its occupants subjected to a body
the warrant must be sought. However, as emphasized by
search, and the inspection of the vehicle is limited to a
the SC in Almeida Sanchez v. US (1973), automobile or
visual search, said routine checks cannot be regarded as
not, there must be probable cause for the search.
violative of an individual’s right against unreasonable
search. In fact, these routine checks, when conducted in a
In this case, the law enforcement had actually received
fixed area, are even less intrusive. [People V. Escaño]
a tip from an informant that triggered their suspicion and
conducted a surveillance wherein they gathered
LIMITS IN THE CONDUCT OF CHECKPOINTS
additional information that strengthened their belief that
indeed, some crimes were actually committed.
(a) where the officer merely draws aside the curtain of a

From the Discussions of Atty. Enrique Jr. Bonocan


University of Mindanao College of Law
REVIEWER ON CONSTITUTIONAL LAW II | CYNDI SATORRE-BICERA | 2nd Sem | S.Y. 2021-2022 10

vacant vehicle which is parked on the public fairgrounds;


indiscriminate seizures nor to extend a general
(b) simply looks into a vehicle;
exploratory search made solely to find evidence of
(c) flashes a light therein without opening the car’s doors;
defendant's guilt. The "plain view" doctrine is usually
[a,b,c – Valmonte vs. De Villa]
applied where a police officer is not searching for
(d) where the occupants are not subjected to a physical or
evidence against the accused, but nonetheless
body search;
inadvertently comes across an incriminating object.
(e) where the inspection of the vehicles is limited to a
visual search or visual inspection; and
(f) where the routine check is conducted in a fixed area.
People vs. Valdez G.R. No. 129296, 9/25/2000
If the conduct of checkpoints is merely limited to above
allowable acts, such searches are less intrusive and • Valdez was charged for violating Sec. 9 of R.A. No.
cannot as yet be considered as violative of Section 2, 6425 as amended by R.A. No. 7659 for allegedly caught
Article III, 1987 Constitution. Never could a vehicle be in flagrante delicto and without authority of law, planted,
legally subjected to extensive search unless there is cultivated and cultured (7) fully grown Indian Hemp. He
probable cause to believe that the vehicle is being used in was arraigned and pleaded not guilty. The witnesses (5
the commission of a crime. Anything illegal obtained during policemen) testified that Valdez admitted that the
the search will be considered admissible. marijuanas was his. They uprooted, took photos of
Valdez standing beside the cannabis, and arrested him
The existence of probable cause justifying the warrantless (WITHOUT A WARRANT). Valdez claimed that he was
search is determined by the facts of each case such as: weeding his vegetable farm when he was called by an
the smell of marijuana emanated from a plastic bag owned unidentified person and was asked to go with the latter to
by the driver, or where the accused was acting see something and 5 armed policemen were present and
suspiciously, and attempted to flee. The mere mobility of they made him stand in front of the hemp plants. He was
those vehicles does not give the police authorities then asked if he knew anything about the marijuana
unlimited discretion to conduct indiscriminate searches growing there. When he denied SPO2 Libunao poked a
without warrant.” – [Aniag vs COMELEC] fist at him and told him to admit ownership of the plants.
Valdez contends that there was unlawful search.
PLAIN VIEW DOCTRINE – an illicit object may be seized
in plain view. Objects falling in plain view of an officer, who HELD: 1There was no S.W. issued by a judge after
has the right to be in the position to have that view, are personal determination of the existence of P.C. given the
subject to seizure and may be introduced in the case. The fact that police had ample time to obtain S.W. The
search must be inadvertent — “Kumbaga di ka confiscated plants were evidently obtained during an
naghahanap pero kusa mo na lang nakikita”. illegal search and seizure. Thus, they cannot be used as
evidence against appellant for being the proverbial fruit of
If an officer encountered prohibited objects only after the poisonous tree. The mantle of protection extended
poking around, the discovery would not be considered by the B.O.R. covers both innocent and guilty alike
inadvertent. In the case of People vs. Tabar the SC said against any form of high-handedness of law enforcers,
where Marijuana sticks fall before the eyes of a police regardless of the praise-worthiness of their intentions.
2
officer, the seizure of those sticks would not of course Before an accused may be convicted of a crime, the
require a warrant. prosecution must establish by proof beyond reasonable
doubt that a crime was committed and that the accused
is the author thereof (fundamental). The evidence must
PEOPLE V. MUSA, (G.R. No. 96177 1/27/1993 not only stand the test of reason, it must also be credible
(Evidence in plain view doctrine) and competent – “generally admissible” evidence — .
“of such a character that the court or judge is bound to
• Citing Ker vs. US (marijuana was visible) in this case, it receive it, that is, allow it to be introduced at trial”. 3The
was not; they had no clue of its contents. If inadvertently Constitution decrees that, “In all criminal prosecutions,
they came across the plastic bag because it was within the accused shall be presumed innocent until the
their "plain view," what may be said to be the object in contrary is proved.” To justify the conviction, the
their "plain view" was just the plastic bag and not the prosecution must adduce that quantum of evidence
marijuana. sufficient to overcome the constitutional presumption of
innocence. The prosecution must stand or fall on its
The U.S. SC stated the following limitations on the evidence and cannot draw strength from the weakness of
application of the doctrine: the evidence for the accused. Absent the required
degree of proof of guilt, he is entitled to an acquittal.
What the "plain view" cases have in common is that the
police officer in each of them had a 1prior justification for
an intrusion in the course of which he 2inadvertently People of the Philippines vs. Idel Aminnudin
came across a piece of evidence incriminating the
accused. The doctrine serves to supplement the prior • Aminnudin was arrested shortly after embarking from
justification — whether it be a warrant for another object, M/V Wilcon 9 at around 8:30 PM in Iloilo City on June 25,
hot pursuit, search incident to lawful arrest, or some 1984 by the Officers who acted on a “tip” received 2 days
other legitimate reason for being present unconnected prior. He was accosted, inspected his bag and found
with a search directed against the accused — and articles which look like marijuana leaves, and took him to
permits the warrantless seizure. Of course, the extension their HQ for investigation. The articles were confiscated
of the original justification is legitimate only where it is and later taken to the NBI lab for examination. When they
immediately apparent to the police that they have were verified as marijuana leaves, an information for
evidence before them; The "plain view" doctrine may not, violation of the Dangerous Drugs Act was filed against
however, be used to launch unbridled searches and

From the Discussions of Atty. Enrique Jr. Bonocan


University of Mindanao College of Law
REVIEWER ON CONSTITUTIONAL LAW II | CYNDI SATORRE-BICERA | 2nd Sem | S.Y. 2021-2022 11

him. Eventually he was convicted. In his defense,


Aminnudin disclaimed the marijuana, contending that all • Cesar Masamlok surrendered to the authorities and
he had in his bag was his clothing. He insisted he did not pointed Ruben Burgos (accused) as a member of the
even know what marijuana looked like and that his NPA who threatened to kill him and his family if he
business was selling watches and sometimes cigarettes. refused to join. The police then formed a task force to
He also contended that his bag was confiscated without arrest Burgos. They went to Burgos’ residence where
a search warrant. they saw him plowing his field. They arrested him and
recovered from his house a caliber .38 revolver buried
HELD: The search and the arrest conducted is under the ground. The arrest was made without any
UNLAWFUL and in violation of Article III, Section 2 of the warrant or at least a search warrant. He was also not
Constitution. There was no warrant of arrest or search reminded of his constitutional rights. According to
warrant issued by a judge after personal determination Burgos, he was not a member of the NPA. The gun was
by him of the existence of probable cause. The officers actually buried by Masamlok himself a few days before
had at least two days within which they could have the arrest without the former’s knowledge since he was
obtained a warrant to arrest and search Aminnudin. His not in his house then. It was only his wife who was
name was known, the vehicle was identified, the date of present and she was threatened by Masamlok not to
its arrival was certain, and from the information they had report the gun to the authorities. After his warrantless
received, they could have persuaded a judge that there arrest, he disclosed that he was tortured for days to
was probable cause to justify the issuance of the warrant, admit the ownership of the recovered revolver.
yet they did nothing. No effort was made to comply with
the law. Amminudin was not, at the moment of his arrest, HELD: The warrantless arrest of Burgos is not justified.
committing a crime nor was it shown that he was about to The exceptions in Sec. 6, Rule 113 of ROC must be
do so or that he had just done so. What he was doing strictly construed. There is no compelling reason for the
was descending the gangplank of the vessel and there haste and the failure to first go through the process of
was no outward indication that called for his arrest. To all obtaining a warrant of arrest, if indeed they had
appearances, he was like any of the other passengers reasonable grounds to believe that the accused had
innocently disembarking from the vessel. It was only committed a crime. Under Section 6(a) of Rule 113, the
when the informer pointed to him as the carrier of the officer arresting a person who has just committed, is
marijuana that he suddenly became suspect and so committing, or is about to commit an offense must have
subject to apprehension. It was the furtive finger that personal knowledge of that fact. The offense must also
triggered his arrest. The Identification by the informer be committed in his presence or within his view. There is
was the probable cause as determined by the officers no such personal knowledge in this case. Whatever
(and not a judge) that authorized them to pounce upon knowledge was possessed by the arresting officers, it
Aminnudin and immediately arrest him. The marijuana came in its entirety from the information furnished by
leaves here were excluded as evidence. Cesar Masamlok. The location of the firearm was given
by the appellant’s wife. At the time of the appellant’s
arrest, Burgos was not in actual possession of any
People vs Tangliben G.R. NO. 63630, April 06, 1990 firearm or subversive document. Neither was he
committing any act which could be described as
• At around 9:30 in the evening the Patrolmen noticed a subversive. He was, in fact, plowing his field at the time
person carrying a red traveling bag who was acting of the arrest. They were still fishing for evidence of a
suspiciously and they confronted him; that the person crime not yet ascertained. The subsequent recovery of
was requested by Patrolman Quevedo and Punzalan to the subject firearm on the basis of information from the
open the red traveling bag but the person refused, only to lips of a frightened wife cannot make the arrest lawful
accede later on when the patrolmen identified and the firearm is inadmissible for being the proverbial
themselves; that found inside the bag were marijuana fruit of the poisonous tree.
leaves wrapped in plastic wrapper and weighing one kilo,
more or less. The counsel for the accused stated that the
marijuana allegedly seized from the accused was a PEOPLE VS. MARIACOS
product of an unlawful search without a warrant and is
therefore inadmissible in evidence. • Mariacos was arrested after she was carrying a bag
alleged to have prohibited drugs inside. The bag, before
HELD: The contention is devoid of merit. This case falls it came into her possession, was found inside a
squarely on the exceptions under Section 12 of Rule 126 passenger jeepney with no owner so the policeman
of the ROC and Rule 113, Sec. 5(a). In contrast to the looked inside it only to find packs of marijuana. The
case of Aminnudin, here presents urgency. There was an policeman was acting on a report made about the bag by
informer who pointed to Tangliben. Faced with such an agent of the Barangay Intelligence Network. Mariacos
on-the-spot information, the police officers had to act invoked Sec. 2, Art. 3.
quickly. There was not enough time to secure a search
warrant. To require search warrants during on-the-spot HELD: The Court opines that the invocation of Section 2,
apprehensions of drug pushers, illegal possessors of Article III of the Constitution is misplaced. At the time,
firearms, jueteng collectors, smugglers of contraband when PO2 Pallayoc looked into the contents of the
goods, robbers, etc. would make it extremely difficult, if suspicious bags, there was no identified owner. He asked
not impossible to contain the crimes with which these the other passengers atop the jeepney but no one knew
persons are associated. who owned the bags. Thus, there could be no violation of
the right when no one was entitled thereto at that time.
The search was conducted in a moving vehicle. Time and
People v. Burgos GR No. L-68955 9/4/1986 again, a search of a moving vehicle has been justified on

From the Discussions of Atty. Enrique Jr. Bonocan


University of Mindanao College of Law
REVIEWER ON CONSTITUTIONAL LAW II | CYNDI SATORRE-BICERA | 2nd Sem | S.Y. 2021-2022 12

the ground that the mobility of vehicles makes it possible


to move out of the locality or jurisdiction in which the • Officers conducted a surveillance on Compacion’s
warrant must be sought. Thus, under the facts, PO2 residence (on a confidential tip supplied by an informant
Pallayoc could not be expected to secure a search that accused was growing and cultivating marijuana
warrant in order to check the contents of the bags which plants). During the surveillance, they saw 2 tall plants in
were loaded on top of the moving jeepney. Otherwise, a the backyard of the accused which they suspected as
search warrant would have been of no use because the marijuana plants. The team proceeded to the house
motor vehicle had already left the locality. The search in despite failure to obtain a warrant. The prosecution
this case is valid. The vehicle that carried the contraband contends that the accused opened the gate and
or prohibited drugs was about to leave. PO2 Pallayoc permitted them to come in. He was immediately asked
had to make a quick decision and act fast. It would be about the suspected marijuana plants and he admitted
unreasonable to require him to procure a warrant before that he planted and cultivated the same for the use of his
conducting the search under the circumstances. Time wife who was suffering from migraine. However,
was of the essence in this case. Compacion’s version stated that while he and his family
NB: Always consider if there is urgency. were sleeping, he heard somebody knocking outside his
house. After he opened the gate, 4 persons whom he
thought were members of the military went inside his
PP vs. Jack Racho y Raquero G.R. No. 186529 house. One of them told him to sit in the living room.
Some went upstairs while the others went around the
• Racho was charged with violation of RA 9165. He was house. None of them asked for his permission to search
arrested alighting from Genesis bus info he relayed to the his house.
confidential agent whom he transacted with for the
purchase of shabu. RTC’s verdict is guilty beyond HELD: The search and seizure conducted was not
reasonable doubt and CA affirmed. authorized by a search warrant. Neither does it fall within
the exceptions. The accused’ right against unreasonable
HELD: Racho can no longer question the validity of his search was clearly violated. It is extant from the records
arrest, but the subsequent warrantless search is not that he’ did not consent to the warrantless search and
valid. He is deemed to have waived his right to question seizure conducted. While such rights may be waived,
the validity of his arrest when he never objected to the either expressly or impliedly, such waiver must constitute
irregularity of his arrest before his arraignment. It is his a valid waiver made voluntarily, knowingly and
first time to raise the issue. He actively participated in the intelligently. The act of the accused in allowing the
trial of the case and voluntarily submitted to the members of the military to enter his premises and his
jurisdiction of the court. The sachet of shabu seized from consequent silence during the unreasonable search and
him during the warrantless search is inadmissible in seizure could not be construed as voluntary submission
evidence against him. It was confiscated during a or an implied acquiescence especially so when members
warrantless search incidental to an unlawful arrest. There of the raiding team were intimidatingly numerous and
was no sufficient probable cause to effect a valid heavily armed. His implied acquiescence, if any, could
warrantless arrest because appellant herein did not not have been more than mere passive conformity given
perform some overt act that would indicate that he has under coercive or intimidating circumstances and is, thus,
committed, was actually committing, or was attempting to considered no consent at all within the purview of the
commit an offense. The tip or “reliable information” alone constitutional guarantee. Consequently, herein
is not sufficient to justify a warrantless arrest. accused-appellants lack of objection to the search and
seizure is not tantamount to a waiver of his constitutional
NOTES: A waiver of an illegal, warrantless arrest does right or a voluntary submission to the warrantless search
not carry with it a waiver of the inadmissibility of evidence and seizure.
seized during an illegal warrantless arrest. The 1987
Constitution states that a search and consequent seizure
must be carried out with a judicial warrant; otherwise, it PP V. DE GRACIA, G.R. No. 102009-10 7/6/1994
becomes unreasonable and any evidence obtained DOCTRINE OF EXIGENT CIRCUMSTANCES
therefrom shall be inadmissible for any purpose in any
proceeding. Said proscription, however, admits certain • There were intelligent reports that a building was being
exceptions. Officers had time to secure the warrant. Also used as headquarters by RAM — former rebel soldiers
the information was only coming from a “confidential during the time of Col. Gringo Honasan (1989 coup
agent” not from the officers themselves, they had no d’etat) A surveillance team was coming out of the
personal knowledge. building but the occupants refused to open the door
This circumstance gives rise to another question: despite repeated requests. Indications were there that
whether that information, by itself, is sufficient probable there were large quantities of explosives and ammunition
cause to effect a valid warrantless arrest. The long inside the building. At the time, nearby courts were
standing rule in this jurisdiction is that “reliable closed and there was general chaos and disorder in the
information” alone is not sufficient to justify a warrantless area. The raiding team forcibly entered the building and
arrest. The rule requires, in addition, that the accused conducted the search. In the search, there were
perform some overt act that would indicate that he has recovered several illegal items (firearms used by former
committed, is actually committing, or is attempting to rebels and subversive documents). The soldiers were
commit an offense. criminally charged before the court. In defense, the
members of the RAM captured interposed the argument
that the evidence against them (firearms and
PEOPLE V. COMPACION G.R. 124442, 7 /20/2001 ammunition, etc.) can never be used as evidence
(WAIVER OF RIGHTS) because they are inadmissible.

From the Discussions of Atty. Enrique Jr. Bonocan


University of Mindanao College of Law
REVIEWER ON CONSTITUTIONAL LAW II | CYNDI SATORRE-BICERA | 2nd Sem | S.Y. 2021-2022 13

be introduced as evidence as the accused was in the act


HELD: Under the foregoing circumstances, the instant of committing a crime when apprehended.
case falls under one of the exceptions to the prohibition
against a warrantless search. In the first place, the CASE NO. 2: PEOPLE VS. SOLAYAO — Accused with
military operatives, taking into account the facts obtained his companions fled upon seeing the government
in this case, had reasonable ground to believe that a agents. Under the circumstances, the government
crime was being committed. There was consequently agents could not possibly have procured a search
more than sufficient probable cause to warrant their warrant first. This case constitutes an instance where a
action. Furthermore, under the situation then prevailing, search and seizure may be effected without first making
the raiding team had no opportunity to apply for and an arrest. There was justifiable cause to "stop and
secure a search warrant from the courts because the frisk". The SC underscored probable cause as the basis
courts were closed at that time. Under such urgency of for the conduct of the Stop and frisk.
the moment, the search warrant should lawfully be
dispensed with it. This is known as warrantless search CASE NO. 3: MALACAT VS. CA — The Supreme Court
due to an exigent circumstance. discussed the justification for an allowable scope of a
"stop-and-frisk" as a "limited protective search of outer
clothing for weapons. While probable cause is not
STOP AND FRISK RULE – TERRY SEARCH
required to conduct a "stop and frisk," it nevertheless
holds that mere suspicion or a hunch will not validate a
CASE NO. 1 POSADAS VS. CA "stop and frisk."

FIRST CITED THE TERRY VS. OHIO CASE (1968): In this A "stop-and-frisk" serves a two-fold interest:
case, two men repeatedly walked past a store window and (1) effective crime prevention and detection.
returned to a spot where they apparently conferred with a (2) safety and self-preservation which permit the police
third man. This aroused the suspicion of a police officer. To officer to take steps to assure himself that the person
the experienced officer, the behavior of the men indicated with whom he deals is not armed with a deadly weapon
that they were sizing up the store for an armed robbery. that could unexpectedly and fatally be used against the
When the police officer approached the men and asked police officer.
them for their names, they mumbled a reply. Whereupon,
the officer grabbed one of them, spun him around and COMPARE THIS DOCTRINE W/ PP VS. MENGOTE
frisked him. Finding a concealed weapon in one, he did the
same to the other two and found another weapon. In the
Policemen saw two men "looking from side to side," one
prosecution for the offense of carrying a concealed weapon,
the defense of illegal search and seizure was put up. The
of whom was holding his abdomen. They approached
United States Supreme Court explained that "where a the duo and identified themselves as policemen,
police officer observes unusual conduct which leads him to whereupon the two tried to run away but were unable to
reasonably conclude in the light of his experience that a escape because the other lawmen had surrounded
criminal activity may be afoot that a person with whom he them. The suspects were then searched. One of them,
may be dealing with is armed and dangerous. Wherein the who turned out to be the accused-appellant, was found
course of the investigation of this behavior he identifies with a .38 caliber Smith and Wesson revolver with six
himself as a policeman and makes reasonable inquiries live bullets in the chamber. His companion, later
and when nothing in the initial stages of encounter serves identified as Nicanor Morellos, had a fan knife recovered
to dispel his reasonable fear for his own or others safety, he from their possession. The weapons were taken from
is entitled for the protection of himself and others in the them and used as evidence in a prosecution for the
area, a carefully limited search of the outer clothing of such possession of unlicensed firearms.
person in an attempt to discover weapons which must be
used to assault him, such a search is reasonable." HELD: There is no question that evidence obtained as a
result of an illegal search or seizure is inadmissible in
• Two policemen on foot patrol along Magallanes St. in any proceeding for any purpose. The police officers
DC noticed a person who was walking so fast and was cannot lean on probable cause when proceeding with
holding a buri bag. When the duo tried to approach him, the search there was no offense involved from
he hurriedly sped up by running as fast as he could. looking side to side and the other one is holding the
When he was captured, a .38 caliber unlicensed revolver abdomen. The essential requisites of probable cause
was found and he was charged with PD1866 must still be satisfied before the warrantless search and
(possession of unlicensed firearm). During trial, the seizure can be had. Probable cause in this case must
prosecution offered the revolver as evidence; the only be based on reasonable grounds of suspicion or
accused questioned the admissibility of the revolver belief that a crime has been committed or is about to be
invoking Section 2 article 3, saying that it can’t be used committed. It is not however decided by the Judge in this
as evidence for being the fruit of the poisonous tree. case as required by Section 2, Article 3 but by the
searching police officers.
HELD: The revolver is admissible as evidence. While it
is true that the apprehending policemen cannot yet
validly interpose probable cause when they tried to the
accused. Nevertheless, when the law tried to catch the
attention of the accused, and he subsequently ran INSTANCES WHERE AN ARREST MAY BE VALIDLY
apparently out of fear as he was aware that he MADE WITHOUT A WARRANT OF ARREST
possessed something illicit, then the situation was
sufficient enough for probable cause to apply. Thus, the GR: No one can be illegally arrested without warrant.
policemen were justified when they searched him XPNs: Section 5 Rule 113 Rules of Court
without a warrant and that the unlicensed firearm may
Section 5. Arrest without warrant; when lawful. — A peace

From the Discussions of Atty. Enrique Jr. Bonocan


University of Mindanao College of Law
REVIEWER ON CONSTITUTIONAL LAW II | CYNDI SATORRE-BICERA | 2nd Sem | S.Y. 2021-2022 14

officer or a private person may, without a warrant, arrest a


person: he allegedly shouted are not slanderous, threatening or
abusive, and thus, could not have tended to disturb the
(a) When, in his presence, the person to be arrested has peace or excite a riot. All told, no P.C. exists to justify
committed, is actually committing, or is attempting to commit Ramon’s warrantless arrest. Arrest should not be
an offense; – IN FLAGRANTE DELICTO exercised in a whimsical manner, else a person’s liberty
be subjected to ubiquitous abuse. As Law enforcers, it is
*Crime must be witnessed by the arresting officer. [People vs largely expected of them to conduct a more circumspect
Posadas – he was arrested for commiting the crime of PD assessment of the situation.
1866]

(b) When an offense has just been committed, and he has Q: Does prior surveillance or “Test Buy” is necessary
probable cause to believe based on personal knowledge of for the validity of a buy bust operation in drug cases?
facts or circumstances that the person to be arrested has
committed it; and – HOT PURSUIT A: No, SC said that it was not required. The court
categorically stated in the case of Ermico vs. People that
*Arresting officer must have personal knowledge by way of the the prior surveillance or Test buy is not required for a valid
information obtained from the eye witness. [People vs Jayson buy bust operation, for as long as the authority is
– the arresting policemen had personal knowledge from the accompanied by their informant. Thus, the court said in
eye witnesses who pointed at the bouncer minutes after he People vs Francisco Manlangit that simple as the rule
tried to flee the scene.] that the absence of prior surveillance of test buy does not
affect the legality of the buy bust operation there is no
(c) When the person to be arrested is a prisoner who has textbook method of conducting buy bust operation, the
escaped from a penal establishment or place where he is court has left to the discretion to the police authorities for
serving final judgment or is temporarily confined while his the selection of defective means to apprehend drug
case is pending, or has escaped while being transferred from
dealers. The prior surveillance was less lengthy one is not
one confinement to another. ESCAPEE
necessary especially where the police operatives are
In cases falling under paragraph (a) and (b) above, the person accompanied by their informant during the entrapment,
arrested without a warrant shall be forthwith delivered to the flexibility is a trade of good police work that when time is of
nearest police station or jail and shall be proceeded against in the essence the police may dispense with need for prior
accordance with Sec. 7, Rule 112. surveillance.

NOTE: • A warrantless arrest effected 3 months after the Atty Bonocan: Proper conduct of buy bust operation
commission of the crime is already invalid. It must be a would require that the poseur buyer must be an organic
regular filing as opposed to inquest proceedings. member of the law enforcement organization. It would be
enough that an informant comes to the police and then the
• For continuing crimes a warrantless arrest may still be police immediately thereafter will conduct a buy bust
applied invoking Section 5(1), Rule 113. Even if the one operation by making use of one its operatives to act as a
who is in the possession of the stolen item is not the one poseur buyer with the drugs marked money and will
who stole it, nonetheless he can still be filed with a criminal perform the orchestrated purchase. In that case when the
case in violation of the anti-fencing law. person is arrested by the drug dealer upon handing over
the illicit merchandise, the poseur buyer along with the
BUY BUST OPERATION – is a form of entrapment. This other operatives may now effect the necessary arrest and
is different from instigation because it is the means by there would be no constitutional violation in that situation.
which the accused is lured into the commission of the
offense charged in order to prosecute him. On the other FAJARDO VS. PEOPLE, G.R. NO. 190889
hand, entrapment is the employment of such ways and “The object must be open to the eye and hand its
means for the purpose of trapping or capturing a discovery must be INADVERTENT”
lawbreaker.
ILLUSTRATIVE CASES: • Members of PISO were instructed to respond to the
complaint of some concerned citizens, that there were
*PEOPLE VS. MARTINEZ G.R. No. 198694 accordingly armed men drinking liquor at the residence of
Elenita Fajardo, and who were indiscriminately firing.
• Officers of Station Anti-Illegal Drugs of the Malate Upon the officers arrival they notice that several persons
Police arrested Ramon for purportedly violating Section run in different direction, the responding team saw one
844 of Manila City Ordinance which punishes breaches Valerio, who was holding two .45 caliber pistols, he fired
of the peace when he shouted "Putanginamo! Limang shots to the policeman before entering the house of
daan na ba ito?" He was asked to empty his pockets Fajardo, then Fajardo was seen tucking a .45 between
and there recovered from him a small transparent plastic her waist and the waistband of her shorts. After which
sachet containing shabu. It was confiscated and Ramon she entered the house and locked the main door. The
was brought to the police station. officers cordoned the perimeter of the house as they
waited for further instructions, this time SP02 Nava who
HELD: The act of shouting in a thickly-populated place, was posted at back portion of the house of Fajardo saw
with many people conversing with each other on the Valerio emerged twice on top of the house and throw
street, would constitute any of the acts punishable under something, SP02 Nava and others who acted as a
Section 844 of the Ordinance. Ramon was not making or witness recovered the said object which turn out to be
assisting in any riot, affray, disorder, disturbance, or two (2) receivers of a .45 caliber pistol. Thus, they were
breach of the peace; he was not assaulting, beating or convicted of illegal possession of firearms and
using personal violence upon another; and, the words ammunition under the plain view doctrine. On appeal,

From the Discussions of Atty. Enrique Jr. Bonocan


University of Mindanao College of Law
REVIEWER ON CONSTITUTIONAL LAW II | CYNDI SATORRE-BICERA | 2nd Sem | S.Y. 2021-2022 15

Fajardo insisted on acquittal; she claimed that the the electric company and the inspectors in accordance
discovery of two receivers does not come in the purview with Art 32 of the new civil code, among other grounds
of the plain view doctrine. he claimed that the inspectors entry into his residence is
not justified and was in violation of Section 2, Article 3 of
HELD: The seizure of the 2 .45 pistols outside the the 1987 constitution because the meter inspectors were
petitioner’s house falls within the purview of the plain not armed with a search warrant.
view doctrine. The presence of SPO2 Nava at the back
of the house and of the other law enforcers around the HELD: The constitutional guarantee against unlawful
premises was justified by the fact that Fajardo and searches and seizures is intended as a restraint against
Valerio were earlier seen holding .45 caliber pistols the Government and its agents tasked with law
before they ran inside the house and sought refuge. The enforcement. It is to be invoked only to ensure freedom
attendant circumstances and their evasive actions when from arbitrary and unreasonable exercise of State power.
the law enforcers arrived engendered a reasonable The Court has made this clear in its pronouncements,
ground for the latter to believe that a crime was being including that made in People v. Marti. If the search is
committed. There was thus sufficient probable cause for made upon the request of law enforcers, a warrant must
the policemen to cordon off the house as they waited for generally be first secured if it is to pass the test of
daybreak to apply for a search warrant. SP02 Nava constitutionality. However, if the search is made at the
clearly saw on two different instances that Valerio behest or initiative of the proprietor of a private
emerged on top of the subject dwelling and threw establishment for its own and private purposes, as in the
suspicious objects. Thus, he had reasonable grounds to case at bar, and without the intervention of police
believe that the things thrown might be contraband items, authorities, the right against unreasonable search and
or evidence of the offense they were suspected of seizure cannot be invoked for only the act of private
committing. individual, not the law enforcers, is involved. In sum, the
The ensuing recovery of the receivers was inadvertent, it protection against unreasonable searches and seizures
is not crucial that the initial citing that the seized cannot be extended to acts committed by private
contraband may be identified and known; all that is individuals so as to bring it within the ambit of alleged
merely required that the law enforcer observes that the unlawful intrusion by the government
seized item may be evidence of a crime or a contraband
subject of seizure.
NOTE: The constitutional protection found in Section 2,
article 3 of the constitution can only be invoked against the
However, the liability for their possession of the receivers
state NOT AGAINST PRIVATE INDIVIDUALS. If we talk
should fall only for Valerio and not for Fajardo. Fajardo
about unreasonable searches and seizures.
was neither in physical nor constructive possession of
the subject receivers. The testimony of SPO2 Nava
clearly bared that he only saw Valerio on top of the house *People vs Cogaed GR NO. 200334, July 30, 2014
when the receivers were thrown. None of the witnesses
saw the petitioner holding the receivers, before or during • Cogaed and Dayao were intercepted in a checkpoint
their disposal. At the very least, Fajardo’s possession initiated by PSI Bayan who "received a text message
of the receivers was merely incidental because from an unidentified civilian informer" that one Marvin
Valerio, the one in actual physical possession, was seen Buya (a.k.a. Marvin Bugat) "would be transporting
at the rooftop of petitioner’s house. Absent any marijuana" from Brgy. Lun-oy to Poblacion. When asked
evidence pointing to petitioner’s participation, about the contents of their bags, both replied that they
knowledge or consent in Valerio’s actions, she did not know since they were transporting the bags as a
cannot be held liable for illegal possession of the favor for their barriomate named Marvin. Both were
receivers. arrested and in the police station they were requested to
empty their bags. Inside Cogaed’s sack was "4 rolled
Requirements to legally hold firearm: pieces of suspected marijuana fruiting tops," and inside
1. LTOP – License to own and possess firearm; Dayao’s yellow bag was a brick of suspected marijuana.
2. License of the firearms; Both were criminally charged but Dayao was dismissed
3. Permit to carry firearms outside residence. for being a 14-year-old minor and Cogaed was convicted.

HELD: The Court held that the marijuana bricks and


*SESBREÑO V. CA G.R. No. 160689, 3/26/2014 fruiting tops are INADMISSIBLE as evidence. The search
involved in this case was initially a "stop and frisk"
• Meter inspectors of an electric company who after search, but it did not comply with all the requirements of
obtaining permission from the housemaid of a particular reasonability. It is the police officer who should observe
residence were allowed entry to the premises of the facts that would lead to a reasonable degree of suspicion
house to inspect the electric meter registered in the of a person but in this case it was the driver. The police
account of Sesbreno who was not around at the time, the officer should not adopt the suspicion initiated by another
inspectors noticed that the electric meter is turned upside person. This is necessary to justify that the person
down handing and its disc not rotating. Pursuant to the suspected be stopped and reasonably searched.
Electric Service Commission Contract, the meter was Anything less than this would be an infringement upon
replaced with a new one and subject to a laboratory test. one’s basic right to security of one’s person and effects.
Then the Electric company then sent Sesbreno a bill on There was not a single suspicious circumstance in this
the unrecorded consumption. (most likely what Sesbreno case, and there was no approximation for the probable
did was use a jumper that will enable him to consume cause requirement for warrantless arrest. The person
amounts of electricity without being recorded in the searched was not even the person mentioned by the
meter) Sesbreno filed a complaint for damages against informant. Even if it was true that Cogaed responded by

From the Discussions of Atty. Enrique Jr. Bonocan


University of Mindanao College of Law
REVIEWER ON CONSTITUTIONAL LAW II | CYNDI SATORRE-BICERA | 2nd Sem | S.Y. 2021-2022 16

saying that he was transporting the bag to Marvin Buya, COMERCIANTE vs PEOPLE GR NO. 205926
this still remained only as one circumstance. This is not
enough reason to search Cogaed and his belongings HELD: Section 2, Article III mandates that a search and
without a valid search warrant. None of the other seizure must be carried out through or on the strength of
exceptions to warrantless searches exist to allow the a judicial warrant predicated upon the existence of
evidence to be admissible. The facts of this case do not probable cause; in the absence of such warrant, such
qualify as a search incidental to a lawful arrest. There search and seizure becomes, as a general rule,
can be no valid waiver of Cogaed’s constitutional rights "unreasonable" within the meaning of said constitutional
even if he did not object when the police asked him to provision.
open his bags. Cogaed’s silence or lack of aggressive
objection was a natural reaction to a coercive To protect people from unreasonable searches and
environment brought about by the police officer’s seizures, Section 3 (2), Article III of the Constitution
excessive intrusion into his private space. The provides for an exclusionary rule which instructs that
prosecution and the police carry the burden of showing evidence obtained and confiscated on the occasion of
that the waiver of a constitutional right is one which is such unreasonable searches and seizures are deemed
knowing, intelligent, and free from any coercion. In all tainted and should be excluded for being the proverbial
cases, such waivers are not to be presumed. fruit of a poisonous tree.

Concept of the right to privacy: in general meaning is


SECTION III, ARTICLE III
the right to be alone, the right to be let alone.
(Privacy of Communication and Correspondence)
HING VS CHOACHUY GR NO 179736
SECTION 3. (1) The privacy of communication and
“The right to privacy is the right to be let alone”
correspondence shall be inviolable except upon lawful
order of the court, or when public safety or order requires
otherwise as prescribed by law. The supreme court in this case explained that the right
to privacy is enshrined in our Constitution and in our
laws. It is defined as “the right to be free from
(2) Any evidence obtained in violation of this or the
unwarranted exploitation of one’s person or from
preceding section shall be inadmissible for any purpose in
intrusion into one’s private activities in such a way
any proceeding. — EXCLUSIONARY RULE.
as to cause humiliation to a person’s ordinary
sensibilities.” It is the right of an individual “to be
NOTE: This constitutional right can only be invoked free from unwarranted publicity, or to live without
against the state, not against a private individual. unwarranted interference by the public in matters in
which the public is not necessarily concerned.”
”Invasion of communication and correspondence is a kind Simply put, the right to privacy is “the right to be let
of search” – Fr. Bernas. alone.”

Ex: The use of postal mail – under this constitutional rule, The Bill of Rights guarantees the people’s right to
nobody can intrude in the privacy of the letter by forcibly privacy and protects them against the State’s abuse of
opening the sealed envelope. Even Verbal Communication power. In this regard, the State recognizes the right of
(Telephone, Landline) – Note that the traditional telephone the people to be secure in their houses. No one, not
(only the person holding the hand held telephone can hear even the State, except “in case of overriding social need
what is on the other line) Under the law, it is regarded as and then only under the stringent procedural
private communication. IIn the same manner of a Smartphone safeguards,” can disturb them in the privacy of their
— (tayo lang nakakarinig to secure the conversation) Even homes
the text messages – they are intended for one receiver only.
Unless you are using a loudspeaker, You cannot legally blame
anyone around who was able to overhear the conversation. As a matter of fact, Article 26 (1) of the NCC states:
Do not use the speaker mode, because you have the
reasonable expectation that the communication is private. Every person shall respect the dignity, personality, privacy
and peace of mind of his neighbors and other persons. The
following and similar acts, though they may not constitute a
Under RA 4200, Anti Wiretapping Act, to secretly criminal offense, shall produce a cause of action for
overhear, intercept, or record such communication done in damages, prevention and other relief:
among others via telephone is criminally punishable
(1) Prying into the privacy of another's residence;
GR: The constitution protects our communication, even
correspondence, to be an inviolable right. So that the state ∙ This part of the civil code seeks to protect an individual's
cannot just intrude into these matters without the lawful right to privacy, and provides legal remedy against abuses
court order, or an express provision of the law. that may be committed by other individuals.

EFFECT OF THE NON-OBSERVANCE OF THIS RULE: ∙ This does not mean that only the residence is entitled to
Any evidence obtained in violation of this rule shall be privacy, because the law also covers similar acts – in
inadmissible for any purpose in any proceeding. Stated other words, a business establishment and an office is
otherwise, the evidence obtained in violation of this likewise entitled to the same privacy when the public is
principle will be considered as the fruit of the poisonous excluded therefrom, and only such individuals allowed
tree. may come in.

Thus, an individual’s right to privacy under Article 26(1) of

From the Discussions of Atty. Enrique Jr. Bonocan


University of Mindanao College of Law
REVIEWER ON CONSTITUTIONAL LAW II | CYNDI SATORRE-BICERA | 2nd Sem | S.Y. 2021-2022 17

the Civil Code should not be confined to his house or (1) the right not to have private information disclosed, and
residence as it may extend to places where he has the (2) the right to live freely without surveillance and intrusion.
right to exclude the public or deny them access. The
phrase “prying into the privacy of another’s residence,” Two-fold Test in determining whether or not a matter is
therefore, covers places, locations, or even situations entitled to the right to privacy:
which an individual considers private. And as long as his
right is recognized by society, other individuals may not 1. subjective test – where one claiming the right must
infringe on his right to privacy. - [Hing vs Choachuy] have an actual or legitimate expectation of privacy over a
certain matter.
∙ Our right to privacy is one of the highest forms of human 2. objective test – where his or her expectation of privacy
rights. must be one society is prepared to accept as objectively
∙ The concept of privacy also involves another relative reasonable.
concept of Zones of Privacy

REPUBLIC ACT No. 4200 : AN ACT TO PROHIBIT


DISINI V. SECRETARY OF JUSTICE GR No.203335 AND PENALIZE WIRE TAPPING AND OTHER
(Discussion on Zones of Privacy) RELATED VIOLATIONS OF THE PRIVACY OF
COMMUNICATION, AND FOR OTHER PURPOSES.
HELD: Only those persons who have deliberately
broadcasted in public libelous remarks through the use Section 1. It shall be unlawful for any person, not being
of the internet are criminally liable. The Zones of authorized by all the parties to any private communication
privacy are recognized and protected in our laws. Within or spoken word, to tap any wire or cable, or by using any
these zones, any form of intrusion is impermissible other device or arrangement, to secretly overhear,
unless excused by law and in accordance with intercept, or record such communication or spoken word by
customary legal process. The meticulous regard we using a device commonly known as a dictaphone or
accord to these zones arises not only from our dictagraph or dictaphone or walkie-talkie or tape recorder,
conviction that the right to privacy is a "constitutional or however otherwise described:
right" and "the right most valued by civilized men," but
It shall also be unlawful for any person, be he a participant
also from our adherence to the Universal Declaration of or not in the act or acts penalized in the next preceding
Human Rights which mandates that, "no one shall be sentence, to knowingly possess any tape record, wire
subjected to arbitrary interference with his privacy" and record, disc record, or any other such record, or copies
"everyone has the right to the protection of the law thereof, of any communication or spoken word secured
against such interference or attacks." either before or after the effective date of this Act in the
manner prohibited by this law; or to replay the same for
In assessing the challenge that the State has any other person or persons; or to communicate the
impermissibly intruded into these zones of privacy, a contents thereof, either verbally or in writing, or to furnish
court must determine whether a person has exhibited a transcriptions thereof, whether complete or partial, to any
reasonable expectation of privacy and, if so, whether other person: Provided, that the use of such record or any
that expectation has been violated by unreasonable copies thereof as evidence in any civil, criminal investigation
or trial of offenses mentioned in section 3 hereof, shall not
government intrusion.
be covered by this prohibition.
The concept of liberty would be emasculated if it does
not likewise compel respect for his personality as a Q: Is RA 4200 redundant with Section 3 Article III?
unique individual whose claim to privacy and interference A: No. Section 3 (Article III) of the constitution provides for
demands respect. In other words, our laws, especially the general rule and the for the exclusionary rule. In RA
the constitution, will always seek to protect our human 4200, not only does it prohibit and exclude evidence
right to privacy, especially on communication. [Morfe vs illegally obtained but it also imposes criminal liability. This is
Mutuc (1968)] another safeguard to enforce our constitutional right.

Constitutional guarantees that create zones of privacy: Gaanan v. IAC G.R. No. L-69809, 16 October 1986

(a) the right against unreasonable searches and seizures, • Gaanan listened to the conversation on the extension
which is the basis of the right to be let alone (the beginning line (as instructed by Laconico) with Atty. Pintor, who
of all freedom); and [Section 2, Article 3 & Section 3(1)] proposed a condition that they would withdraw the
complaint for direct assault if they would pay 8,000 to
which he agreed. When he brought the money as
(b) right to privacy of communication and correspondence.
instructed by Pintor, he was arrested. The following day,
Gaanan executed an affidavit stating that he heard
Father Bernas in his book added that the right to privacy is Pintor demand 8,000 for the withdrawal of the case.
but an aspect of the right to be secured in one’s person. Laconico attached the affidavit and charged them for
robbery and extortion. Atty. Pintor also charged Gaanan
Q: How is Privacy Classified? In the case of Whalen vs of violation of the Anti-wiretapping law. On appeal, they
Roe, decided by the US Supreme Court in 1997, The court argued that the telephone extension is not covered by
classified privacy into two categories: the term device under RA 4200.

1. Decisional Privacy – involves the right to independence HELD: While the conversations were indeed private, the
in making certain important decisions law refers to a "tap" of a wire or cable or the use of a
2. Informational Privacy – refers to the interest in avoiding "device or arrangement" for the purpose of secretly
disclosure of personal matters. — has two aspects: [Disini]

From the Discussions of Atty. Enrique Jr. Bonocan


University of Mindanao College of Law
REVIEWER ON CONSTITUTIONAL LAW II | CYNDI SATORRE-BICERA | 2nd Sem | S.Y. 2021-2022 18

overhearing, intercepting, or recording the makes no distinction. The statute's intent to penalize all
communication. There must be either a physical persons unauthorized to make such recording is
interruption through a wiretap or the deliberate underscored by the use of the qualifier "any" “even a
installation of a device or arrangement in order to person privy to a communication who records his private
overhear, intercept, or record the spoken words. An conversation with another without the knowledge of the
extension telephone cannot be placed in the same latter will qualify as a violator" under RA 4200.
category as a dictaphone, dictograph or the other
devices enumerated in Section 1 of RA No. 4200 as the
use thereof cannot be considered as "tapping" the wire Zulueta v. Court of Appeals, G.R. No. 107383
or cable of a telephone line. The telephone extension in
this case was not installed for that purpose. It just • Cecilia Zulueta suspected her husband Dr. Alfredo
happened to be there for ordinary office use. The Martin-Zulueta to have an affair. At some point, Cecilia
purpose there was for convenience and not to violate the entered the clinic of her husband and in the presence of
anti wire-tapping law. her mother, a driver and private respondent's secretary,
forcibly opened the drawers and cabinet in her husband's
clinic and took 157 documents consisting of private
Additional Cases on Privacy of Communication and correspondence between Dr. Martin and his alleged
Correspondence: paramours, greetings cards, canceled checks, diaries,
Dr. Martin's passport, and photographs. The documents
Salcedo–Ortañez v. CA G.R. No. 100662, 08/04/1994 and papers were seized for use in evidence in a case for
legal separation and for disqualification from the practice
• Rafael Ortanez filed with the RTC for annulment of of medicine which she had filed against her husband.
marriage with damages against his wife on the grounds
of lack of marriage license and physical capacity of HELD: The documents and papers in question are
Teresita. On trial, Rafael presented 3 cassette tapes inadmissible in evidence. Spouses are not exempted
containing alleged recordings of conversation with from the constitutional injunction declaring "the privacy of
Teresita and an unidentified person. Teresita interposed communication and correspondence to be inviolable".
an objection but the trial court denied it as well as her The only exception to the prohibition in the Constitution is
motion for reconsideration. She appealed to the CA via a if there is a "lawful order from a court or when public
petition via certiorari but affirmed the decision of the safety or order requires otherwise, as prescribed by law."
RTC. Thus, she elevated the case to SC. Any violation of this provision renders the evidence
obtained inadmissible "for any purpose in any
HELD: The tapes could not be admissible as evidence. proceeding."
In fact, Rep. Act No. 4200 entitled "An Act to Prohibit
and Penalize Wire Tapping and Other Related Violations The intimacies between spouses do not justify any one of
of the Privacy of Communication, and for other them in breaking the drawers and cabinets of the other
purposes" expressly makes such tape recordings and in ransacking them for any telltale evidence of
inadmissible in evidence. Apart from the fact of course marital infidelity. A person, by contracting marriage, does
that said act considered it a crime. Clearly, respondents, not shed his/her integrity or his right to privacy as an
the Trial court and Court of Appeals failed to consider the individual and the constitutional protection is ever
afore-quoted provisions of the law in admitting in available to him or her.
evidence the cassette tapes in question. Absent a clear
showing that both parties to the telephone conversations
allowed the recording of the same, the inadmissibility of People v. Andre Marti, G.R. No. 81561, 1/18/1991
the subject tapes is mandatory under Rep. Act No. 4200.
• Andre Marti was charged for violation of Dangerous
Drugs RA 9262 when his package was delivered by his
Ramirez v. Court of Appeals, G.R. No. 93833 common-law wife, Shirley to the booth of the "Manila
Packing and Export Forwarders" was withheld by the
• Socorro Ramirez filed with the RTC a civil case alleging owners when they inspected it following and SOP and
that one Ester Garcia in a confrontation in the latter's suspected from a peculiar odor emitted that the
office, allegedly vexed, insulted and humiliated her in a packaged contained a contraband to which they called
"hostile and furious mood" and in a manner offensive to the NBI to assist. Marti contends that he was illegally
petitioner's dignity and personality," contrary to morals, searched and seized by the NBI.
good customs and public policy. In support of her claim,
petitioner produced a verbatim transcript of the event HELD: The evidence was obtained by a private person,
culled from a tape recording of the confrontation made by acting in a private capacity and without the intervention
Ramirez as a result of Ramirez’s recording of the event in and participation of State authorities. In the absence of
secret, Garcia filed a criminal case before the RTC of governmental interference, the liberties guaranteed by
Pasay for violation of RA 4200. Ramirez argued that the the Constitution cannot be invoked against the State
provision of RA 4200 does not apply to the taping of a since the contraband having come into possession of the
private conversation by one of the parties in the Government without the latter transgressing appellant's
conversation. rights against unreasonable search and seizure.

HELD: RA 4200 clearly and unequivocally makes it The Bill of Rights embodied in the Constitution is not
illegal for any person, not authorized by all the parties to meant to be invoked against acts of private individuals.
any private communication to secretly record such The constitutional proscription against unlawful searches
communication by means of a tape recorder. The law and seizures therefore applies as a restraint directed only

From the Discussions of Atty. Enrique Jr. Bonocan


University of Mindanao College of Law
REVIEWER ON CONSTITUTIONAL LAW II | CYNDI SATORRE-BICERA | 2nd Sem | S.Y. 2021-2022 19

Ortega where a physician, Dr. Ortega, investigated for


against the government and its agencies tasked with the
mismanagement of the psychiatric residency program,
enforcement of the law. Thus, it could only be invoked
sexual harassment of female hospital employees and other
against the State to whom the restraint against arbitrary
irregularities searched his office and seized personal items
and unreasonable exercise of power is imposed.
from his desk and filing cabinets. The Court categorically
declared that "individuals do not lose Fourth Amendment
NOTE: The contraband is not a “communication”.
rights merely because they work for the government instead
of a private employer." A plurality of four Justices concurred
that the correct analysis has two steps: first, because "some
Waterous Drugs v. NLRC, G.R. 113271, 10/16/1997 government offices may be so open to fellow employees or
the public that no expectation of privacy is reasonable", a
• Antonia Catolico, a pharmacist employed by Waterous court must consider "the operational realities of the
Drugs, was involved in various irregularities. She was workplace" in order to determine whether an employee’s
being investigated for overpricing of the medicines of Fourth Amendment rights are implicated; and next, where
Waterous Drugs. After confirmation from one of the an employee has a legitimate privacy expectation, an
company’s clients it was found that she accepted a check employer’s intrusion on that expectation "for non
under her name in the amount of P640 (as a investigatory, work-related purposes, as well as for
refund/kickback). The cheque was discovered by Ms. investigations of work-related misconduct, should be judged
Saldana (employee) of the company. Catolico questioned by the standard of reasonableness under all the
the act of Saldana in opening the envelope. Later circumstances."
Catolico was dismissed and she questioned the
admissibility of the check for having been searched and In US v. Mark L. Simons, Simons, an employee of CIA,
seized without warrant. was convicted of receiving and possessing materials
containing child pornography despite the search and
HELD: The arguments tendered by Catolico and NLRC seizure done in his office. Because he already knew that
were both wrong. The Court found no reason to revise The policy also stated that users shall understand that the
the doctrine laid down in People v. Marti that the Bill of agency will periodically audit, inspect, and/or monitor the
Rights does not protect citizens from unreasonable user’s Internet access as deemed appropriate.
searches and seizures perpetrated by private individuals.
It is not true, as counsel for Catolico claims, that the The preceeding US jurisprudence were discussed in
citizens have no recourse against such assaults. On the the case below.
contrary, and as said counsel admits, such an invasion
gives rise to both criminal and civil liabilities.
Briccio A. Pollo v. Karina Constantino-David

Our present constitutional provision on the guarantee FACTS: Pollo is a former Supervising Personnel
against unreasonable search and seizure had its origin in Specialist of the CSC Regional Office No. IV and also the
the 1935 Charter Sec 1(3), Article. This was in turn derived Officer-in-Charge of the Public Assistance and Liaison
almost verbatim from the Fourth Amendment to the United Division (PALD) under the "Mamamayan Muna Hindi
States Constitution. As such, the Court may turn to the Mamaya Na" program of the CSC.
pronouncements of the US Supreme Court and Appellate
Courts which are considered doctrinal in this jurisdiction. On January 3, 2007 at around 2:30 p.m., an unsigned
letter-complaint addressed to respondent CSC
In the 1967 case of Katz v. United States, SC held that the Chairperson Karina Constantino-David which was
act of FBI agents in electronically recording a conversation marked "Confidential" and sent through a courier service
made by petitioner in an enclosed public telephone booth (LBC) from a certain "Alan San Pascual" of Bagong
violated his right to privacy and constituted a "search and Silang, Caloocan City, was received by the Integrated
seizure". Because the petitioner had a reasonable Records Management Office (IRMO) at the CSC Central
expectation of privacy in using the enclosed booth to make Office. Following office practice in which documents
a personal telephone call, the protection of the Fourth marked "Confidential" are left unopened and instead sent
Amendment extends to such an area. In the concurring to the addressee, the aforesaid letter was given directly
opinion of Mr. Justice Harlan, it was further noted that the to Chairperson David.
existence of privacy right under prior decisions involved a
two-fold requirement: first, that a person has exhibited an The letter contained and questioned if Pollo is allowed to
actual (subjective) expectation of privacy; and second, that be “a lawyer of an accused gov’t employee having a
the expectation be one that society is prepared to recognize pending case in the CSC”.
as reasonable (objective).
David conducted an investigation and a few days later,
the computers of Pollo was secured and the 17 diskettes
In Mancusi v. DeForte, which addressed the reasonable
contained by 42 documents were drafts, pleadings, in
expectations of private employees in the workplace, the US
connections with
Supreme Court held that a union employee had Fourth
Based on that, Pollo was administratively charged and
Amendment rights with regard to an office at union
later dismissed from the service. Pollo appealed the
headquarters that he shared with other union officials, even
dismissal to the SC invoking the O’Connor doctrine.
as the latter or their guests could enter the office. The Court
thus "recognized that employees may have a reasonable
ISSUE: Was there a violation of his right to privacy?
expectation of privacy against intrusions by police."
HELD: No, there was none. failed to prove that he had
The Fourth Amendment equally applies to a government an actual (subjective) expectation of privacy either in his
workplace was addressed in the 1987 case of O’Connor v. office or government-issued computer which contained

From the Discussions of Atty. Enrique Jr. Bonocan


University of Mindanao College of Law
REVIEWER ON CONSTITUTIONAL LAW II | CYNDI SATORRE-BICERA | 2nd Sem | S.Y. 2021-2022 20

Q: What are the four basic rights under the free speech
his personal files. Petitioner did not allege that he had a
clause?
separate enclosed office which he did not share with
anyone, or that his office was always locked and not
open to other employees or visitors. Neither did he allege Section 4. No law shall be passed abridging the 1freedom
that he used passwords or adopted any means to of speech, of 2expression, or 3of the press, or the 4right of
prevent other employees from accessing his computer the people peaceably to assemble and petition the
files. On the contrary, he submits that being in the public government for redress of grievances.
assistance office of the CSC-ROIV, he normally would
have visitors in his office like friends, associates and FRANCISCO CHAVES VS. RAUL GONZALES, G.R.
even unknown people, whom he even allowed to use his NO. 168338, 15 February 2008
computer which to him seemed a trivial request. He
described his office as "full of people, his friends, Q: Explain the nature of the free speech clause.
unknown people" and that in the past 22 years he had
been discharging his functions at the PALD, he is
Freedom of expression has gained recognition as a
"personally assisting incoming clients, receiving
fundamental principle of every democratic government
documents, drafting cases on appeals, in charge of
and it has been given a status of a preferred right that
accomplishment report, Mamamayan Muna Program.
stands on a higher level than substantive economic
freedom or the other liberties. In one case the SC said
Petitioner’s claim of violation of his constitutional right to
that the cogent rights codified by Article 3, Section 4 of
privacy must necessarily fail. His other argument
the Constitution copied almost verbatim from the First
invoking the privacy of communication and
Amendment of the US Bill of Right where considered the
correspondence under Section 3(1), Article III of the 1987
necessary consequence of Republican institution and the
Constitution is also untenable considering the recognition
complement of free speech. This preferred status of free
accorded to certain legitimate intrusions into the privacy
speech has also been codified at the international level,
of employees in the government workplace under the
its recognition now enshrined in international law as a
aforecited authorities. We likewise find no merit in his
customary norm that binds all nations. In this jurisdiction
contention that O’Connor and Simons are not relevant
(PH) the primacy and highest (esteem?) accorded
because the present case does not involve a criminal
freedom of expression is a fundamental postulate of our
offense like child pornography. As already mentioned, the
Constitutional system. This right is elevated to
search of petitioner’s computer was justified there being
Constitutional status in the 1935, 1973, and 1987
reasonable ground for suspecting that the files stored
Constitution reflecting our own lesson of history both
therein would yield incriminating evidence relevant to the
political and legal that freedom of speech is an
investigation being conducted by CSC as government
indispensable condition for nearly every other form of
employer of such misconduct subject of the anonymous
freedom.
complaint. This situation clearly falls under the exception
to the warrantless requirement in administrative searches
For it is only when the people have unbridled access to
defined in O’Connor.
information and the press that they will be capable of
rendering enlightened judgments. In the oft-quoted words
of Thomas Jefferson, we cannot both be free and
ignorant.
MANILA ELECTRIC COMPANY, ET AL V. ROSARIO
GOPEZ LIM, G.R. NO. 184769, 10/05/2010 Q: What kind of ideas may be expressed under the
free speech clause?
Rules on Habeas Data
The ideas that may be expressed under this freedom are
May an employee invoke the remedies available under
confined not only to those that are conventional or
such writ where an employer decides to transfer her
acceptable to the majority. To be truly meaningful,
workplace on the basis of copies of an anonymous letter
freedom of speech and of the press should allow and
posted therein ─ imputing to her disloyalty to the
even encourage the articulation of the unorthodox view,
company and calling for her to leave, which imputation it
though it be hostile to or derided by others; or though
investigated but fails to inform her of the details thereof?
such view "induces a condition of unrest, creates
dissatisfaction with conditions as they are, or even stirs
Rosario G. Lim (respondent), also known as Cherry Lim,
people to anger." To paraphrase Justice Holmes, it is
is an administrative clerk at the Manila Electric Company
freedom for the thought that we hate, no less than for the
(MERALCO).
thought that agrees with us.
On June 4, 2008, an anonymous letter was posted at the
door of the Metering Office of the Administration building Freedom of speech and of the press means something
of MERALCO Plaridel, Bulacan Sector, at which more than the right to approve existing political beliefs or
respondent is assigned, denouncing respondent. The economic arrangements, to lend support to official
letter reads: measures, and to take refuge in the existing climate of
opinion on any matter of public consequence. When
atrophied, the right becomes meaningless. The right
Read Republic Act 10173: Data Privacy Act of 2012 belongs as well -- if not more – to those who question,
who do not conform, who differ.
(Section 4, Article III)
Q: What is the scope of the free speech clause?
“The Free Speech Clause”

From the Discussions of Atty. Enrique Jr. Bonocan


University of Mindanao College of Law
REVIEWER ON CONSTITUTIONAL LAW II | CYNDI SATORRE-BICERA | 2nd Sem | S.Y. 2021-2022 21

political rights. In fact, it was always held that no society


can remain free, open, and democratic without FOE. FOE
The scope of freedom of expression is so broad that it guarantees full, spirited and even contentious discussion of
extends protection to nearly all forms of communication. all social, economic, and political issues and to survive, a
It protects speech, print and assembly regarding secular free and democratic society must zealously safeguard FOE.
as well as political causes, and is not confined to any FOE even allows the citizens to expose and check abuses
particular field of human interest. The protection covers of public officials, to make informed choices of candidates
myriad matters of public interest or concern embracing all for a public office,allows the competition of ideas, the clash
issues, about which information is needed or appropriate, of claims and counterclaims from which the truth will likely
so as to enable members of society to cope with the emerge, allows the erring of social grievances, mitigating
exigencies of their period. The constitutional protection sudden eruptions of violence from marginal groups who
assures the broadest possible exercise of free speech otherwise would not be heard by the government, provides
and free press for religious, political, economic, scientific, civilized way of engagement among political, ideological,
news, or informational ends, inasmuch as the religious or ethnic opponents. If one cannot use his tongue
Constitution's basic guarantee of freedom to advocate to argue, he might use his fist instead. FOE is the freedom
ideas is not confined to the expression of ideas that are to disseminate ideas and beliefs whether competing,
conventional or shared by a majority. informing or otherwise. It is the freedom to express to
others what one likes or dislikes as it is the freedom of
The constitutional protection is not limited to the others to express to one and all what they favor or disfavor;
exposition of ideas. The protection afforded free speech it's the free expression for the ideas we love or hate.
extends to speech or publications that are entertaining as Indeed, the function of FOE is to stir disputes.
well as instructive or informative.
It may indeed best serve its high purpose when the
Q: What are the limits and restraints of free speech? exercise of FOE induces a condition of unrest, creates
dissatisfaction with conditions as they are or even stirs
The freedom of expression is not absolute nor is it an people to anger because speech is often provocative and
unbridled license. Therefore, all speech are not treated challenging, it may strike at prejudices and preconceptions
the same. and have profound and unsettling effects as it presses for
acceptance of an idea. Prohibits for enactment of any law
that curtails the FOE. [Dissenting Opinion of J. Carpio]
For freedom of expression is not an absolute, nor is it an
"unbridled license that gives immunity for every possible
use of language and prevents the punishment of those Q: What is prior restraint?
who abuse this freedom." A: According to Fr. Bernas, means the official governmental
restrictions on the press or other forms of expressions IN
ADVANCE (BEFORE) of actual publication or
Thus, all speech are not treated the same. Some types of dissemination. Its most blatant form is the system of
speech may be subjected to some regulation by the licensing administered by the Executive department. Ex.
State under its pervasive police power, in order that it movie censorship (must be reviewed by MTRCB). Another
may not be injurious to the equal right of others or those form is the Judicial prior restraint (that may come in a form
of the community or society. The difference in treatment of injunctive writ - oppositors of the film filed an injunction
is expected because the relevant interests of one type of case); license taxes measured by gross receipts for the
speech, e.g., political speech, may vary from those of privilege of engaging in the business of advertising in any
another, e.g., obscene speech. Distinctions have news paper (1936 case of C American Press: according to
therefore been made in the treatment, analysis, and the oppositors, the requirement of the Government of
evaluation of the permissible scope of restrictions on license is a form of an invalid prior restraint but the US SC
various categories of speech. We have ruled, for held that its valid because they are not talking about the
example, that in our jurisdiction slander or libel, lewd and contents of the newspapers but for the advertisements
obscene speech, as well as "fighting words" are not where they can gain profit)
entitled to constitutional protection and may be
penalized. Flat licenses for the privilege of selling religious goods –
where the SC invalidated the Manila ordinance imposing
Q: Cite the four aspects of the freedom of the press. license charges and taxes upon the ABS the rate of which
was based on the gross sales of their religious goods. It is a
Philippine jurisprudence, even as early as the period clear violation of ABS constitutional freedom of the press
under the 1935 Constitution, has recognized four aspects [American Bible Society v City of Manila]
of freedom of the press. These are:
(1) freedom from prior restraint; Prior restraint is generally understood as an imposition in
(2) freedom from punishment subsequent to publication; advance of a limit upon speech or other forms of
(3) freedom of access to information; and expression. In determining whether a restriction is a prior
(4) freedom of circulation. restraint, one of the key factors considered is whether the
restraint prevents the expression of a message. In
Nebraska Press Association v Stuart (1976), prior restraint
Q: Discuss the relationship between freedom of by definition has an immediate and irreversible sanction if it
expression (Section 4), prior restraint and subsequent can be said that a threat of criminal or civil sanction after
punishment. publication 'chills' speech, prior restraint 'freezes' it at least
for the time.
Jurisprudence teaches that FOE is the foundation of a free,
open, and democratic society. FOE is an indispensable The term prior restraint is used to describe administrative
condition to the exercise of almost all other civic and and judicial orders forbidding certain communications when

From the Discussions of Atty. Enrique Jr. Bonocan


University of Mindanao College of Law
REVIEWER ON CONSTITUTIONAL LAW II | CYNDI SATORRE-BICERA | 2nd Sem | S.Y. 2021-2022 22

issued in advance of the time that such communication is to


Article IV of the Constitution;
occur. Prior restraint can be both valid (RPC punishing
slander, oral defamation, libel, cyber libel, inciting to
ISSUE: Was the NIB exercising prior restraint upon the
sedition) or invalid (like in the case of American Bible
press freedom of the petitioners?
Society).
HELD: SC rendered this moot and academic but still
Q: What is subsequent punishment?
answered in the affirmative. It is not Idle to note that
A: The act of punishing the purveyor of illegal speech
ordinarily, an invitation to attend a hearing and answer
AFTER it is published. The exceptions to this rule are
some questions, which the person invited may heed or
pornography, false or misleading advertisement, advocacy
refuse at his pleasure, is not illegal or constitutionally
of imminent lawless action, and danger to national security.
objectionable. Under certain circumstances, however,
The imposition of liability to the individual exercising his
such an invitation can easily assume a different
freedom may be in any form such as penal, civil, or
appearance. Thus, where the invitation comes from a
administrative penalty. Those Penal penalties (mentioned
powerful group composed predominantly of ranking
above) are valid subsequent punishment, but other than
military officers issued at a time when the country has
that, the government cannot impose a subsequent
just emerged from martial rule and when the suspension
punishment for the exercise of the person’s freedom of
of the privilege of the writ of habeas corpus has not
expression.
entirely been lifted and the designated interrogation site
is a military camp, the same can easily be taken, not as a
strictly voluntary invitation which it purports to be, but as
BURGOS V. CHIEF OF STAFF G.R. No. L-64261, 26 an authoritative command which one can only defy at his
December 1984 peril, especially where, as in the instant case, the
invitation carries the ominous seaming that "failure to
FACTS: Two [2] search warrants issued on December 7, appear . . . shall be considered as a waiver . . . and this
1982 by respondent Judge Ernani Cruz-Pano. Committee will be constrained to proceed in accordance
with law."
ISSUE: Was there a prior restraint in this case.

HELD: The premises searched were the business and GERONIMO SANTIAGO V. FAR EASTERN
printing offices of the "Metropolitan Mail" and the "We BROADCASTING, G.R. No. L-48683, 11/04/1941
Forum newspapers. As a consequence of the search and
seizure, these premises were padlocked and sealed, with FACTS: The petitioner argues that the broadcasting of
the further result that the printing and publication of said the speeches of the candidates of the Sumulong Popular
newspapers were discontinued. Front Party in the evening of September 23, 1941, could
not by any means offer any danger to public safety or
Such closure is in the nature of previous restraint or public morality, and that the respondent was not justified
censorship abhorrent to the freedom of the press "to previously censor and approve the complete texts of
guaranteed under the fundamental law, and constitutes the speeches of said candidates allowing the same to be
a virtual denial of petitioners' freedom to express broadcasted in its station."
themselves in print. This state of being is patently
anathematic to a democratic framework where a free, ISSUE: May the Executive Department through the
alert and even militant press is essential for the political Secretary of Interior may issue a regulation requiring
enlightenment and growth of the citizenry. radio stations to submit the speeches 24 hours before it
can be broadcasted (another form of prior restraint)?

BABST V. NATIONAL INTELLIGENCE BOARD, G.R. HELD: Yes, because it is an exercise of the police power.
No. L-62992, 28 September 1984 The SC failed to perceive the cogency of such an
argument. It does not bolster up the case for the
FACTS: Petitioners are columnists, feature article writers petitioner. It impliedly admits — and correctly, we think —
and reporters of various local publications. At different that a speech that may endanger public safety may be
dates since July, 1980, some of them have allegedly censored and disapproved for broadcasting. How could
been summoned by military authorities who have the censor verify the petitioner's claim that the speeches
subjected them to sustained interrogation on various he intended to broadcast offered no danger to public
aspects of their works, feelings, sentiments, beliefs, safety or public morality, if the petitioner refused to
associations and even their private lives. Typical of the submit the manuscript or even the gist thereof? If the
letters received by the petitioners from respondent NIB is petitioner had complied with respondent's requirement
that addressed to petitioner Arlene Babst. and the respondent had arbitrarily and unreasonably
refused to permit said speeches to be broadcasted, he
Petitioners maintain that the respondents have no might have reason to complain. It is not the duty of the
jurisdiction over the proceedings which are violative of respondent as a public service corporation to broadcast
the constitutional guarantee on free expression since speeches without requiring the submission of the
they have the effect of imposing restrictive guidelines and manuscript thereof in advance, but that, on the contrary,
norms on mass media; that they are a punitive ordeal or the laws and regulations expressly authorize the
subsequent punishment of petitioners for lawful respondent to make such a requirement.
publications; that they amount to a system of censorship,
curtailing the "free flow of information and petition and
opinion," indispensable to the right of the people to know Q: Explain the following tests on government
matters of public concern guaranteed in Section 6 of suppression of free speech:

From the Discussions of Atty. Enrique Jr. Bonocan


University of Mindanao College of Law
REVIEWER ON CONSTITUTIONAL LAW II | CYNDI SATORRE-BICERA | 2nd Sem | S.Y. 2021-2022 23

A. Dangerous tendency rule – means that the evil may amount to speech, are subject to lesser but still
consequences of the comment or the utterance must be heightened scrutiny. [NEWSOUNDS BROADCASTING V.
extremely serious and the degree of imminence must be CESAR DY, ET AL]
high before it is punished. The danger to be guarded
against it is the substantive evil sought to be prevented – *Content-based restraint is the invalid one.
the primarily disorderly and unfair administration of justice.
This test establishes a definite rule in the Constitutional law,
it provides the criterion as to what words may be published. NEWSOUNDS BROADCASTING V. CESAR DY, ET AL,
Under this rule, the advocacy of ideas cannot G.R. No. 170270 & 179411, 02 April 2009
constitutionally be abridged unless there is a clear and
present danger that such advocacy will harm the FACTS: In 1996, Newsounds commenced relocation of
administration of justice. its broadcasting stations, management office and
transmitters on property located in Minante, Cauayan
City, Isabela. The property is owned by CBS
B. Clear and present danger rule – has been adopted in
Development Corporation (CDC), an affiliate corporation
cases where extreme difficulty is confronted determining
under the Bombo Radyo network which holds title over
where the freedom of expression ends and the right of
the properties used by Bombo Radyo stations throughout
courts to protect their independence begins. There must be
the country. On 28 June 1996, CDC was issued by the
a remedy to borderline cases and the basic principle of this
then municipal government of Cauayan a building permit
rule lies in that the freedom of speech and of the press, as
authorizing the construction of a commercial
well as the right to petition for redress of grievance, while
establishment on the property. The (HLURB) issued a
guaranteed by the constitution, are not absolute. They are
Zoning Decision certifying the property as commercial.
subject to restrictions and limitations, one of them being the
That same day, the Office of the Municipal Planning and
protection of the courts against contempt. If the words
Development Coordinator (OMPDC) of Cauayan affirmed
uttered create a dangerous tendency which the state has a
that the commercial structure to be constructed by CDC
right to prevent, then such words are punishable. It is not
conformed to local zoning regulations, noting as well that
necessary that some definite or immediate acts of force,
the location "is classified as a Commercial area." Similar
violence, or unlawfulness be advocated. It is sufficient that
certifications would be issued by OMPDC from 1997 to
such acts be advocated in general terms. Nor is it
2001.
necessary that the language used be reasonably calculated
to incite persons to acts of force, violence or unlawfulness.
A building was consequently erected on the property, and
It is sufficient if the natural tendency and probable effect of
therefrom, DZNC and Star FM operated as radio
the utterance be to bring about the substantive evil the
stations. Both stations successfully secured all necessary
utterance be to bring about the substantive evil which the
operating documents, including mayor’s permits from
legislative body seeks to prevent. [G.R. No. L-8974]
1997 to 2001. During that period, CDC paid real property
taxes on the property based on the classification of the
C. Balancing of interest test – even if there is clear and land as commercial.
present danger but to the mind of the court halos
magkapareho ang point or reasons of the opposing parties All that changed beginning in 2002. On 15 January of
(Government vs. the Speaker) the Court will have to that year, petitioners applied for the renewal of the
balance whose reasons is much more beneficial to all. mayor’s permit. petitioners formally requested then City
Under this rule, it rests on the theory that it is the court’s Zoning Administrator-Designate Bagnos Maximo
function in a case before it when it finds public interests (Maximo) to issue a zoning clearance for the property.
served by legislation, on the one hand, and the free Maximo, however, required petitioners to submit "either
expression clause affected by it, on the other, to balance an approved land conversion papers from the
one against the other and arrive at a judgment where the Department of Agrarian Reform (DAR) showing that the
greater weight shall be placed. If, on balance, it appears property was converted from prime agricultural land to
that the public interest served by restrictive legislation is of commercial land, or an approved resolution from the
such nature that it outweighs the abridgment of freedom, Sangguniang Bayan or Sangguniang Panglungsod
then the court will find the legislation valid. In short, the authorizing the reclassification of the property from
balance-of-interests theory rests on the basis that agricultural to commercial land." Petitioners had never
constitutional freedoms are not absolute, not even those been required to submit such papers before, and from
stated in the free speech and expression clause, and that 1996 to 2001, the OMPDC had consistently certified that
they may be abridged to some extent to serve appropriate the property had been classified as commercial.
and important interests. [Soriano v Laguardia, GR No.
164785] Due to this refusal by Maximo to issue the zoning
clearance, petitioners were unable to secure a mayor’s
Q: Distinguish between content-neutral and permit. Petitioners filed a petition for mandamus17 with
content-based regulation of the free speech clause. the Regional Trial Court (RTC) of Cauayan City to
compel the issuance of the 2002 mayor’s permit. RTC of
A: Jurisprudence distinguishes between a content-neutral Cauayan denied petitioners’ accompanying application
regulation, i.e., merely concerned with the incidents of the for injunctive relief, they filed a special civil action for
speech, or one that merely controls the time, place or certiorari with the Court of Appeals, but this would be
manner, and under well defined standards; and a dismissed by the appellate court due to the availability of
content-based restraint or censorship, i.e. the restriction other speedy remedies with the trial court. In February of
is based on the subject matter of the utterance or speech. 2003, the RTC dismissed the mandamus action for being
Content-based laws are generally treated as more suspect moot and academic.19
than content-neutral laws because of judicial concern with
discrimination in the regulation of expression. In the meantime, petitioners sought to obtain from the
Content-neutral regulations of speech or of conduct that

From the Discussions of Atty. Enrique Jr. Bonocan


University of Mindanao College of Law
REVIEWER ON CONSTITUTIONAL LAW II | CYNDI SATORRE-BICERA | 2nd Sem | S.Y. 2021-2022 24

DAR Region II Office a formal recognition of the


conversion of the CDC property from agricultural to However, the circumstances of this case dictate that we
commercial. The matter was docketed as Adm. Case No. view the action of the respondents as a content-based
A-0200A-07B-002. Then DAR Region II Director Abrino restraint.
L. Aydinan (Director Aydinan) granted the application and
issued an Order that stated that "there remains no doubt WHY? Prior to 2002, petitioners had not been frustrated
on the part of this Office of the non-agricultural in securing the various local government requirements
classification of subject land before the effectivity of for the operation of their stations. It was only in the
Republic Act No. 6657 otherwise known as the beginning of 2002, after the election of respondent
Comprehensive Agrarian Reform Law of 1988. On 16 Ceasar Dy as mayor of Cauayan, that the local
January 2003, petitioners filed their applications for government started to impose these new requirements
renewal of mayor’s permit for the year 2003, attaching substantiating the conversion of CDC’s property for
therein the DAR Order. Their application was approved. commercial use. Petitioners admit that during the 2001
However, on 4 March 2003, respondent Felicisimo Meer, elections, Bombo Radyo "was aggressive in exposing the
Acting City Administrator of Cauayan City, wrote to widespread election irregularities in Isabela that appear
petitioners claiming that the DAR Order was spurious or to have favored Respondent Dy and other members of
void. the Dy political dynasty."52 Respondents’ efforts to close
petitioners’ radio station clearly intensified immediately
The controversy continued into 2004. In January of that before the May 2004 elections, where a former employee
year, petitioners filed their respective applications for of DZNC Bombo Radyo, Grace Padaca, was mounting a
their 2004 mayor’s permit, again with the DAR Order credible and ultimately successful challenge against the
attached to the same. A zonal clearance was issued in incumbent Isabela governor, who happened to be the
favor of petitioners. Yet in a letter dated 13 January 2004, brother of respondent Dy. It also bears notice that the
respondent Meer claimed that no record existed of DAR requirements required of petitioners by the Cauayan City
Adm. Case No. A-0200A-07B-002 with the Office of the government are frankly beyond the pale and not
Regional Director of the DAR or with the RCLUPPI.23 As conventionally adopted by local governments throughout
a result, petitioners were informed that there was no the Philippines.
basis for the issuance in their favor of the requisite
zoning clearance needed for the issuance of the mayor’s All those circumstances lead us to believe that the steps
permit. employed by respondents to ultimately shut down
petitioner’s radio station were ultimately content-based.
The deadline for application for the mayor’s permit Being content-based, it is invalid and unconstitutional
lapsed on 15 February 2004, despite petitioners’ plea for under Section 4.
another extension. On 17 February 2004, respondents
Meer and Racma Fernandez-Garcia, City Legal Officer of
Cauayan City, arrived at the property and closed the Q: Distinguish between “void for vagueness” and
radio stations. “overbreadth doctrine”.

ISSUE: Was the closure valid? A: The law itself is invalid because it is vague to the extent
that the people must guess its meaning. It's invalid.
HELD: No, the closure was not valid. That the acts Overbreadth doctrine - there is a law that prohibits
imputed against respondents constitute a prior restraint something which gives a chilling effect. SC will not hesitate
on the freedom of expression of respondents who to strike these laws invalid.
happen to be members of the press is clear enough.
There is a long-standing tradition of special judicial To be sure, the doctrine of vagueness and the doctrine of
solicitude for free speech, meaning that governmental overbreadth do not operate on the same plane.
action directed at expression must satisfy a greater
burden of justification than governmental action directed
at most other forms of behavior. We had said in SWS v. A statute or act suffers from the defect of vagueness when
COMELEC: "Because of the preferred status of the it lacks comprehensible standards that men of common
constitutional rights of speech, expression, and the intelligence must necessarily guess at its meaning and
press, such a measure is vitiated by a weighty differ as to its application. It is repugnant to the Constitution
presumption of invalidity. Indeed, ‘any system of prior in two respects: (1) it violates due process for failure to
restraints of expression comes to this Court bearing a accord persons, especially the parties targeted by it, fair
heavy presumption against its constitutional validity. . . . notice of the conduct to avoid; and (2) it leaves law
The Government 'thus carries a heavy burden of showing enforcers unbridled discretion in carrying out its provisions
justification for the enforcement of such restraint.’ There and becomes an arbitrary flexing of the Government
is thus a reversal of the normal presumption of validity muscle. The overbreadth doctrine, meanwhile, decrees that
that inheres in every legislation." a governmental purpose to control or prevent activities
constitutionally subject to state regulations may not be
The Supreme Court here distinguished content-neutral achieved by means which sweep unnecessarily broadly and
restraint and content-based restraint or censorship. thereby invade the area of protected freedoms.

Ostensibly, the act of an LGU requiring a business of As distinguished from the vagueness doctrine, the
proof that the property from which it operates has been overbreadth doctrine assumes that individuals will
zoned for commercial use can be argued, when applied understand what a statute prohibits and will accordingly
to a radio station, as content-neutral since such a refrain from that behavior, even though some of it is
regulation would presumably apply to any other radio protected (chilling effect). [Southern Hemisphere vs.
station or business enterprise within the LGU. Anti-Terrorism Council]

From the Discussions of Atty. Enrique Jr. Bonocan


University of Mindanao College of Law
REVIEWER ON CONSTITUTIONAL LAW II | CYNDI SATORRE-BICERA | 2nd Sem | S.Y. 2021-2022 25

Q: What is meant by the doctrine of strict scrutiny? invalidation of the questioned statute is warranted to
counter the "chilling effect" on protected speech that comes
In terms of judicial review of statutes or ordinances, strict from its overbreadth as any person may simply restrain
scrutiny refers to the standard for determining the quality himself from speaking or engaging in any partisan political
and the amount of governmental interest brought to justify activity anywhere in order to avoid being charged of an
the regulation of fundamental freedoms. Strict scrutiny is electoral offense. Indeed, an overbroad law that "chills one
used today to test the validity of laws dealing with the into silence" should be invalidated on its face.
regulation of speech, gender, or race as well as other
fundamental rights as expansion from its earlier As applied challenge - na violate mo na and you were
applications to equal protection. The United States criminally charged. That is the time that you will challenge
Supreme Court has expanded the scope of strict scrutiny to the said law.
protect fundamental rights such as suffrage, judicial access
and interstate travel. [White Light Corp vs City of Manila] Simply put, facial challenge – questioning the validity of the
law on its face (vague or overbreadth); you have the right to
Note: The overbreadth and vagueness doctrines then have question it before the Court (Rule 64 – declaratory relief
special application only to free speech cases. They are prior to breach of a law or a contract).
inapt for testing the validity of penal statutes
In both cases, it can’t be used when the law which you seek
As the U.S. Supreme Court put it, in an opinion by Chief to get annulled by the Court are those ordinary penal laws –
Justice Rehnquist, "we have not recognized an do not regulate in any way the exercise of free speech.
'overbreadth' doctrine outside the limited context of the First
Amendment." A facial challenge is allowed to be made to a vague statute
and to one which is overbroad because of possible "chilling
In Broadrick v. Oklahoma, the Court ruled that "claims of effect" upon protected speech. The theory is that "[w]hen
facial overbreadth have been entertained in cases involving statutes regulate or proscribe speech and no readily
statutes which, by their terms, seek to regulate only spoken apparent construction suggests itself as a vehicle for
words" and, again, that "overbreadth claims, if entertained rehabilitating the statutes in a single prosecution, the
at all, have been curtailed when invoked against ordinary transcendent value to all society of constitutionally
criminal laws that are sought to be applied to protected protected expression is deemed to justify allowing attacks
conduct." For this reason, it has been held that "a facial on overly broad statutes with no requirement that the
challenge to a legislative act is the most difficult challenge person making the attack demonstrate that his own conduct
to mount successfully, since the challenger must establish could not be regulated by a statute drawn with narrow
that no set of circumstances exists under which the Act specificity." The possible harm to society in permitting some
would be valid." As for the vagueness doctrine, it is said unprotected speech to go unpunished is outweighed by the
that a litigant may challenge a statute on its face only if it is possibility that the protected speech of others may be
vague in all its possible applications. "A plaintiff who deterred and perceived grievances left to fester because of
engages in some conduct that is clearly proscribed cannot possible inhibitory effects of overly broad statutes.
complain of the vagueness of the law as applied to the
conduct of others." This rationale does not apply to penal statutes. Criminal
statutes have general in terrorem effect resulting from their
In sum, the doctrines of strict scrutiny, overbreadth, and very existence, and, if facial challenge is allowed for this
vagueness are analytical tools developed for testing "on reason alone, the State may well be prevented from
their faces" statutes in free speech cases or, as they are enacting laws against socially harmful conduct. In the area
called in American law, First Amendment cases. They of criminal law, the law cannot take chances as in the area
cannot be made to do service when what is involved is a of free speech.
criminal statute. With respect to such statute, the
established rule is that "one to whom application of a
statute is constitutional will not be heard to attack the
statute on the ground that impliedly it might also be taken
as applying to other persons or other situations in which its SANTIAGO DIVINAGRACIA V. CBS & PBS, G.R. No.
application might be unconstitutional." As has been pointed 162272, 07 April 2009
out, "vagueness challenges in the First Amendment
context, like overbreadth challenges typically produce facial Congress unquestionably has the power to grant and
invalidation, while statutes found vague as a matter of due deny licenses and to eliminate existing stations. No one
process typically are invalidated [only] 'as applied' to a has a First Amendment right to a license or to
particular defendant." monopolize a radio frequency; to deny a station license
because "the public interest" requires it "is not a denial of
free speech."
Q: Distinguish between “facial challenge” and “as
applied challenge” concerning laws regulating the free
The absence of government regulation in that market had
speech clause.
led to the emergence of hundreds of radio broadcasting
stations, each using frequencies of their choice and
A facial challenge is allowed to be made to a vague statute changing frequencies at will, leading to literal chaos on
and to one which is overbroad because of possible "chilling the airwaves. It was the Radio Act of 1927 which
effect" upon protected speech. introduced a licensing requirement for American
broadcast stations, to be overseen eventually by the
In Loida Nicolas-Lewis vs Comelec, the SC said that It Federal Communications Commission (FCC).
may not be amiss to point out, at this juncture, that a facial

From the Discussions of Atty. Enrique Jr. Bonocan


University of Mindanao College of Law
REVIEWER ON CONSTITUTIONAL LAW II | CYNDI SATORRE-BICERA | 2nd Sem | S.Y. 2021-2022 26

This pre-regulation history of radio broadcast stations


illustrates the continuing necessity of a government role Petitioner Iglesia ni Cristo, however, is directed to refrain
in overseeing the broadcast media industry, as opposed from offending and attacking other existing religions in
to other industries such as print media and the Internet. showing "Ang Iglesia ni Cristo" program.
Without regulation, the result would be a free-for-all
market with rival broadcasters able with impunity to INC moved for reconsideration of the said judgment
sabotage the use by others of the airwaves. praying: (a) for the deletion of the second paragraph of
the dispositive portion of the Decision, and (b) for the
Other rationales may have emerged as well validating Board to be perpetually enjoined from requiring petitioner
state regulation of broadcast media, but the reality of to submit for review the tapes of its program which was
scarce airwaves remains the primary, indisputable and granted by the trial court. The board appealed to the CA
indispensable justification for the government regulatory which reversed the decision of the trial court. INC went to
role. The integration of the scarcity doctrine into the the SC crying invalid prior restraint.
jurisprudence on broadcast media illustrates how the
libertarian ideal of the free expression clause may be ISSUE: Was the reversal of the trial court’s decision by
tempered and balanced by actualities in the real world the Court of Appeals correct?
while preserving the core essence of the constitutional
guarantee. Indeed, without government regulation of the HELD: No. The SC reversed the ruling of the Appellate
broadcast spectrum, the ability of broadcasters to clearly Court. First. Deeply ensconced in our fundamental law is
express their views would be inhibited by the anarchy of its hostility against all prior restraints on speech,
competition. Since the airwaves themselves are not including religious speech. Hence, any act that restrains
susceptible to physical appropriation and private speech is hobbled by the presumption of invalidity and
ownership, it is but indispensable that the government should be greeted with furrowed brows. It is the burden
step in as the guardian of the spectrum. of the respondent Board to overthrow this presumption. If
it fails to discharge this burden, its act of censorship will
be struck down. It failed in the case at bar.
IGLESIA NI CRISTO V. CA, G.R. No. 119673, 7/26/96
Second. The evidence shows that the respondent Board
FACTS: Petitioner Iglesia ni Cristo has a television x-rated petitioners TV series for "attacking" either
program entitled "Ang Iglesia ni Cristo" aired on Channel religions, especially the Catholic church. An examination
2. The program presents and propagates petitioner's of the evidence, especially Exhibits "A," "A-1," "B," "C,"
religious beliefs, doctrines and practices often times in and "D" will show that the so-called "attacks" are mere
comparative studies with other religions. criticisms of some of the deeply held dogmas and tenets
of other religions. The videotapes were not viewed by the
Sometime in the months of September, October and respondent court as they were not presented as
November 1992 petitioner submitted to the respondent evidence. Yet they were considered by the respondent
Board of Review for Moving Pictures and Television the court as indecent, contrary to law and good customs,
VTR tapes of its TV program Series Nos. 116, 119, 121 hence, can be prohibited from public viewing under
and 128. The Board classified the series as "X" or not for section 3(c) of PD 1986. This ruling clearly suppresses
public viewing on the ground that they "offend and petitioner's freedom of speech and interferes with its right
constitute an attack against other religions which is to free exercise of religion. It misappreciates the essence
expressly prohibited by law." of freedom to differ.

INC pursued 2 courses of action against the respondent The word attack is not synonymous with the word
Board and it appealed to the Office of the President the "offend." Moreover, Article 201 (2) (b) (3) of the Revised
classification of its TV Series No. 128. It succeeded in its Penal Code should be invoked to justify the subsequent
appeal and the Office of the President reversed the punishment of a show which offends any religion. It
decision of the respondent Board. Forthwith, the Board cannot be utilized to justify prior censorship of speech. It
allowed Series No. 128 to be publicly telecast. must be emphasized that E.O. 876, the law prior to PD
1986, included "attack against any religion" as a ground
December 14, 1992, INC also filed against the for censorship. The ground was not, however, carried
respondent Board a civil case in the RTC. Petitioner over by PD 1986. Its deletion is a decree to disuse it.
alleged that the respondent Board acted without
jurisdiction or with grave abuse of discretion in requiring
petitioner to submit the VTR tapes of its TV program and MTRCB V. ABS-CBN, G.R. No. 155282, 1/17/2005
in x-rating them. It cited its TV Program Series Nos. 115,
119, 121 and 128. In their Answer, respondent Board FACTS: On October 15, 1991, at 10:45 in the evening,
invoked its power under PD No. 1986 in relation to Article respondent ABS-CBN aired "Prosti-tuition," an episode of
201 of the Revised Penal Code. the television (TV) program "The Inside Story" produced
and hosted by respondent Legarda. It depicted female
After trial, the RTC rendered a judgment which students moonlighting as prostitutes to enable them to
dispositive portion of it reads: pay for their tuition fees. In the course of the program,
student prostitutes, pimps, customers, and some faculty
WHEREFORE, judgment is hereby rendered ordering members were interviewed. The Philippine Women’s
respondent Board of Review for Moving Pictures and University (PWU) was named as the school of some of
Television (BRMPT) to grant petitioner Iglesia ni Cristo the students involved and the facade of PWU Building at
the necessary permit for all the series of "Ang Iglesia ni Taft Avenue, Manila conspicuously served as the
Cristo" program. background of the episode.

From the Discussions of Atty. Enrique Jr. Bonocan


University of Mindanao College of Law
REVIEWER ON CONSTITUTIONAL LAW II | CYNDI SATORRE-BICERA | 2nd Sem | S.Y. 2021-2022 27

Apparently, newsreels are straight presentation of events.


The showing of "The Inside Story" caused uproar in the They are depiction of "actualities." Correspondingly, the
PWU community. Its officials, the PWU Parents and MTRCB Rules and Regulations implementing P. D. No.
Teachers Association filed letter-complaints3 with 1986 defines newsreels as "straight news reporting, as
petitioner MTRCB. Both complainants alleged that the distinguished from news analyses, commentaries and
episode besmirched the name of the PWU and resulted opinions. Talk shows on a given issue are not considered
in the harassment of some of its female students. newsreels." Clearly, the "The Inside Story" cannot be
considered a newsreel. It is more of a public affairs
In their answer, respondents explained that the "The program which is described as a variety of news
Inside Story" is a "public affairs program, news treatment; a cross between pure television news and
documentary and socio-political editorial," the airing of news-related commentaries, analysis and/or exchange of
which is protected by the constitutional provision on opinions. Certainly, such kind of program is within
freedom of expression and of the press. Accordingly, petitioner’s review powers of the MTRCB.
petitioner has no power, authority and jurisdiction to
impose any form of prior restraint upon respondents.
IN RE VICENTE SOTTO
The Board (MTRCB) after hearing fined ABS CBN and
required it to submit further and all other programs for the FACTS: Vicente Sotto, who was required by their Court
board’s review before showing. ABS CBN then filed a on December 7, 1948, to show cause why he should not
civil action before the RTC questioning the board’s be punished for contempt to court for having issued a
judgment. The RTC annulled the decision of the board. written statement in connection with the decision of this
Hence, the MTRCB went to the SC. Court in In re Angel Parazo for contempt of court, which
statement, as published in the Manila Times and other
ISSUE: Was the RTC’s ruling correct? daily newspapers of the locality, reads as follows:

HELD: No. "The Inside Story" is a television program, As author of the Press Freedom Law (Republic Act No.
hence, it is within the jurisdiction of the MTRCB over 53.) interpreted by the Supreme Court in the case of
which it has power of review. Here, respondents sought Angel Parazo, reporter of a local daily, who now has to
exemption from the coverage of the term "television suffer 30 days imprisonment, for his refusal to divulge the
programs" on the ground that the "The Inside Story" is a source of a news published in his paper, I regret to say
"public affairs program, news documentary and that our High Tribunal has not only erroneously
socio-political editorial" protected under Section 4, Article interpreted said law, but that it is once more putting in
III of the Constitution. Albeit, respondent’s basis is not evidence the incompetency of narrow mindedness o the
freedom of religion, as in Iglesia ni Cristo, but freedom of majority of its members, In the wake of so many
expression and of the press, the ruling in Iglesia ni Cristo mindedness of the majority deliberately committed during
applies squarely to the instant issue. It is significant to these last years, I believe that the only remedy to put an
note that in Iglesia ni Cristo, this Court declared that end to so much evil, is to change the members of the
freedom of religion has been accorded a preferred status Supreme Court. To his effect, I announce that one of the
by the framers of our fundamental laws, past and first measures, which as its objects the complete
present, "designed to protect the broadest possible reorganization of the Supreme Court. As it is now
liberty of conscience, to allow each man to believe as his constituted, a constant peril to liberty and democracy. It
conscience directs x x x." Yet despite the fact that need be said loudly, very loudly, so that even the deaf
freedom of religion has been accorded a preferred may hear: the Supreme Court very of today is a far cry
status, still this Court, did not exempt the Iglesia ni from the impregnable bulwark of Justice of those
Cristo’s religious program from petitioner’s review power. memorable times of Cayetano Arellano, Victorino Mapa,
Manuel Araullo and other learned jurists who were the
The Court further said that Respondents claim that the honor and glory of the Philippine Judiciary.
showing of "The Inside Story" is protected by the
constitutional provision on freedom of speech and of the Due to this The Supreme Court cited Sotto in contempt of
press. However, there has been no declaration at all by Court.
the framers of the Constitution that freedom of
expression and of the press has a preferred status. In his defense, Sotto justified himself by invoking the free
speech clause. Is he correct?
If this Court, in Iglesia ni Cristo, did not exempt religious
programs from the jurisdiction and review power of The Supreme Court said in the negative. It is true that the
petitioner MTRCB, with more reason, there is no constitutional guaranty of freedom of speech and the
justification to exempt therefrom "The Inside Story" press must be protected to its fullest extent, but license
which, according to respondents, is protected by the or abuse of liberty of the press and of the citizen should
constitutional provision on freedom of expression and of not be confused with liberty in its true sense. As
the press, a freedom bearing no preferred status. important as the maintenance of an unmuzzled press
and the free exercise of the right of the citizen, is the
The only exceptions from the MTRCB’s power of review maintenance of the independence of the judiciary. As
are those expressly mentioned in Section 7 of P. D. No. Judge Holmes very appropriately said: "The
1986, such as (1) television programs imprinted or administration of justice and the freedom of the press,
exhibited by the Philippine Government and/or its though separate and distinct, are equally sacred, and
departments and agencies, and (2) newsreels (actual neither should be violated by the other. The press and
happening is reported i.e. vehicular accident; unlike the courts have correlative rights and duties and should
“Inside Story” that is scripted). cooperate to uphold the principles of the Constitution and

From the Discussions of Atty. Enrique Jr. Bonocan


University of Mindanao College of Law
REVIEWER ON CONSTITUTIONAL LAW II | CYNDI SATORRE-BICERA | 2nd Sem | S.Y. 2021-2022 28

laws, from which the former receives its prerogatives and use irritating language centers not on persuading the
the latter its jurisdiction. The right of legitimate publicity readers but on creating disturbances, the rationable of
must be scrupulously recognized and care taken at all free speech cannot apply and the speaker or writer is
times to avoid impinging upon it. In a clear case where it removed from the protection of the constitutional
is necessary, in order to dispose of judicial business guaranty. In this instance, the attack on the President
unhampered by publications which reasonably tend to passes the furthest bounds of free speech and common
impair the impartiality of verdicts, or otherwise obstruct decency. More than a figure of speech was intended.
the administration of justice, this court will not hesitate to There is a seditious tendency in the words used, which
exercise its undoubted power to punish for contempt. could easily produce disaffection among the people and
This Court must be permitted to proceed with the a state of feeling incompatible with a disposition to
disposition if its business in an orderly manner free from remain loyal to the Government and obedient to the laws
outside interference obstructive of its constitutional
functions. This right will be insisted upon as vital to an
impartial court, and, as a last resort, as a individual PEOPLE VS ISAAC PEREZ GR No. L-21049
exercises the right of self-defense, it will act to preserve
its existence as an unprejudiced tribunal. . . ." FACTS: Isaac Perez, the municipal secretary of Pilar,
Sorsogon and one a citizen of the same municipality
happened to meet at the presidencia of Pilar, they
ESPUELAS VS. PEOPLE GR No. L-2990 became engaged in a discussion regarding the
administration of Governor-General Wood, which
FACTS: Oscar Espuelas had his picture taken, making it resulted in Perez shouting a number of times: "The
to appear as if he were hanging lifeless at the end of a Filipinos, like myself, must use bolos for cutting off
piece of rope suspended form the limb of the tree, when Wood's head for having recommended a bad thing for
in truth and in fact, he was merely standing on a barrel. the Filipinos, for he has killed our independence."
After securing copies of his photograph, Espuelas sent Charged in the CFI of Sorsogon with a violation of article
copies of same to several newspapers and weeklies of 256 of the Penal Code having to do with contempt of
general circulation for their publication with a suicide note ministers of the Crown or other persons in authority, and
or letter, wherein he made to appear that it was written by convicted thereof, Perez has appealed the case to the
a fictitious suicide, Alberto Reveniera and addressed to Supreme Court.
the latter's supposed wife translation of which letter
reads: ISSUE: Must the conviction be reversed on the ground of
free speech.
Dearest wife and children, bury me five meters deep. Over my
grave don't plant a cross or put floral wreaths, for I don't need HELD: No. The Supreme Court said that criticism is
them. Please don't bury me in the lonely place. Bury me in the permitted to penetrate even to the foundations of
Catholic cemetery. Although I have committed suicide, I still Government. Criticism, no matter how severe, on the
have the right to burried among Christians. But don't pray for Executive, the Legislature, and the Judiciary, is within the
me. Don't remember me, and don't feel sorry. Wipe me out of range of liberty of speech, unless the intention and effect
your lives. My dear wife, if someone asks to you why I be seditious. But when the intention and effect of the act
committed suicide, tell them I did it because I was not pleased is seditious, the constitutional guarantees of freedom of
with the administration of Roxas. Tell the whole world about speech and press and of assembly and petition must
this. And if they ask why I did not like the administration of yield to punitive measures designed to maintain the
Roxas, point out to them the situation in Central Luzon, the prestige of constituted authority, the supremacy of the
Leyte. constitution and the laws, and the existence of the State.

Dear wife, write to President Truman and Churchill. Tell them Here, the person maligned by the accused is the Chief
that here in the Philippines our government is infested with Executive of the Philippine Islands. His official position,
many Hitlers and Mussolinis. Teach our children to burn like the Presidency of the United States and other high
pictures of Roxas if and when they come across one. I offices, under a democratic form of government, instead,
committed suicide because I am ashamed of our government of affording immunity from promiscuous comment, seems
under Roxas. I cannot hold high my brows to the world with rather to invite abusive attacks. But in this instance, the
this dirty government. I committed suicide because I have no attack on the Governor-General passes the furthest
power to put under Juez de Cuchillo all the Roxas people now bounds of free speech was intended. There is a seditious
in power. So, I sacrificed my own self. tendency in the words used, which could easily produce
disaffection among the people and a state of feeling
Later it was found out that he was just faking his death. incompatible with a disposition to remain loyal to the
For that, he was criminally charged for scurrilous libels Government and obedient to the laws.
against the Government. In his defense, Espuelas
invoked the free speech clause of the Constitution.
PEOPLE VS JUAN FELEO GR No. L-36429
ISSUE: Should he be acquitted?
FACTS: Juan Feleo was a leader of the communists at
HELD: No. The freedom of speech secured by the the time delivered a speech to the effect:
Constitution "does not confer an absolute right to speak
or publish without responsibility whatever one may “You must imitate the French soldiers who in a battle at . . .
choose." It is not "unbridled license that gives immunity instead of pointing the arms at the enemies, instead of pointing
for every possible use of language and prevents the the arms at them, what they did was to shoot at their chiefs.
punishment of those who abuse this freedom." When the What we want to say is for you to use them (the guns) not

From the Discussions of Atty. Enrique Jr. Bonocan


University of Mindanao College of Law
REVIEWER ON CONSTITUTIONAL LAW II | CYNDI SATORRE-BICERA | 2nd Sem | S.Y. 2021-2022 29

against the communists but against the American government. provide the most accurate and comprehensive means of
We do hope that when the time comes the Constabulary man conveying the proceedings to the public and in
and the scouts would be deserters in order to side with the reds acquainting the public with the judicial process in action;
to defend the Philippines. . . . I Likewise hope the Constabulary nevertheless, within the courthouse, the overriding
men do not have bad conscience possessed by the American consideration is still the paramount right of the accused
imperialist as they (the Constabulary) are under military to due process which must never be allowed to suffer
discipline, not to suppress the sucker, the American diminution in its constitutional proportions. Justice Clark
government, because we consider them oppressed. Imagine the thusly pronounced, "while a maximum freedom must be
salary of a constabulary private, P15, only, and see what the allowed the press in carrying out the important function of
captains do, General Nathorst, Colonel Bowers and others, informing the public in a democratic society, its exercise
how big are their salaries? They receive hundreds and must necessarily be subject to the maintenance of
thousands. But their chiefs are the ones far away from the absolute fairness in the judicial process.
scene of the battle, they are in their offices, and whenever there
is promotion they are the captains and the lieutenants. Those Atty. Bonocan Comment: There seems to be a clash of
who scratch their bellies are the ones who receive big pay. The two (2) constitutional rights i.e. the freedom of the press
soldiers starve to death. . . . My companions, if we all unite vs. Erap’s right to due process. The Supreme Court has
there will be no more trust (?) Belo, there will be no oppressive given more weight to the constitutional right of Erap in
American government and they will go away if we all have a refusing allowance to televise the trial conducted by the
rebellious heart. If we all be united, that captain (referring to SandiganBayan.
the Provincial Commanders S. F. Cacdac) will fall, and we
shall be the owners of those haciendas, the railroad company
and others.” AYER PRODUCTIONS V. CAPULONG G.R. 82380

He was charged, tried, and convicted for seditious libel. FACTS: Right after EDSA Revolution, there was a
His defense during the appeal, he only said it in the project movie entitled “The Four Day Revolution” where
exercise of the constitutionally protected freedom of one of the characters is Juan Ponce Enrile (JPE) who
speech. was the former Minister of National Defense. JPE tried to
block the filming of the movie invoking his right to privacy.
ISSUE: Was the conviction valid?
ISSUE: Was JPE correct?
HELD: It is a well established doctrine that the
constitutional guarantee of the freedom of speech and of HELD: Whether the "balancing of interests test" or the
the press does not give a person an unqualified right to clear and present danger test" be applied in respect of
speak or publish, without responsibility, whatever he may the instant Petitions, the Court believes that a different
choose. That a state in the exercise of its police power conclusion must here be reached: The production and
may punish those who abuse the freedom conferred by filming by petitioners of the projected motion picture "The
the constitutional provision, and whose language tends to Four Day Revolution" does not, in the circumstances of
disturb the public peace, is not open to question. The this case, constitute an unlawful intrusion upon private
words spoken by the appellant on the occasion respondent's "right of privacy."
mentioned incited the hearers to imitate French soldiers The subject matter of "The Four Day Revolution" relates
in battle who, instead of pointing arms at their enemies, to the non-bloody change of government that took place
directed their weapons towards their own chiefs. As an at Epifanio de los Santos Avenue in February 1986, and
end to be accomplished by this course of action it was the trian of events which led up to that denouement.
said that the provincial commander of the Constabulary Clearly, such subject matter is one of public interest and
would fall and the communists would become owners of concern. The subject thus relates to a highly critical stage
the haciendas as well as of the railroad company and in the history of this countryand as such, must be
other companies. In the same speech the hope was regarded as having passed into the public domain and as
expressed that the Constabulary force and the scouts an appropriate subject for speech and expression and
would desert in order to side with the reds. Words of this coverage by any form of mass media. The subject mater,
kind are properly considered seditious because they tend as set out in the synopsis provided by the petitioners and
to incite the people to take up arms against the quoted above, does not relate to the individual life and
constituted authorities and to rise against the established certainly not to the private life of private respondent
government. Ponce Enrile. "The Four Day Revolution" is not
principally about, nor is it focused upon, the man Juan
Ponce Enrile' but it is compelled, if it is to be historical, to
SECRETARY OF JUSTICE VS. SANDIGANBAYAN refer to the role played by Juan Ponce Enrile in the
precipitating and the constituent events of the change of
FACTS: During the criminal trial involving the former government in February 1986.
President Joseph Estrada for his plunder cases at the
SandiganBayan. The media requested if they can cover Similarly the SC likewise emphasized that JPE cannot
for purposes of ensuring transparency and the effectively invoke his right to privacy because he was a
constitutional guarantee of public trial. public figure and more importantly the film does not talk
about his private life.
ISSUE: Should the request be granted?

HELD: No. The courts recognize the constitutionally SWS VS. COMELEC GR No. 147571
embodied freedom of the press and the right to public
information. It also approves of media's exalted power to FACTS: RA No.9006 (Fair Election Act) Section 5.4

From the Discussions of Atty. Enrique Jr. Bonocan


University of Mindanao College of Law
REVIEWER ON CONSTITUTIONAL LAW II | CYNDI SATORRE-BICERA | 2nd Sem | S.Y. 2021-2022 30

in mendiola and there was a dispersal by the government


states:
through the PNP and used water cannons teofisto
guingonia was hit. The government said they are
Surveys affecting national candidates shall not be
implementing the PCRP and the provisions under BP 880.
published fifteen (15) days before an election and
The Supreme Court did not agree with the argument
surveys affecting local candidates shall not be published
tendered by the Government. The BP 880 is not an
seven (7) days before an election.
absolute ban for public assemblies but a restriction that
simply regulates the time, place, and manner of the
It created a chilling effect. Thus. SWS questioned the
assembly (content-neutral vs content-based regulations). In
provision as violative of the free speech clause.
this case, the petition was partly granted and the PCRP
was nullified and the SC ordered for the immediate
ISSUE: Was SWS correct in assailing the constitutionality
designation of Freedom parks according to BP 880 (no
of Section 5.4 of RA 9006.
need to apply for a permit).
HELD: The Court held that that §5.4 of R.A. No. 9006
constitutes an unconstitutional abridgment of freedom of Nicolas Lewis vs Comelec — what’s involved here was
speech, expression, and the press. Section 36.08 of RA 9189 (Overseas Absentee Voting Act).
Lewis assailed the prohibition of campaigning during
To be sure, §5.4 Iays a prior restraint on freedom of absentee voting period at any place abroad; it violates her
speech, expression, and the press prohibiting the constitutional right of free speech. SC nullified Sec. 36.08 of
publication of election survey results affecting candidates RA 9189 as an impermissible content-neutral regulation for
within the prescribed periods of fifteen (15) days being overbroad violating the free speech clause under
immediately preceding a national election seven (7) days Section 4 of the 1987 Constitution. In this case, the
before a local election. Because of tile preferred status of challenged provision's sweeping and absolute prohibition
tile constitutional rights of speech, expression, and he against all forms of expression considered as partisan
press, such a measure is vitiated by a weighty political activities without any qualification is more than what
presumption of invalidity. Indeed, any system of prior is essential to the furtherance of the contemplated
restraints of expression comes to this Court bearing a governmental interest. On its face, the challenged law
heavy Presumption against its constitutional validity. provides for an absolute and substantial suppression of
...The Government thus carries a heavy burden of speech as it leaves no ample alternative means for one to
showing justification for in enforcement of such restraint. freely exercise his or her fundamental right to participate in
"' There, thus a reversal of the normal presumption of partisan political activities. By banning partisan political
validity that inheres in every legislation. activities or campaigning even during the campaign period
within embassies, consulates, and other foreign service
establishments, regardless of whether it applies only to
CASES relative to the right to peacefully assemble candidates or whether the prohibition extends to private
governed by BP 880 also known as the “Public persons, it goes beyond the objective of maintaining order
Assembly Act of 1985”. during the voting period and ensuring a credible election. To
be sure, there can be no legally acceptable justification,
Primicias vs. Fugoso from the cases of Fuguso these are whether measured against the strictest scrutiny or the most
cases that are relative to the right to peaceably assemble lenient review, to absolutely or unqualifiedly disallow one to
(BP 880, Peaceable Assembly Act). In fact, this is where campaign within our jurisdiction during the campaign
they based that everytime there is a rally, they cannot just period. For the foregoing reasons, SC declared Section
do so if there is no permit coming from Under this law and 36.8 of RA 9189 as amended by RA 10590 unconstitutional
based on case laws, the Mayor or Chief Executive of the for violating Sec. 4, Art. III of the 1987 Constitution.
LGU concerned does not have discretionary powers
whether to issue the permit to rally or not; it's only —- END OF PRELIMS COVERAGE —--
ministerial. Except during the conduct of the rally, there is
violence. In Fuguso, the denial of the permit is based on an
ordinance that prohibits the conduct of rally on the ground
that it might cause chaos — the Supreme Court invalidated Section 5, Article III: Freedom of Religion Clause
that ordinance because the law merely confers upon the
municipal council the right to regulate not to altogether
prohibit and more importantly, it goes into the constitutional Section 5. No law shall be made respecting an establishment
right of the people and the group applying for the rally of religion, or prohibiting the free exercise thereof. The free
permit to peacefully assemble and petition the government exercise and enjoyment of religious profession and worship,
for redress of grievances. without discrimination or preference, shall forever be allowed.
No religious test shall be required for the exercise of civil or
political rights.
Randolph vs. Arroyo the act involved here is Presidential
Proclamation No. 1017 (PP 1017) and General Order No. 5
Historical Background: Spanish Constitution, the church and
(G.O. No. 5). PGMA revoked all the rally permits. That is
the state are one entity (church is part of the State). In fact,
why it was assailed because the rest did not know it was Spanish laws at that time had provided a lot of measures to
revoked and the rest still continued the rally despite the protect the church. However, when American sovereignty took
revocation. Thus, they were arrested and criminally over, there was already the separation of the church and the
charged. The Supreme Court invalidated the action of the state. The Philippine Constitution has always been patterned
government as held that PP 1017 was unconstitutional. before the American Constitution. – (Fr. Bernas)

Bayan vs Ermita — what’s involved here was PGMA’s This provision contains two aspects:
Preemptive Calibrated Response policy. There was a rally
(1) Non-establishment clause (xxx no law shall be made xxx)

From the Discussions of Atty. Enrique Jr. Bonocan


University of Mindanao College of Law
REVIEWER ON CONSTITUTIONAL LAW II | CYNDI SATORRE-BICERA | 2nd Sem | S.Y. 2021-2022 31

– the law is very clear that Congress cannot pass a statute the other.
that seeks to establish any religion. It is not allowed for any
religion to have a legal charter. Nor can Congress pass a law GR: Congress cannot include in the GAA the appropriation to
through an RA officially recognizing a certain religion as the fund religious projects or similar purposes. Because it violates
official religion of the state or create a new religion. the separation of church and state no matter how noble the
intention is.
(2) Free exercise clause (or prohibiting the exercise thereof) – XPNs: (See cases below)
Congress cannot pass a law that prohibits the exercise of
religion.
AGLIPAY V. RUIZ, G.R. No. L-45459, 13 March 1937
In interpreting this free exercise clause embodied in the
Constitution, the Court has consistently adhered to the doctrine FACTS: 33rd International Eucharistic Congress was held in
that: The right to religious profession and worship has a the Philippines. The Director of Post printed postage stamps
two-fold aspect (sub-aspect) of Free exercise clause: that highlighted that event under the provisions of Act No.
4052 of the Philippine Legislature which appropriated funds.
(1) freedom to believe – absolute as long as the belief is The sale of the postage stamps was questioned.
confined within the realm of thought.
Ex: “I believe that Satan is my God”. Will you be criminally Q: Was there a violation of the non-establishment clause?
charged? No. (The Government cannot even prevent you)
HELD: No. It should be stated that what is guaranteed by
(2) freedom to act on one’s beliefs – subject to regulation our Constitution is religious liberty, not mere religious
where the belief is translated into external acts that affect the toleration. Religious freedom, however, as a constitutional
public welfare. mandate is not inhibition of profound reverence for religion
Ex: If you perform overt acts that would disrupt the public and is not denial of its influence in human affairs. In the
peace and welfare, such as killing a virgin for sacrifice/rituals. preamble of the Constitution, it manifested reliance upon
Him who guides the destinies of men and nations. The
elevating influence of religion in human society is
Q: Can you still invoke your constitutional right of recognized here as elsewhere.
freedom to act on one’s belief? No, it’s already a murder or
homicide as the case may be (unless the victim consents). While the issuance and sale of the stamps in question may
be said to be inseparably linked with an event of a religious
As a matter of fact, J. Isagani Cruz in his opinion in the case of character, the resulting propaganda, if any, received by the
REQUEST OF MUSLIM EMPLOYEES IN THE DIFFERENT Roman Catholic Church, was not the aim and main purpose
COURTS IN ILIGAN CITY said: of the Government. The Government should not be
embarrassed in its activities simply because of incidental
The individual is free to believe (or disbelieve) as he pleases results, more or less religious in character, if the purpose
concerning the hereafter. He may indulge his own theories had in view is one which could legitimately be undertaken by
about life and death; worship any god he chooses, or none at appropriate legislation. The main purpose should not be
all; embrace or reject any religion; acknowledge the divinity of frustrated by its subordination to mere incidental results not
God or of any being that appeals to his reverence; recognize or contemplated.
deny the immortality of his soul – in fact, cherish any religious
conviction as he and he alone sees fit. However absurd his
beliefs may be to others, even if they be hostile and heretical to AUSTRIA V. NLRC, G.R. No. 124382, 16 August 1999
the majority, he has full freedom to believe as he pleases. He
may not be required to prove his beliefs. He may not be Pastor Austria worked with the 7th Day Adventist with 12
punished for his inability to do so. Religion, after all, is a matter churches under him. In 1991, he was transferred to Bacolod.
of faith. "Men may believe what they cannot prove." Everyone Failure to account for the church tites. Because of this, he
has a right to his beliefs and he may not be called to account was terminated. That is why he filed an illegal termination
because he cannot prove what he believes . . . . BUT . . . case with the Labor Arbiter. The Labor Arbiter ruled against
him which he appealed before the NLRC which upheld his
Where the individual externalizes his beliefs in acts or termination. He now questions the jurisdiction of the NLRC
omissions that affect the public, his freedom to do so becomes over the case because the matter involved is ecclesiastical.
subject to the authority of the State. As great as this liberty
may be, religious freedom, like all other rights guaranteed in ISSUE: Has the NLRC has jurisdiction over the case.
the Constitution, can be enjoyed only with a proper regard for
the rights of others. It is an error to think that the mere HELD: Yes. The case does not concern an ecclesiastical or
invocation of religious freedom will stalemate the State and purely religious affair as to bar the State from taking
render it impotent in protecting the general welfare. The cognizance of the same. An ecclesiastical affair involves the
inherent police power can be exercised to prevent religious relationship between the church and its members and
practices inimical to society. And this is true even if such relates to matters of faith, religious doctrines, worship and
practices are pursued out of sincere religious conviction and governance of the congregation i.e. proceedings for
not merely for the purpose of evading the reasonable excommunication, ordinations of religious ministers,
requirements or prohibitions of the law. administration of sacraments and other activities with
attached religious significance which the State cannot
Justice Frankfurter put it succinctly: The constitutional meddle. The case does not even remotely concern any of
provision on religious freedom terminated disabilities, it did not the above cited examples. He was not ex-communicated or
create new privileges. It gave religious liberty, not civil expelled from the membership of the SDA but was
immunity. Its essence is freedom from conformity to religious terminated from employment. What is involved here is the
dogma, not freedom from conformity to law because of relationship of the church as an employer and the minister
religious dogma. as an employee; it is purely secular.

NON-ESTABLISHMENT CLAUSE: It prohibits the state from UCCP V. BRADFORD UNITED CHURCH OF CHRIST, INC
passing laws which aid one religion or prefer one religion over

From the Discussions of Atty. Enrique Jr. Bonocan


University of Mindanao College of Law
REVIEWER ON CONSTITUTIONAL LAW II | CYNDI SATORRE-BICERA | 2nd Sem | S.Y. 2021-2022 32

G.R. No. 171905, 20 June 2012 G.R. No. 204819, etc., 08 April 2014 (BAR Q - RH BILL)

FACTS: Petitioner United Church of Christ in the Q: Does the duty to “refer” make them criminally liable?
Philippines, Inc. (UCCP) is a religious corporation duly
organized and existing under the laws of the Philippines. It is A: The Court is of the view that the obligation to refer
a national confederation of incorporated and unincorporated imposed by the RH Law violates the religious belief and
self-governing Evangelical churches of different conviction of a conscientious objector. Once the medical
denominations, devised for fellowship, mutual counsel and practitioner, against his will, refers a patient seeking
cooperation. It is the ecclesiastical successor of the information on modem reproductive health products,
Evangelical Church of the Philippines, the Philippine services, procedures and methods, his conscience is
Methodist Church and the United Evangelical Church of the immediately burdened as he has been compelled to perform
Philippines. an act against his beliefs.

Respondent BUCCI, used to be part of UCCP but in 1979, it Though it has been said that the act of referral is an opt-out
formed its own incorporation under SEC. UCCP opposed clause, it is, however, a false compromise because it makes
this and requested the SEC for its annulment alleging that it pro-life health providers complicit in the performance of an
is not allowed under UCCP’s by-laws. act that they find morally repugnant or offensive. They
cannot, in conscience, do indirectly what they cannot do
ISSUE: WON the disaffiliation of BUCCI is valid and is directly. One may not be the principal, but he is equally guilty
purely an ecclesiastical affair outside the jurisdiction of SEC. if he abets the offensive act by indirect participation.

HELD: No. UCCP and BUCCI, being corporate entities and The Bill of Rights guarantees the liberty of the individual to
grantees of primary franchises, are subject to the jurisdiction utter what is in his mind and the liberty not to utter what is
of the SEC. Section 3 of PD No. 902-A provides that SEC not in his mind. While the RH Law seeks to provide freedom
shall have absolute jurisdiction, supervision and control over of choice through informed consent, freedom of choice
all corporations. Even with their religious nature, SEC may guarantees the liberty of the religious conscience and
exercise jurisdiction over them in matters that are legal and prohibits any degree of compulsion or burden, whether
corporate. BUCCI, as a juridical entity separate and distinct direct or indirect, in the practice of one's religion.
from UCCP, possesses the freedom to determine its steps.

The Court owes but recognition to BUCCI’s decision as it **CASES ON FREE EXERCISE CLAUSE**
concerns its legal right as a religious corporation to AMERICAN BIBLE SOCIETY V CITY OF MANILA,
disaffiliate from another religious corporation via legitimate G.R. No. L-9637, 30 April 1957
means –a secular matter well within the civil courts’ purview.
FACTS: ABS has been distributing and selling bibles and/or
gospel throughout the Philippines and translating the same
TARUC . DE LA CRUZ, G.R. No. 144801, 10 March 2005 into several Philippine dialects. In 1953, the acting City
Treasurer of the City of Manila informed ABS that it was
FACTS: Taruc and other petitioners were lay members of conducting the business of general merchandise since
the Philippine Independent Church (PIC). Taruc tried to November, 1945, without providing itself with the necessary
organize an open mass to be celebrated by Fr. Ambong Mayor's permit and municipal license, in violation of
during the town fiesta of Socorro. When Taruc informed Ordinance No. 3000, as amended, and Ordinances Nos.
Bishop Dela Cruz of the plan, the Bishop tried to dissuade 2529, 3028 and 3364, and required ABS to secure, within 3
him because Fr. Ambong was not a member of the clergy of days, the corresponding permit and license fees, together
the diocese of Surigao and his credentials were in doubt. with compromise covering the period from the Q4 of 1945 to
When the Bishop failed to stop Taruc, he declared Q2 of 1953, in the total sum of P5,821.45. ABS questioned
petitioners in 1993 as expelled/excommunicated from the the directive as violative of the free exercise clause.
PIC. Because of this, Taruc et al filed a complaint for
damages with a preliminary injunction against Bishop De la HELD: Yes, such an It may be true that in the case at bar
Cruz before the RTC. They contend that their expulsion was the price asked for the bibles and other religious pamphlets
illegal because there was no due process of law. Bishop was in some instances a little bit higher than the actual cost
questioned the jurisdiction of the court invoking the of the same but this cannot mean that appellant was
separation of church and the state and that the matters engaged in the business or occupation of selling said
involved here is ecclessiastical. "merchandise" for profit. For this reason We believe that the
provisions of City of Manila Ordinance No. 2529, as
ISSUE: Has the court jurisdiction over the case? amended, cannot be applied to appellant, for in doing so it
would impair its free exercise and enjoyment of its religious
HELD: No. In our jurisdiction, we hold the Church and the profession and worship as well as its rights of dissemination
State to be separate and distinct from each other. "Give to of religious beliefs.
Caesar what is Caesar's and to God what is God’s."

SC agreed with the CA that the expulsion/excommunication COMPARE WITH THE CASE OF TOLENTINO
of members of a religious institution/organization is a matter
best left to the discretion of the church officials, and the laws TOLENTINO V. SEC. OF FINANCE
and canons of said institution/organization. It is not for the G.R. No. 115455, 25 August 1994
courts to exercise control over church authorities in the
performance of their discretionary and official functions. FACTS: This entity invoking the case of ABS, questioned
Rather, it is for the members of religious institutions or BIR in making the products/reading materials subject to VAT
organizations to conform to church regulations. law. They were asked to add-on 10% VAT and remit it to the
BIR. Philippine Bible Society assailed this. They were
likewise exempt for being subjected to the VAT law.
SPOUSES IMBONG V. OCHOA, ET AL

From the Discussions of Atty. Enrique Jr. Bonocan


University of Mindanao College of Law
REVIEWER ON CONSTITUTIONAL LAW II | CYNDI SATORRE-BICERA | 2nd Sem | S.Y. 2021-2022 33

HELD: What was involved in the ABS case was local during the flag ceremony while their classmates and
business taxes. In this case, PBS was subjected to the VAT teachers salute the flag, sing the national anthem and recite
law imposed upon the buyer of the goods and not the seller. the patriotic pledge, such conduct will NOT possibly disturb
the peace, or pose "a grave and present danger of a serious
evil to public safety, public morals, public health or any other
GERONA V BOARD OF EDUCATION legitimate public interest that the State has a right and duty
G.R. No. L-13954, 12 August 1959 to prevent.

FACTS: Petitioners were members of Jehovah’s Witnesses. ANOTHER CASE WHERE SC MADE THE RELIGIOUS
Their children were expelled from the school for their refusal FREEDOM PREVAIL OVER A STATUTORY OBLIGATION
to join the flag ceremony because it is against their religious
belief. Thus, they wrote to the Secretary of Education
invoking the free exercise clause but denied the petition. ALEJANDRO ESTRADA v. SOLEDAD S. ESCRITOR
Thus, they elevated the case to the SC.
FACTS: An admin case against Escritor, a court employee
ISSUE: Was the denial of DEPED as regards to their for disgraceful and immoral conduct was filed. Escritor,
request for the exemption and for the reinstatement of their already a widow when she entered the judiciary in 1999 and
expelled children, valid? admitted that she has been living with Quilapio (who still has
HELD: Yes. The flag is not an image but a symbol of the legal impediment) without the benefit of marriage for 20
Republic of the Philippines, an emblem of national years and that they have a son. But their conjugal
sovereignty, of national unity and cohesion and of freedom arrangement is in conformity with their religious beliefs. In
and liberty which it and the Constitution guarantee and fact, after 10 years of living together, she executed in July
protect. Considering the complete separation of church and 1991 a "Declaration of Pledging Faithfulness," – it makes
state in our system of governments, the flag is utterly devoid the union moral and binding within the congregation all over
of any religious significance. Saluting the flag consequently the world except in countries where divorce is allowed. Only
does not involve any religious ceremony. couples who have been baptized and in good standing may
execute that declaration which requires the approval of the
In requiring school pupils to participate in the flag salute, the elders of the Congregation. As a matter of practice, the
State thru the Secretary of Education was not imposing a marital status of the declarant and their respective spouses'
religion or religious belief or test on said students. It was commission of adultery are even investigated before the
merely enforcing a non-discriminatory school regulation declarations are allowed to be executed. In sum, therefore,
applicable to all alike whether Christian, Muslim, Protestant insofar as the congregation is concerned, there is nothing
or Jehovah's Witness. The State was merely carrying out immoral about the conjugal arrangement between Escritor
the duty imposed upon it by the Constitution which charged and Quilapio and they remain members in good standing in
it with supervision over and regulation of all educational the congregation. Escritor invoked the religious belief
institutions, to establish and maintain a complete and practice and she should not be held administratively liable.
adequate system of public education, and to see to it that all
schools aim to develop among other things, civic conscience ISSUE: Is she administratively liable for disgraceful and
and teach the duties of citizenship [Sec. 5, Art. XIV]. It does immoral conduct?
nothing more than try to inculcate in the minds of the school
population during the formative period of their life. Whatever HELD: No. The present case involves purely conduct arising
is taught to the youth during this period, such as love of from religious belief. Not any interest of the state would
country and love of the flag, all of which make for united and suffice to prevail over the right to religious freedom as this is
patriotic citizenry. a fundamental right that enjoys a preferred position in the
hierarchy of rights – the most inalienable and sacred of all
human rights. This right is sacred for an invocation of the
EBRALINAG V. DIVISION SUPERINTENDENT OF Free Exercise Clause. In this particular case and under the
SCHOOLS OF CEBU, G.R. No. 95770, 03/01/1993 distinct circumstances, respondent’s conjugal arrangement
(stare de cisis - abandoned the Gerona ruling) cannot be penalized as she has made out a case for
exemption from the law based on the fundamental right of
FACTS: Same as the Gerona case. RA 1265 (Flag Law) freedom of religion. Escritor must enjoy her religious
and Dept. Order 8 making the flag ceremony compulsory. freedom. In the area of religious exercise as a preferred
freedom, however, man stands accountable to an authority
HELD: SC abandoned the Gerona ruling. Religious freedom higher than the State and so the state interest sought to be
is a fundamental right which is entitled to the highest priority upheld must be so compelling that its violation will erode the
and the amplest protection among human rights, for it very fabric of the State that will also protect that freedom. In
involves the relationship of man to his Creator. What the the absence of showing that such state interest exists,
petitioners seek only is exemption from the flag ceremony, man must be allowed to subscribe to the infinite.
not exclusion from the public schools where they may study
and learn the Constitution as part of the curricula and Q: What is meant by the Doctrine of Benevolent Neutrality
acquire knowledge and skills from these educational or Accommodation?
institutions pursuant to the mandate in Sec. 3(2), Art. XIV of
the Constitution. The exemption may be accorded to them In the case of Estrada vs. Escritor, the Benevolent Neutrality
however "bizarre" those beliefs may seem to others. Theory believes that with respect to these governmental
Nevertheless, their right not to participate in the flag actions, accommodation of religion may be allowed not to
ceremony does not give them a right to disrupt such patriotic promote the government's favored form of religion, but to allow
exercises. While the highest regard must be afforded their individuals and groups to exercise their religion without
right to the free exercise of their religion, "this should not be hindrance. Their purpose or effect therefore is to remove a
taken to mean that school authorities are powerless to burden on, or facilitate the exercise of, a person's or
discipline them" if they should commit breaches of the peace institution's religion. What is sought under the theory of
by actions that offend the sensibilities, both religious and accommodation is not a declaration of unconstitutionality of a
patriotic, of other persons. If they quietly stand at attention facially neutral law, but an exemption from its application or its

From the Discussions of Atty. Enrique Jr. Bonocan


University of Mindanao College of Law
REVIEWER ON CONSTITUTIONAL LAW II | CYNDI SATORRE-BICERA | 2nd Sem | S.Y. 2021-2022 34

burdensome effect whether by the legislature or the courts.


application was accepted. This Court's decision in Victoriano
vs. Elizalde Rope Workers' Union, upholding the right of
BENJAMIN VICTORIANO VS. ELIZALDE ROPE members of the INC sect not to join a labor union for being
FACTORY (ERF) , G.R. NO. L-25246, 12 September 1974 contrary to their religious beliefs, does not bar the members
of that sect from forming their own union. The public
FACTS: Victoriano, a member of INC and employed in ERF, respondent correctly observed that the "recognition of the
was compelled (against his will because INC does not allow tenets of the sect should not infringe on the basic right of
them) to join the Union because of an existing collective self-organization granted by the constitution to workers,
bargaining agreement that contains a closed shop provision regardless of religious affiliation."
– where it says that if you are not a member, you may be
validly terminated from employment. Congress then enacted
RA 3350 (legally exempts members of religious sects that ISLAMIC DA’WAH COUNCIL OF THE PHILS (IDCP) V.
prohibits members to join Unions). Victoriano presented his EXECUTIVE SECRETARY, G.R. No. 153888, 09 July 2003
resignation from his Union which later wrote to the
management for his termination on account of their Union IDCP has always been the official corporation to issue Jalal
security clause. Certificate on food products. In 2001, the Office of the
Executive Secretary issued EO 465 creating the Philippine
ISSUE: May the management validly terminate Victoriano Halal Certification Scheme and designating co-respondent
based on the violation of the CBA? OMA to oversee its implementation. Under the EO, OMA
has the exclusive authority to issue halal certificates and
HELD: NO. The free exercise of religious profession or perform other related regulatory activities. IDCP questioned
belief is superior to contractual rights. In case of conflict, the this.
latter must, therefore, yield to the former. Religious freedom,
although not unlimited, is a fundamental personal right and ISSUE: Does EO 46 violate the separation of state and the
liberty, and has a preferred position in the hierarchy of church clause of the Constitution?
values. Contractual rights, therefore, must yield to freedom
of religion. It is only where unavoidably necessary to prevent HELD: Yes.OMA deals with the societal, legal, political and
an immediate and grave danger to the security and welfare economic concerns of the Muslim community as a "national
of the community that infringement of religious freedom may cultural community" and not as a religious group. Thus,
be justified, and only to the smallest extent necessary to bearing in mind the constitutional barrier between the
avoid the danger. To compel persons to join and remain Church and State, the latter must make sure that OMA does
members of a union to keep their jobs in violation of their not intrude into purely religious matters lest it violate the
religious scruples, would hurt, rather than help, labor unions. non-establishment clause and the "free exercise of religion"
Congress has seen it fit to exempt religious objectors lest provision found in Sec. 5, Art. III of the 1987 Constitution.
their resistance spread to other workers, for religious
objections have contagious potentialities more than political Without doubt, classifying a food product as halal is a
and philosophic objections. religious function because the standards used are drawn
Q: In the ruling of Elizalde does it mean that the members from the Qur'an and Islamic beliefs. By giving OMA the
of INC are already precluded from joining unions? NO exclusive power to classify food products as halal, EO 46
encroached on the religious freedom of Muslim
organizations like IDCP to interpret for Filipino Muslims what
KAPATIRAN SA MEAT AND CANNING DIVISION VS. food products are fit for Muslim consumption. Also, by
CALLEJA, G.R. NO. 82914, 20 June 1988 arrogating to itself the task of issuing halal certifications, the
State has in effect forced Muslims to accept its own
FACTS: From 1984 to 1987 TUPAS was the sole and interpretation of the Qur'an and Sunnah on halal food. EO
exclusive CBA Rep of the workers in the Meat and Canning 46 was invalidated.
Division of the Universal Robina Corporation (URC), with a
3-year collective bargaining agreement (CBA). Within the
freedom period of 60 days prior to the expiration of its CBA, RE: REQUEST OF MUSLIM EMPLOYEES IN THE
TUPAS filed an amended notice of strike on September 28, DIFFERENT COURTS IN ILIGAN CITY,
1987 as a means of pressuring the company to extend, A.M. No. 02-2-10-SC, 14 December 2005
renew, or negotiate a new CBA with it. URC obtained an
injunction against the strike, resulting in an agreement to
FACTS: Muslim Court employees requested to be exempted
return to work and for the parties to negotiate a new CBA.
from observing office hours on two occassions: (1) to hold
office hours from 7:30 a.m. to 3:30 p.m. without lunch or
On October 8, 1987, the NEW ULO, composed mostly of
coffee breaks during the month of Ramadan; (2) to be
workers belonging to the INC sect, registered as a labor
excused from work from 10:00 a.m. to 2:00 p.m. every
union, claiming that it has "the majority of the daily wage
Friday (Muslim Prayer Day) during the entire calendar year.
rank and file employees numbering 191," and filed a petition
for a certification election — legal process to select a
ISSUE: Should the requests be granted based on the
legitimate CBA Rep will be chosen from among 2 or more
exercise clause?
existing CBU — at the Bureau of Labor Relations. TUPAS
moved for the dismissal for the petition of the certification
HELD: The Court recognizes that the observance of
election for being defective in form and averring that the
Ramadan and the Friday Muslim Prayer Day is integral to
NEW ULO is composed mostly of members of INC which
the Islamic faith. Thus it partly granted the request but only
refused to affiliate with other Labor Unions and accused
during the Ramadan finding support in Section 3 (a) of P.D.
URC in using NEW ULO to defeat the bargaining rights of
No. 291, as amended by P.D. No. 322. However, the Court
TUPAS.
did not grant the second request because this will now be
inconsistent with the CSC laws and for lack of statutory
ISSUE: IS the NEW ULO a legitimate labor union?
basis. To allow the Muslim employees in the Judiciary to be
excused from work from 10:00 a.m. to 2:00 p.m. every
HELD: Yes. Because they were registered and their
Friday (Muslim Prayer Day) during the entire calendar year

From the Discussions of Atty. Enrique Jr. Bonocan


University of Mindanao College of Law
REVIEWER ON CONSTITUTIONAL LAW II | CYNDI SATORRE-BICERA | 2nd Sem | S.Y. 2021-2022 35

would mean a diminution of the prescribed government in the district by the police after closing the district where
working hours. For then, they would be rendering service 12 they work) to Davao as laborers. Without their knowledge,
hours less than that required by the CSC rules for each consent, and was given no opportunity to collect their
month. Further, this would encourage other religious belongings. The Counsel for the relatives and the friends of
denominations to request for similar treatment. deportees filed an application for habeas corpus alleging
that those women were illegally refrained from their liberty.
The performance of religious practices, whether by the
Muslim employees or those belonging to other religious ISSUE: By authority of what law did the Mayor and the Chief
denominations, should not prejudice the courts and the of Police presume to act in deporting by duress these
public. Indeed, the exercise of religious freedom does not persons from Manila to another distant locality within the
exempt anyone from compliance with reasonable Philippine Islands?
requirements of the law, including civil service laws.
RULING: NOTHING. The act of the respondents was illegal.
Under the American constitutional system, liberty of abode
ANG LADLAD LGBT PARTY V. COMELEC, is a principle so deeply embedded in jurisprudence and
G.R. No. 190582, 08 April 2010 considered so elementary in nature as not even to require a
constitutional sanction. Even the Gov-Gen or the President
FACTS: ANG LADLAD was denied application for their of the US, who has often been said to exercise more power
accreditation as a Party List. COMELEC cited provisions in than any king or potentate, has no such arbitrary
the scripture: Romans 1:26, 27 and the Koran verses. prerogative, either inherent or express. Much less, therefore,
has the executive of a municipality, who acts within a sphere
ISSUE: Was the denial by the COMELEC valid? of delegated powers. If the mayor and the chief of police
could, at their mere behest, render the liberty of the citizen
HELD: No. Our Constitution provides in Article III, Section 5 so insecure, then any other official can do the same. And if a
that "no law shall be made respecting an establishment of prostitute could be sent against her wishes and under no
religion, or prohibiting the free exercise thereof." At bottom, law from one locality to another within the country, then it
what our non-establishment clause calls for is "government can hold the same club over the head of any citizen. The
neutrality in religious matters." Clearly, "governmental forcible taking of these women, deprived them of freedom of
reliance on religious justification is inconsistent with locomotion just as effectively as if they had been
this policy of neutrality." It was a grave violation of the imprisoned. Placed in Davao without either money or
non-establishment clause for the COMELEC to utilize the personal belongings, they were prevented from exercising
Bible and the Koran to justify the exclusion of ANG the liberty of going when and where they pleased.
LADLAD. Rather than relying on religious belief, the
legitimacy of the Assailed Resolutions should depend,
instead, on whether the COMELEC is able to advance some RICARDO SILVERIO VS CA GR 94284 (COURT ORDER)
justification for its rulings beyond mere conformity to
religious doctrine. Otherwise stated, the government must FACTS: Silverio was charged with violation of Sec. 20 (4) of
act for secular purposes and in ways that have primarily the Revised Securities Act. He posted bail but an Order was
secular effects. issued directing the DFA to cancel Petitioner's passport or to
deny his application therefor, and the Commission on
Immigration to prevent him from leaving the country on the
Section 6, Article III – Liberty of Abode & Travel grounds that he had gone abroad several times without the
necessary Court approval resulting in postponements of the
arraignment and hearings. He contends that the scheduled
Section 6. The liberty of abode and of changing the same arraignments can’t be had because there was a pending
within the limits prescribed by law shall not be impaired except MTQ and (2) the right to travel can be impaired upon lawful
upon lawful order of the court. Neither shall the right to travel order of the Court, even on grounds other than the "interest
be impaired except in the interest of national security, public of national security, public safety or public health."
safety, or public health, as may be provided by law.
RULING: Silverio is NOT CORRECT. Sec. 6, Art. III should
Q: What is the concept of the liberty of abode? by no means be construed as delimiting the inherent power
of the Courts to use all means necessary to carry their
A: Here, no one can be compelled to change his/her home orders into effect in criminal cases pending before them in
except in accordance with law. According to Fr. Bernas, Liberty order to attain finality and avoid undue delay. All auxiliary
can be impaired only upon lawful order of the court and upon writs, process and other means necessary to carry it into
the issuance of that lawful order, the Court is to be guided by effect may be employed by such Court or officer (Rule 135,
the limits prescribed by law. Sec. 6, ROC). Silverio is facing a criminal charge. He has
posted bail but has violated the conditions thereof by failing
Ex: The civilians found in a rebel-infested locality, the military to appear before the Court when required. Warrants for his
commanders usually ask them in one area or would require arrest have been issued. Those orders and processes would
them to spend a night in a safe area. This HAMLETING could be rendered nugatory if an accused were to be allowed to
have come under this provision but unfortunately, this issue did leave or to remain, at his pleasure, outside the territorial
not reach the SC. confines of the country. Holding an accused in a criminal
case within the reach of the Courts by preventing his
“LIMITS PRESCRIBED BY LAW” – examples are the final departure from the Philippines must be considered as a
and executory orders issued by the Court in ejectment cases valid restriction on his right to travel so that he may be dealt
or Order of Demolition. with in accordance with law.

VILLAVICENCIO V. LUKBAN, G.R. No. L-14639, 3/25/1919 EFRAIM GENUINO, ET AL V. DE LIMA, G.R. No. 197930

The Mayor of Lukban and the city authorities deported 170 FACTS: Criminal complaints were filed against former
ill-reputed women (who were kept confined to their houses

From the Discussions of Atty. Enrique Jr. Bonocan


University of Mindanao College of Law
REVIEWER ON CONSTITUTIONAL LAW II | CYNDI SATORRE-BICERA | 2nd Sem | S.Y. 2021-2022 36

President GMA before the DOJ. Thus, De Lima issued DOJ well-considered view that the right to return may be
WLO (Watch List Order) pursuant to her authority under considered, as a generally accepted principle of
DOJ Circular No. 41 against GMA. She also ordered for the international law and, under our Constitution, is part of the
inclusion of GMA's name in the Bureau of Immigration law of the land. However, it is distinct and separate from the
watchlist. GMA filed the present Petition to annul and set right to travel and enjoys a different protection under the
aside DOJ Circular No. 41 and WLOs issued against her for ICCPR i.e., against being "arbitrarily deprived" thereof.
allegedly being unconstitutional.

ISSUE: May the DOJ validly Issue HDO, WLO or allow DO? Q: What are the known statutory limits to the exercise of
the right to travel?
RULING: NO. The right to travel is part of the "liberty" of
which a citizen cannot be deprived without due process of Explained in the case of Leave Division, OAS-OCA v.
law. It is part and parcel of the guarantee of freedom of Heusdens, the exercise of one’s right to travel or the freedom
movement that the Constitution affords its citizens. Under to move from one place to another, as assured by the
Sec. 6, Article III of the Constitution, there are only three Constitution, is not absolute. There are constitutional, statutory
considerations that may permit a restriction on the right to and inherent limitations regulating the right to travel. Section 6
travel: national security, public safety or public health. As a itself provides that "neither shall the right to travel be impaired
further requirement, there must be an explicit provision of except in the interest of national security, public safety or public
statutory law or the ROC providing for the impairment. health, as may be provided by law." Some of these statutory
limitations are the following:
It is clear from the foregoing that the liberty of abode may
only be impaired by a lawful order of the court. There is no 1] The Human Security Act of 2010 or Republic Act (R.A.)
law particularly providing for the authority of the SOJ to No. 9372. The law restricts the right to travel of an individual
curtail the exercise of the right to travel, in the interest of charged with the crime of terrorism even though such person is
national security, public safety or public health. DOJ Circular out on bail.
No. 41 is not a law. It is not a legislative enactment which
underwent the scrutiny and concurrence of lawmakers, and 2] The Philippine Passport Act of 1996 or R.A. No. 8239.
submitted to the President for approval. Without a law to Pursuant to said law, the Secretary of Foreign Affairs or his
justify its action, the issuance of DOJ Circular No. 41 is an authorized consular officer may refuse the issuance of, restrict
unauthorized act of the DOJ of empowering itself under the the use of, or withdraw, a passport of a Filipino citizen.
pretext of dire exigency or urgent necessity. This action runs
afoul the separation of powers between the three branches 3] The "Anti- Trafficking in Persons Act of 2003" or R.A.
of the government and cannot be upheld. No. 9208. Pursuant to the provisions thereof, the Bureau of
Immigration, in order to manage migration and curb trafficking
in persons, issued Memorandum Order Radjr No. 2011-011,12
MARCOS V. MANGLAPUS, G.R. 88211, 9/15/1989 allowing its Travel Control and Enforcement Unit to "offload
(President’s Residual Power in re: right to travel) passengers with fraudulent travel documents, doubtful purpose
of travel, including possible victims of human trafficking" from
FACTS: The government of Corazon Aquino imposed a ban our ports.
on Former Pres. Marcos regarding his return to the
Philippines. This was assailed by the Marcoses as violative 4] The Migrant Workers and Overseas Filipinos Act of 1995
of their right to travel and abode averring that there is or R. A. No. 8042, as amended by R.A. No. 10022. In
nothing in the Constitution written to ban a person to return enforcement of said law, the Philippine Overseas Employment
to his home. In fact, Sec. 6 says that freedom to travel is a Administration (POEA) may refuse to issue deployment permit
constitutional right that can be limited by a law. to a specific country that effectively prevents our migrant
workers to enter such country.
ISSUE: May the President in the exercise of the powers
granted by Constitution prohibit the Marcos’ from returning 5] The Act on Violence against Women and Children or
to the Philippines. R.A. No. 9262. The law restricts movement of an individual
against whom the protection order is intended.
RULING: YES. It is the President’s residual power. The right
involved here is the right to return to one's country, a totally 6] Inter-Country Adoption Act of 1995 or R.A. No. 8043.
distinct right under international law, independent from, Pursuant thereto, the Inter-Country Adoption Board may issue
although related to, the right to travel. Thus, the UDHR and rules restrictive of an adoptee’s right to travel "to protect the
the ICCPR treat both rights as separate and distinct Filipino child from abuse, exploitation, trafficking and/or sale or
from each other. The Declaration speaks of the "right to any other practice in connection with adoption which is
freedom of movement and residence within the borders of harmful, detrimental, or prejudicial to the child."
each state" separately from the "right to leave any country,
including his own, and to return to his country." On the other Inherent limitations on the right to travel are those that naturally
hand, the Covenant guarantees the "right to liberty of emanate from the source. These are very basic and are built-in
movement and freedom to choose his residence" and the with the power. An example of such inherent limitation is the
right to "be free to leave any country, including his own." power of the trial courts to prohibit persons charged with a
which rights may be restricted by such laws as "are crime to leave the country. In such a case, permission of the
necessary to protect national security, public order, public court is necessary. Another is the inherent power of the
health or morals or enter one's own country" of which one legislative department to conduct a congressional inquiry in aid
cannot be "arbitrarily deprived." It would therefore be of legislation. In the exercise of legislative inquiry, Congress
inappropriate to construe the limitations to the right to has the power to issue a subpoena and subpoena duces
return to one's country in the same context as those tecum to a witness in any part of the country, signed by the
pertaining to the liberty of abode and the right to travel. chairperson or acting chairperson and the Speaker or acting
The right to return to one's country is not among the rights Speaker of the House; or in the case of the Senate, signed by
specifically guaranteed in the Bill of Rights, which treats only its Chairman or in his absence by the Acting Chairman, and
of the liberty of abode and the right to travel, but it is our approved by the Senate President.

From the Discussions of Atty. Enrique Jr. Bonocan


University of Mindanao College of Law
REVIEWER ON CONSTITUTIONAL LAW II | CYNDI SATORRE-BICERA | 2nd Sem | S.Y. 2021-2022 37

relevant to a government contract.In Chavez v. Public Estates


Section 7, Article III – Right to Information Authority thus laid down the rule that the constitutional right to
information includes official information on on-going
negotiations before a final contract. The information, however,
SECTION 7. The right of the people to information on matters
must constitute definite propositions by the government and
of public concern shall be recognized. Access to official
should not cover recognized exceptions like privileged
records, and to documents, and papers pertaining to official
information, military and diplomatic secrets and similar matters
acts, transactions, or decisions, as well as to government
affecting national security and public order. In addition,
research data used as basis for policy development, shall be
Congress has prescribed other limitations on the right to
afforded the citizen, subject to such limitations as may be
information in several legislations.
provided by law.

TO SUM IT UP, according to the case of Chavez vs PCGG,


NOTE: that this Right to information is intertwined with the
these are the limitations to the right to information under
government’s constitutional duty to full public disclosure of all
Section 7, Article III: [BINT-COC-6713]
transactions involving public interest. Section 28, Article II — it
is a policy of the State to adhere to full transparency in all
1] Banking Transactions – under the Bank Secrecy law; XPN:
transactions involving public interest. Subject to reasonable
RA 670 (Anti-Graft and Corrupt Practices Law)
conditions prescribed by law, the State adopts a policy of full
public disclosure of all its transactions involving public interest.
2] Intelligence Information;
Section 7, Art. III is the individual right and Section 28, Art. 2 is
the correlative duty of the government to observe full public
3] National Security matter;
disclosure of all its transactions involving public interest.
4] Trade Secrets – patented inventions in the possession of the
Now, what do you think is the purpose of this provision? IPO can’t be disclosed;

In the case of Chavez vs. Public Estates Authority and 5] Criminal Matters – on going investigations as regards the
Amari Coastal Bay Development Corporation, these twin commission of a crime
provisions of the Constitution seek to promote transparency in 6] Other Confidential Information – classified info; Executive
policy-making and in the operations of the government, as well Privilege; Deliberative Process Privilege [Akbayan vs. Tomas
as provide the people sufficient information to exercise Aquino]
effectively other constitutional rights.
7] Sec 3 Rule 4 of RA 6713 or the Code of Ethical Standards
for Public Officers and Employees.
These twin provisions are essential to the exercise of freedom
of expression. If the government does not disclose its official Pursuant to the mandate of this law, the CSC promulgated the
acts, transactions and decisions to citizens, whatever citizens implementing rules. Sec. 3 Rule 4 provides that every
say, even if expressed without any restraint, will be speculative department, office or agency shall provide official information,
and amount to nothing. These twin provisions are also records or documents to any requesting public, EXCEPT if:
essential to hold public officials "at all times x x x accountable
to the people," for unless citizens have the proper information, (a) such information, record or document must be kept secret
they cannot hold public officials accountable for anything. in the interest of national defense or security or the conduct of
Armed with the right information, citizens can participate in foreign affairs; (restatement of the ruling in the cases of
public discussions leading to the formulation of government Chavez v PEA and Chavez v PCGG)
policies and their effective implementation. An informed
citizenry is essential to the existence and proper functioning of (b) such disclosure would put the life and safety of an
any democracy. Now, this right to information however is not individual in imminent danger;
without limitation. According to Fr. Bernas, the two sentences Ex: Philippine witness protection program.
of Section 7 guarantee only one general right, that is, the right
to information on matters of public concern. The right of access (c) the information, record or document sought falls within the
to official records is given as an implementation of the right to concepts of established privilege or recognized exceptions as
information. Thus, the right to information on matters of public may be provided by law or settled policy or jurisprudence;
concern is both the purpose and the limit of the right of access
to public documents. Thus, too, regulatory discretion must Ex: Presidential communications privilege (in general, an
include both authority to determine what matters are of public executive privilege)
concern and authority to determine the manner of access to
them. (d) such information, record or document comprises drafts of
decisions, orders, rulings, policy decisions, memoranda, etc.;
LIMITS TO THE RIGHT TO INFORMATION: In the case of
IDEAL v. PSALM, SC distinguished the duty to disclose Ex: Drafts of judicial decisions (can’t access that), EXCEPT
information from the duty to permit access to information on the final copies of these documents. As a matter of fact, the
matters of public concern under Sec. 7, Art. III of the concerned government agency may destroy these documents.
Constitution. Unlike the disclosure of information which is
mandatory under the Constitution, the other aspect of the (e) it would disclose information of a personal nature where
people’s right to know requires a demand or request for one to disclosure would constitute a clearly unwarranted invasion of
gain access to documents and paper of the particular agency. personal privacy;
Moreover, the duty to disclose covers only transactions
involving public interest, while the duty to allow access has a Ex: Data Privacy Act and information covered by this law are
broader scope of information which embraces not only likewise mentioned in this particular IRR.
transactions involving public interest, but any matter contained
in official communications and public documents of the (f) it would disclose investigatory records compiled for law
government agency. Such relief must be granted to the party enforcement purposes or information which if written would be
requesting access to official records, documents and papers contained in such records, but only to the extent that the
relating to official acts, transactions, and decisions that are production of such records or information would:

From the Discussions of Atty. Enrique Jr. Bonocan


University of Mindanao College of Law
REVIEWER ON CONSTITUTIONAL LAW II | CYNDI SATORRE-BICERA | 2nd Sem | S.Y. 2021-2022 38

(i) interfere with enforcement proceedings; Q: May the public request for the copies of SALNs,
(ii) deprive a person of a right to a fair trial or an impartial Personal Data Sheet (PDS) and CV of the SC Justices?
adjudication;
(iii) disclose the identity of a confidential source and in the case
of a record compiled by a criminal law enforcement authority in RE: Request for the copies of the 2008 SALN, PDS OR
the course of a criminal investigation, or by an agency CV of The Justices of the Supreme Court and Officers
conducting a lawful national security intelligence investigation, and Employees of the Judiciary
confidential information furnished only by the confidential
source; or RE: Request of Philippine Center for Investigative
(iv) unjustifiably disclose investigative techniques and Journalism [PICJ] for the 2008 SALN and PDS of the
procedures; Ex: those records under the custody of the police Court of Appeals Justices
in the conduct of fact-finding investigation in relation to the
commission of a particular crime. The SC answered in the affirmative, the court finds no
reason to deny the public access to the SALNs, PDS and
(g) it would disclose information the premature disclosure of CVs of the justices of the court and other magistrates of the
which would: judiciary subject of course to the limitations provided under
(i) in the case of a department, office or agency which agency RA 6713. The court notes the valid concerns of the other
regulates currencies, securities, commodities, or financial magistrates regarding the possible illicit motives of some
institutions, be likely to lead to significant financial speculation individuals in their request for access to such personal
in currencies, securities, or commodities, or significantly information and their publication. However, custodians of
endanger the stability of any financial institution; or public documents must not concern themselves with the
(ii) in the case of any department, office or agency be likely or motives, reasons and objects of the persons seeking access
significantly to frustrate implementation of a proposed official to these records. Rather, the moral or material injury which
action, except that subparagraph (f) their misuse might inflict on others is the requestor’s
(iii) shall not apply in any instance where the department, office responsibility and lookout. Any publication is made subject
or agency has already disclosed to the public the content or to the consequences of the law. While public officers in the
nature of its proposed action, or where the department, office custody or control of public records have the discretion to
or agency is required by law to make such disclosure on its regulate the manner in which records may be inspected,
own initiative prior to taking final official action on such examined or copied by interested persons, such discretion
proposal. does not carry with it the authority to prohibit access,
inspection, examination, or copying of the records. After all,
REMEMBER: As a member of the public, you cannot just come public office is a public trust. Public officers and employees
to the concerned agency and ask them to show you and even must, at all times, be accountable to the people, serve them
allow you to copy these documents at any time that suits your with utmost responsibility, integrity, loyalty, and efficiency, act
convenience. That could only be possible if of course you do it with patriotism and justice, and lead modest lives – (Dogma
during official working hours. found in Sec 1, Article 11).

IMPORTANT CASE LAW DOCTRINE:


Initiatives for Dialogue and Empowerment through
IDEAL vs. PSALM – explanation on the relationship between Alternative Legal Services, Inc. (IDEAL, Inc.) v Power
these constitutional rights of the people to information and the Sector Assets and Liabilities Management Corp.
duty of the government to disclose. (PSALM) G.R. No. 192088, October 9, 2012.

Chavez vs. Public Estates Authority and Chavez v PCGG – FACTS: PSALM is a GOCC mandated by RA 9136 (Electric
exceptions provided for by case law as regards to this Power Industry Reform Act of 2001 or the EPIRA Law) to
particular constitution of right. In both cases, the SC manage the orderly sale, disposition, and privatization of the
ratiocinated that like all constitutional guarantees, the right to assets of the NPC over a 25-year period. PSALM held a
information with its right of access to its official records is not public bidding for the sale of AHEPP, a 246-MW
absolute. While providing guarantee for this particular right, the hydroelectric power plant and awarded the sale to K-Water,
Constitution also provides that the people’s right to know is But even before K-Water was given the Notice of Award,
limited to matters of public concern further subject to such IDEALS had been sending letters to PSALM to request for
limitation as may be provided for by law. copies of documents pertaining to the sale. The first letter
requests for copies of the Terms of Reference and proposed
AKBAYAN vs. Thomas Aquino – this case involved the bids submitted by the bidders. There was no response
JPEPA while negotiation were still on going between the because at the time no bids had been submitted yet
representatives of the Philippine and Japanese government, (updates about the ongoing bidding were posted on the
there were people, among others, members of AKBAYAN who PSALM website). The second letter requested information
would want to have access to records particularly those offers regarding the winning bidder, such as company profile,
and counteroffers made by both parties prior to the conclusion contact person, office address, and Philippine registration.
of JEPEPA. One of the issues raised by the government in that Despite press releases announcing K-Water as the winning
case is that the requesting public cannot yet gain access to bidder, PSALM failed to sufficiently provide the petitioners
those documents because they were not matters of public with the information they were asking for, almost as if
concern but the SC held that JPEPA is a public concern PSALM officials were trying to hide something.
covered by the right to information here sought to meet the
requirement that must be a matter of public concern. From the ISSUES: Was there compliance on the rule of transparency
nature of the JPEPA as an international trade agreement, it is on the part of PSALM?
evident that the Philippine and Japanese offers submitted
during the negotiations towards its execution are matters of HELD: NO. Aside from its duty to disclose material
public concern. Thus, respondents do not dispute. They only information regarding the sale of AHEPP, PSALM further
claim that diplomatic negotiations are covered by the doctrine has the duty to allow access to information on matters of
of executive privilege, thus constituting an exception to the public concern. This burden requires a demand or request
right to information and the policy of full public disclosure. from a member of the public, to which the right properly
belongs. "The gateway to information opens to the public the

From the Discussions of Atty. Enrique Jr. Bonocan


University of Mindanao College of Law
REVIEWER ON CONSTITUTIONAL LAW II | CYNDI SATORRE-BICERA | 2nd Sem | S.Y. 2021-2022 39

following: (1) official records; (2) documents and papers be penalized administratively.
pertaining to official acts, transactions, or decisions; and (3)
government research data used as a basis for policy ISSUE: Were the Petitioners correct?
development.
HELD: NO. Those public-school teachers who went on a
PSALM veritably violated the petitioners’ right to information. sit-down strike were actually staging a strike. Such an act
It should have permitted access to the specific documents was invalid. The question of whether the concerted mass
containing the desired information, in light of the disclosure actions launched by the public school teachers has long
of the same information thus made in its website. The been settled. In the case of Manila Public School Teachers
documents referred to are neither confidential nor privileged Association, et al. v. Laguio, Jr., it was held that, these 'mass
in nature, as the gist thereof had already been published in actions' were to all intents and purposes a strike; they
the news bulletins in the website of PSALM, and as such, constituted a concerted and unauthorized stoppage of, or
access thereto must be granted to the petitioner. On the absence from work which it was the teachers' duty to
contrary, the documents requested partake of the nature of perform, undertaken for essentially economic reasons. It is
official information. an undisputed fact that there was a work stoppage and that
petitioners' purpose was to realize their demands by
withholding their services. The fact that the conventional
Section 8, Article III term 'strike' was not used by the striking employees to
RIGHT TO FORM ASSOCIATIONS describe their common course of action is inconsequential,
since the substance of the situation, and not its appearance,
will be deemed to be controlling. It is a well-settled rule in
Section 8. The right of the people, including those employed in this jurisdiction that employees in the public service may not
the public and private sectors, to form unions, associations, or engage in strikes, mass leaves, walkouts, and other forms of
societies for purposes not contrary to law shall not be mass action that will lead to the temporary stoppage or
abridged. disruption of public service. The right of government
employees to organize is limited to the formation of unions
This provision is in relation to Sec. 2(5) of Art. IX-B of the CSC. or associations only, without including the right to strike.
And the enumeration of the constitutional powers of the CSC. Moreover, the general proposition is that a public official is
Sec. 2(5) of Art. IX-B: “The right to self-organization shall not not entitled to any compensation if he has not rendered any
be denied to government employees.” Section 8, Art. III is like service. As he works, he shall earn. Since petitioners did
a restatement strengthening the right of all people to not work during the period for which they are now claiming
self-organization. [in-depth discussion in Labor Relations Law.] salaries, there can be no legal or equitable basis to order
the payment of such salaries."
In a certain CBU, when the management does not receive from
the workers a proposal to bargain, the law says that it is
NOTE: Sec. 8 does not only cover unionism; it also includes all
incumbent upon the employer or management to initiate talks
other things which involves association.
for purposes of collective bargaining.
Examples: Drivers of PUJs or tricycle drivers (TODA), they
Q: May government workers also be allowed to unionize?
have associations. The very catch all of this is the purpose of
A: YES but with limitations.
organization, it must not be contrary to law.
Also, in relation to Sec. 2(5) of Art. IX-B, government workers
Q: Do you commit a crime by merely being a member of
are entitled to unionize, just like the workers in the private
the CPP-NPA knowing that it is an aggrupation of
sector. As a matter of fact, when EO 180 was issued, it
criminals not just because they are rebels but they are
contained rules that should be observed when it comes to
now considered as terrorists?
unionization in the public sector. However, the number of rights
is not the same in the private and public sectors; the latter is
A: BEFORE, in the case of People v Hernandez, the SC held
limited. According to EO 180, these government workers
that mere membership in the Communist Party or in the
unions must be registered in the CSC and the Department of
CLO renders the member liable, either of rebellion or of
Labor through the BLR. Also, although the workers in the
conspiracy to commit rebellion, because mere
public sector are entitled to unionization, they cannot stage
membership and nothing more merely implies advocacy of
strikes because the terms and conditions of their employment
abstract theory or principle without any action being
are fixed by law and cannot be subject to stipulation or
induced thereby; and that such advocacy becomes criminal
bargaining. [MPSTA and SSSDA cases]
only if it is coupled with action namely, actual rebellion or
conspiracy to commit rebellion, or acts conducive thereto or
Strike – under the Labor Code, strike is the temporary
evincing the same. SUBSEQUENTLY, then President Marcos
cessation of work because of a labor dispute.
signed the Anti-Subversion Act. It is explicit under Sec. 4, RA
1700, that whoever knowingly, willfully and by overt acts
Two (2) grounds for the workers to validly conduct a strike:
affiliates himself with, becomes or remains a member of the
1. bargaining deadlock; 2. unfair labor practice (ULP).
Communist Party of the Philippines and/or its successor or of
any subversive association as defined in section two hereof
GESITE v COURT OF APPEALS shall be punished by the penalty of arresto mayor and shall be
disqualified permanently from holding any public office,
FACTS: Petitioners (public school teachers) admitted that appointive and elective, and from exercising the right to vote.
they participated in concerted mass actions in Metro
Manila from September to October 1990 which temporarily
SECTION 9 ART. III EMINENT DOMAIN
disrupted classes. They went on a sit-down strike — they go
to school but they will just sit down and not do anything.
Section 9. Private property shall not be taken for public use
Petitioners claim that they were merely exercising their without just compensation.
constitutional right to peaceably assemble and petition the
government for redress of grievances. Thus, they may not Three (3) inherent powers of the State:

From the Discussions of Atty. Enrique Jr. Bonocan


University of Mindanao College of Law
REVIEWER ON CONSTITUTIONAL LAW II | CYNDI SATORRE-BICERA | 2nd Sem | S.Y. 2021-2022 40

(1 Police power; (2) Power to tax; and


(3) Power of eminent domain (power to expropriate) — the There were many instances where the Government has taken
power of the state to take private property for public use and private property, usually vast tracts of land for the
with just compensation. implementation of housing programs.

The constitution requires particularly under Sec. 9 of Art. 3 that In the case of Sumulong Vs. Guerrero, the alleged public
private property shall not be taken for public use without just purpose of the taking, was questioned simply because the
compensation. private land was taken for purposes of the state's housing
project. According to the petitioners, housing projects cannot
Ex: Project of the national government i.e. the Mindanao pass the required constitutional scrutiny on public use
Railways, road expansions or new roads. The just because, once the construction of the house is completed and
compensation must be the fair market value (zonal value) of already awarded to the beneficiary, and the moment the
the property involved. The issue of just compensation is always beneficiary takes possession of the house, it becomes private.
judicial, if the negotiation on the compensation is not met the But the Supreme Court here ruled otherwise because there
government can file an expropriation case or proceeding were a notable number of the beneficiaries who were to
against the private person but only to settle the just receive housing units from the government. There was real
compensation which will be determined by the Court. public use insofar as the taking was concerned. Even if not
ALL can benefit, the State is mandated by law, particularly by
The ultimate right of sovereign power to expropriate not only the constitution, to make good with its housing project or
public but private property owned by all citizens within the housing responsibilities as one of the required public services it
territorial sovereignty for public purpose may be taken. This is is supposed to deliver to the public.
an inherent power and cannot be taken away from the
Government. **railway (train) projects, or schools, that is for public use.

Q: May the LGUs exercise the power of eminent domain? Just compensation – the rule is that the State is going to pay
Yes, by virtue of this well-acclaimed constitutional principle of the fair market value of the property.
valid delegation power.
But there was no strict observance of the rule back in the
Fr. Bernas: the power to expropriate pertaining to local 1960s. The government can just pay the landowner any
government units, in a strict sense, is actually called the power amount as long as the owner agrees without really knowing the
of inferior domain. If it’s the national government that fair market value of the land. Question, are the heirs entitled to
exercises this power, it’s called the power of eminent domain or just compensation from the government? The answer is YES.
the power of expropriation. But when it comes to local
government units, it is properly termed as the power of inferior The just compensation shall be computed based on the fair
domain. It was said that in the hands of government agencies, market value at the time of the taking, not at the time they
local governments and public utilities is merely a delegated asserted the right. — that is the reckoning period.
power.
The heirs cannot also deny or assail the “time of the taking”
Q: What are the constitutional requisites of the exercise of because there was already a payment made and there has to
the power of eminent domain? be certain documents found in the appropriate governmental
depository to prove that the project was used on that specific
A: There are two (2) Constitutional Limitations here: year.
1. The purpose of the taking which must be for public use; and
2. Just compensation must be given to the private owners. If the taking is done just recently, the basis of the
determination of just compensation on the day the government
Q: What is the extent of the term “public use” as a took possession of the property.
requisite in the exercise of the power to expropriate?
A: There are actually two (2) schools of thought insofar as this Q: Can the State take possession of the private property
matter is concerned. sought to be expropriated immediately?
A: YES. Upon payment of 15% of the just compensation. The
Under the old school, public use simply means the possession remaining balance will be paid after the required paperworks
or the use of particular property involved by the public or by are complied with.
certain public or government agencies. However, our legal
authorities have introduced a new school of thought as regards Another unique exercise by the state of its power of
the definition of public use – an expansive definition of the term eminent domain. That's in the case of RA No. 6657 or the
public use. Comprehensive Agrarian Reform Law (CARL). Under this
program, the government will take possession of a particular
PUBLIC USE, according to case law, means public usefulness, agricultural land both publicly and privately owned. The
utility or advantage, or what is productive of general benefit so purpose here is to distribute these lands to the landless. But if
that any appropriating of private property by the state under its you become a beneficiary under CARL, you do not receive this
right of eminent domain, for purposes of great advantage to the piece of land for free, there’s a fee. Once you become a CARL
community, is a taking for public purpose. beneficiary, the government will issue a title or CLOA
(Certificate of Land Ownership Award), you are the beneficiary
In Justa Guido vs. Rural Progress, the government intended but you will have to pay the government the monthly
to expropriate private property for the benefit of about 30 or 50 amortization to the Land Bank of the Philippines.
individuals for their housing. The decision handed down by the
Supreme Court in that case, states that it was not legally There were instances where the payment of bonds was also
considered as public use insofar as the norm governing the being questioned. Where the government paid 50% cash to
power of eminent domain is concerned. Because there were those landowners for expropriation but the remainin 50% is
only a few beneficiaries. But this does not mean that the state paid in bonds – negotiable bonds issued by banks. The
can no longer take private lands for purposes of sustaining or Government is allowed to pay via tax credits. This was
implementing housing projects pursuant to its mandate under questioned by the landowners to be invalid because there was
the constitution. Housing is actually one of that specific type of no actual payment of just compensation and asserts that it is
public services which the state is bound to deliver. not under Mercantile Law because it is not a legal tender. In

From the Discussions of Atty. Enrique Jr. Bonocan


University of Mindanao College of Law
REVIEWER ON CONSTITUTIONAL LAW II | CYNDI SATORRE-BICERA | 2nd Sem | S.Y. 2021-2022 41

fact, in contracts of sale, the consummation of the contract of


sale must be by way of the delivery of payment which must be
in legal tender (official Philippine currency). Even checks are FACTS: Lahug Airport was owned by the Ouano’s the
not legal tender until it is encashed. But the Supreme Court portion of that was taken by the State for the purpose of
justified the payment scheme adopted in the implementation of airport expansion. After the expropriation proceedings and
the CARL by saying that this is actually the revolutionary form the Ouano’s were already paid, the project did not push
of expropriation and there is a need for this kind of through, worse, some of the portions were taken by
expropriation in order to implement the program because this squatters and some portions were turned into commercial
involved hundreds of thousands of hectares of lands, and that establishments. The Ouano’s want to take it back but the
would entail billions of pesos on the part of the government. Government said that they can no longer take it back
Q: Does the principle of res judicata apply to expropriation because the expropriation proceedings are over. However,
proceedings? the Supreme Court awarded back the land in question to the
owner.
Res judicata refers to the rule that a final judgment or decree
on the merits by a court of competent jurisdiction is conclusive The Rule is that: A condemnor (Government) should commit
of the rights of the parties or their privies in all later suits on all to use the property pursuant to the purpose stated in the
points and matters determined in the former suit. petition for expropriation, failing which it should file another
petition for the new purpose. If not, then it behooves the
Simply put, you can no longer file another case because the condemnor to return the said property to its private owner, if
issue had already been passed upon by the judicial court and the latter so desires. Of course with the private owner’s
in fact, that decision had already become final and executory, payment of the prevailing value of the land.
hence, can no longer be disturbed.

Ex: The State will file an expropriation proceeding against a SECTION 10, ARTICLE III
private owner (in case there is no conformity from the land NON-IMPAIRMENT CLAUSE
owner as to the just compensation offered by the government
because it’s only the Court who can determine the just
compensation). In the end, the Government lost maybe Section 10. No law impairing the obligation of contracts shall
because hindi sinunod ng government ang procedure. be passed.

Say next year, Can the government go back to that property BEFORE, during the time of the Americans, the government, in
again for the purpose of expropriation? order to evade its obligations under a contract it entered into
with private entities, they will just pass a law.
A: YES. The principle of res judicata does not apply to
expropriation proceedings. The Government can still exercise
its power of eminent domain to take that same private property Ex: Government enters into a contract with ABC Corporation
for purposes of expropriation provided there is just for the construction of the airport. When the project has been
compensation. completed, the Government will not pay because they thought
the amount in the contract is excessive (15% surcharge for the
inflation rate). What it will do is, Congress will pass a law to
QUEZON CITY V. ERICTA, G.R. No. L- 34915 6/24/1983 reduce the price. That is not allowed because it is a clear
violation of the sanctity of contracts. The contract has already
The City of QC passed an ordinance requiring private been executed. Under the law on Obligations and Contracts,
cemeteries to allot 6% of the total area of the cemetery for as a general rule, both parties must perform their respective
pauper burials. The cemetery owners complained because it obligations. Especially on the part of the government.
amounts to taking and in violation of the 1987 Constitution
because it was without just compensation.
THE GOVT OF THE PHILIPPINE ISLANDS vs. FRANK G.
HELD: There is no reasonable relation between the setting R. No. 2935; March 23, 1909
aside of at least 6% of the total area of private cemeteries
for charity burial grounds of deceased paupers and the In 1903, in the city of Chicago, in the state of Illinois, in the
promotion of health, morals, good order, safety, or the United States, the defendant, through a respective of the
general welfare of the people. The ordinance is actually a Insular Government of the Philippine Islands, entered into a
taking without compensation of a certain area from a private contract for a period of two years with the plaintiff, by which
cemetery to benefit paupers who are in charge of the the defendant was to receive a salary of 1,200 dollars per
municipal corporation. Instead of building or maintaining a year as a stenographer in the service of the said plaintiff,
public cemetery for this purpose, the city passes the burden and in addition thereto was to be paid in advance the
to private cemeteries. The expropriation without expenses incurred in traveling from the said city of Chicago
compensation of a portion of private cemeteries is not to Manila, and one-half salary during said period of travel.
covered by Section 12(t) of RA 537, the Revised Charter of There was a disagreement where it was alleged that both
Quezon City which empowers the city council to “prohibit the parties expressly agreed to said contract that Laws No. 80
burial of the dead within the center of population of the city and No. 224 should constitute a part of said contract.To the
and to provide for their burial in a proper place”. Even the complaint of the plaintiff the defendant filed a general denial
LGC, BP 337 provides in Section 177 (q) that a and a special defense, alleging in his special defense that
Sangguniang panlungsod may “provide for the burial of the the Government of the Philippine Islands had amended
dead in such place and in such manner as prescribed by law Laws No. 80 and No. 224 and had thereby materially altered
or ordinance” it simply authorizes the city to provide its own the said contract.
city owned land or to buy or expropriate private properties to
construct public cemeteries. Q: Will the amended law negatively affect the existing
contract? No. The subsequent amendment of the statute
could not have an effect of amending the terms of the
contract.
ANUNCIACION VDA. DE OUANO ET AL VS. REPUBLIC,
G.R. NO. 168770, 09 February 2011
Another example, during the infancy of the VAT law,

From the Discussions of Atty. Enrique Jr. Bonocan


University of Mindanao College of Law
REVIEWER ON CONSTITUTIONAL LAW II | CYNDI SATORRE-BICERA | 2nd Sem | S.Y. 2021-2022 42

demand that said deposits should be considered as made with


Ex: The vatable goods – you buy a pair of shoes and under the an implied specification of currency, and hence Article 307 of
law, the shoes is subject to VAT so if the price is 1,000.00, the the Code of Commerce, which provides that "when the
seller is obliged to add on 10% of that selling price deposits consist in cash with specification of the currency
representing the VAT. So the total price now becomes constituting the same, the increase or reduction in value
P1,100.00 because of the 10% VAT. (Now it’s already at 12%). suffered by the same shall be for the account of the depositor,"
Once the seller accepts the payment, the seller has the legal
duty to remit the P100 to the BIR. HELD: There is no violation of the sanctity of contract clause.

Assuming that there is a contract where it covers 10% VAT on


the price and subsequently, the new law on 12% VAT is SECTION 11, ARTICLE III
passed, will the VAT increase to 12% instead? NO. Because
applying that amended law will be violative of the sanctity of Section 11. Free access to the courts and quasi-judicial bodies
the contract laws protected by Section 10 of Article III. and adequate legal assistance shall not be denied to any
person by reason of poverty.
VAT is a kind of excise tax placed upon the shoulders of not of
the seller but upon the buyer. In Remedial Law, there are specific matters related to this like
pauper litigation. Ex: When a person files a case, especially
In GASPAR v. MOLINA – there was this law prescribing that the civil cases, there is a corresponding docket fee that needs
money judgments rendered by courts should be stated in terms to be paid. And those docket fees like those money claims will
of the new Philippine currency in the amount equivalent to the be based on a certain percentage from the claim. If the person
value of the old currency. The law was questioned as violative has no means to pay for it, the law will allow for the litigation of
of the sanctity of contract laws. The Supreme Court held that it the case but there are certain procedures for it like establishing
is not violative of the constitution because it did not require the that the client is really indigent. Another thing, in PAO (seems
debtor to pay more nor the creditor to receive less than they to be the biggest law firm in this jurisdiction), it caters to the
were required to pay or receive under the former law. Although legal needs of the indigents – those that cannot afford the
the currency is new, the value was still based on the old services of a private counsel.
currency. So there was no problem here.

BUT… that is not the case in CLEMONS VS. NOLTING – this


—- END OF MIDTERM COVERAGE —-
was when we were still under American rule. There was this
law passed by the Government decreeing to satisfy an
obligation (government’s debt – foreign loan) to pay in USD by
the payment of PHP at the rate of P2 to a $ when the Section 12. Rights During Custodial Investigation
commercial value of the USD was much more. Where in fact,
the dollar rate at that time was already P4. This law clearly
violates the sanctity of contract clause (non-impairment clause) Section 12. (1) Any person under investigation for the
protected by Section 10, Article III of the Constitution apart commission of an offense shall have the right to be informed of
from that, it would also amount to unjust enrichment. his right to remain silent and to have competent and
REMEMBER: This is applicable on contracts entered into by independent counsel preferably of his own choice. If the
the Government. Because Article III is addressed to the State. person cannot afford the services of counsel, he must be
provided with one. These rights cannot be waived except in
HILADO V. DELA COSTA – this case involves Japanese war
notes which was made a legal tender during the Japanese writing and in the presence of counsel.
occupation. When the occupation was over, the notes no
longer had value. However, there were notes deposited in the (2) No torture, force, violence, threat, intimidation, or any other
bank. When the depositors wanted to withdraw it the Bank means which vitiate the free will shall be used against him.
denied saying that the notes no longer had value. (This case is Secret detention places, solitary, incommunicado, or other
a bit complicated because the Supreme Court treated this case similar forms of detention are prohibited.
as one for DEPOSIT).

BANKS AND BANKING; DEPOSIT OF WAR NOTES DURING (3) Any confession or admission obtained in violation of this or
OCCUPATION BECAME VALUELESS; WHO WILL BEAR THE Section 17 hereof shall be inadmissible in evidence against
LOSS. — It may safely be laid down as a rule that when a him.
deposit is made with a bank or a person of notes made legal
tender or currency by the military occupant of an enemy (4) The law shall provide for penal and civil sanctions for
territory, and the occupation does not ripen into a conquest by violations of this section as well as compensation to and
the occupant because the territory is liberated and reoccupied
by its legitimate government, the deposit must be considered rehabilitation of victims of torture or similar practices, and their
as with specification of currency, that is, as a deposit of money families.
made legal tender or currency by the occupant, without
necessity of stating it expressly, unless there is evidence to the THREE (3) LAWS RELATED TO ARTICLE 12:
contrary, because it is the only kind of money or legal currency
in circulation after the genuine money of the territory has 1. RA 7438 – Criminalization of the Violation of the Rules
disappeared from circulation. Therefore, if such currency
Setforth in Section 12 on the Right of a Person under Custodial
becomes valueless, the depositor shall have to suffer the loss,
because the currency so deposited is exactly of the same Investigation.
condition and validity as that kept in the pockets or safe of the
depositor. 2. RA 7945 – Philippine Anti-Torture Law

IMPLICATION AS TO SPECIFICATION OF CURRENCY. — 3. Article 33, NCC – In cases of defamation, fraud, and
While it does not expressly appear that plaintiff’s deposit in physical injuries, a civil action for damages, entirely separate
Japanese war notes were made with a specification of the
and distinct from the criminal action, may be brought by the
currency deposited, the minimum requirements of justice

From the Discussions of Atty. Enrique Jr. Bonocan


University of Mindanao College of Law
REVIEWER ON CONSTITUTIONAL LAW II | CYNDI SATORRE-BICERA | 2nd Sem | S.Y. 2021-2022 43

injured party. Such civil action shall proceed independently of RA 7438 for violation of the rules set forth in Section 12 on the
the criminal prosecution, and shall require only a right of a person under custodial investigation.
preponderance of evidence.
INSTANCES WHEN SECTION 12. CANNOT BE INVOKED:
NOTE: Section 12 is important as this is synonymous with this
doctrine taught by American Jurisprudence called the Miranda 1. CONFESSION MADE IN FRONT OF THE MEDIA – PP v.
Warning. The Miranda Warning is a very good legal principle ORDONIO
involving respect to the protection of human rights culled from
the two most important American cases of Escobedo vs Illinois 10 years ago, there was this rich guy who has a cousin, a
and Miranda vs Arizona. This is where the Constitution member of the TF Davao, this rich guy lent money to this
mandates that a police officer or law enforcement officer soldier. Later, the soldier could not pay and got irritated
placing a person under arrest either by virtue of a warrant or by because the rich guy kept bugging him of his debt. One day,
virtue of a warrantless arrest, must state the Miranda rights of the soldier took the cousin somewhere in Mandug, Davao City
the person to wit: and killed him and buried him in the cliff of the Banana
Plantation. Some drivers saw him who found him to be too
“You have the right to remain silent. Anything you say can and suspicious coming out of the Banana Plantation. The soldier
will be used against you in a court of law. You have a right to was apprehended and it happened that the media was there.
an attorney. If you cannot afford an attorney, one will be He confessed to the after being interviewed by one of the
appointed for you.” reporters saying, “Humihingi po ako ng sorry sa auntie ko, di
ko po yun sinasadya, nagdilim lang ang paningin ko.” During
1. The right to be informed of his right to remain silent; trial, the recorded video was used by the prosecution as
2. The right to have competent and independent counsel evidence, in his defense, he imposed his constitutional right
preferably of his own choice. If the person cannot afford the under Section 12 saying that he was interviewed and was not
services of counsel, he must be provided with one; Mirandized.
3. These rights (Miranda) cannot be waived except in writing
and in the presence of counsel. – two requisites must concur: The Court here held that the accused is NOT CORRECT.
(1) in writing and (2) assisted by a counsel – for the absence of Because Section 12 may only be invoked against the
one will invalidate the other. government or its agent acting for or its behalf. The radio
announcer or tv reporter is not one of the agents of the State.
The rights under Section 12, Par. 1 – may only be invoked by a The incriminating statement from the accused is ADMISSIBLE
person when in a strict legal sense he/she is already under in court for the exclusionary rule found in Paragraph 3 cannot
custodial investigation? be applied.

Q: How do we determine if a person is already under 2. CONFESSION MADE IN A CONFIDANT THAT IS A LAW
custodial investigation? ENFORCEMENT OFFICER – PP v. WELLA

In the case of Escobedo v. Illinois, the Court said that where GR: If the confession is done in front of a law enforcement
a police investigation is no longer a general inquiry into an officer is INADMISSIBLE.
unsolved crime but has begun to focus on a particular suspect XPN: If the accused made the confession to a mayor (also
in police custody and the police carry out an interrogation that considered as a law enforcement officer) but not because he
causes them to solicit incriminating statements. was a mayor but because he was his confidante as it turns out
that they were best of friends at that time. Here, the
Ex: An incident of stabbing occurred and the suspect fled, you confession, if done before a government official without
called 911, when the police officers came and asked you, “who complying with the requirements of Section 12, is
was stabbed?” – you cannot say that you should be read your ADMISSIBLE in evidence.
Miranda rights yet because this is only a general inquiry. 3. ON SPONTANEOUS STATEMENTS – Ex. A person
Hence, you are not yet placed under custodial investigation. confessed spontaneously to a desk officer on duty for the day
that he killed a person. If that police officer testifies as to the
BUT, if another person points you as the suspect and the fact that the accused made the confession before him in his
police uttered, “Why did you stab him?” – then you are placed presence, it is admissible because the statement was voluntary
under custodial investigation because the police is focusing and the person was not placed under custodial investigation,
upon you as the possible suspect. he willingly walked in the police precinct to confess.

Even if the real suspect was being successfully apprehended, However, insofar as the further confessions or questions are
it cannot be said that he is already under custodial made such as the manner of killing, the motive of killing, etc. is
investigation. The RECKONING point is when the officers start NO LONGER SPONTANEOUS, what is spontaneous is only
to ask you incriminating questions. upto the extent that the police officer heard that the accused
committed the crime. What the police officer should do is to
REMEMBER: Strict compliance with the requirement under place the person under custodial investigation and read him his
Section 12 is compulsory on the part of government agents. Miranda right until then, any answers to the questions he
Thus, if the confession is obtained by these officers without obtained is INADMISSIBLE.
observing this rule, the EXCLUSIONARY RULE under the 3rd
paragraph will apply and the officers will also be held liable for 4. PARAFFIN TEST – as a matter of police procedure, the
police will place the accused on a paraffin test (although SC

From the Discussions of Atty. Enrique Jr. Bonocan


University of Mindanao College of Law
REVIEWER ON CONSTITUTIONAL LAW II | CYNDI SATORRE-BICERA | 2nd Sem | S.Y. 2021-2022 44

ruled a couple of times that this is not conclusive) and the


must be made in writing and with the assistance of a
result came back positive. Sec. 12 cannot be invoked because
counsel. Admission under custodial investigation without the
there is no custodial investigation involved there, it is merely
assistance of a counsel is a BARRED. The records are bare
mechanical in nature (soaking the hand in a melted wax).
of any indication that the accused has waived his rights to
Unless of course, while doing the paraffin test, the police officer
counsel, and hence, any of his admissions are inadmissible
starts asking incriminating questions..
in evidence; this extends as well as to the gun surrendered
by the accused.
**THE MIRANDA RIGHTS**

1. RIGHT TO REMAIN SILENT – this is in relation to the REMEMBER!!! Even if the person is GUILTY as hell, if there’s
Constitutional Principle found in Section 14 or the Presumption violation in the rules set forth in Section 12, the guy walks free.
of Innocence.
PP v. AYSON – Private respondent Felipe Ramos was a
2. RIGHT TO HAVE A COUNSEL – in custodial investigation,
ticket freight clerk of the Philippine Airlines (PAL), assigned
there is no case yet filed in court. According to Section 12, the
at its Baguio City station. It has allegedly come to light that
counsel must be the accused’s preferred choice. If he can
he was involved in irregularities in the sales of plane tickets.
afford to hire a private lawyer, his wishes should be followed. If
Ramos was charged with Estafa. The private prosecutors
he can’t afford, PAO lawyers are there to assist as well as
made a written offer of evidence which included the
volunteer lawyers as the case may be.
statement of the accused taken at the PAL office as his
handwritten admission.
a. Competent – a PAO lawyer is a competent lawyer. A
bar flunker is a competent lawyer. The only time a
HELD: It is clear from the undisputed facts of this case that
lawyer is incompetent is if he deliberately neglects his
Felipe Ramos was not in any sense under custodial
duties for the case.
interrogation, as the term should be properly understood,
b. Independent – means that this lawyer must not be
prior to and during the administrative inquiry into the
interested in the prosecution of the case. When the
discovered irregularities in ticket sales in which he appeared
accused during his arrest was assisted by a lawyer
to have had a hand. The constitutional rights of a person
who is also a police officer, any confession made by
under custodial interrogation under Section 20, Article IV of
the accused is INADMISSIBLE. Because that
the 1973 Constitution did not therefore come into play, were
lawyer/police officer is interested in the case. Even a
of no relevance to the inquiry. It is also clear, too, that
judge who is related to the accused must inhibit or
Ramos had voluntarily answered questions posed to him on
must be disqualified.
the first day of the administrative investigation, February 9,
1986 and agreed that the proceedings should be recorded,
Q: When does Custodial Investigation end?
the record having thereafter been marked during the trial of
A: According to Fr. Bernas, for as soon as the interrogation is
the criminal action subsequently filed against him as Exhibit
over and there have been filing of the criminal case with the
A, just as it is obvious that the note (later marked as Exhibit
court.
K) that he sent to his superiors on February 8,1986, the day
before the investigation, offering to compromise his liability
CONFESSION v. ADMISSION – A confession, as
in the alleged irregularities, was a free and even
distinguished from an admission, is a declaration made at any
spontaneous act on his part. They may not be excluded on
time by a person, voluntarily and without compulsion or
the ground that the so-called “Miranda rights” had not been
inducement, stating or acknowledging that he had committed
accorded to Ramos.
or participated in the commission of a crime. The term
admission, on the other hand, is usually applied in criminal
cases to statements of fact by the accused which do not
PP v. BONOLA – FACTS: Bonola was accused of Robbery.
directly involve an acknowledgment of the guilt of the accused
He was arrested while selling fishball at the Public Market.
or of criminal intent to commit the offense with which he is
Inside the Markina Police Station, the police interrogated
charged.
him until he admitted his participation in the crime. The
police officer did not ask the accused if he wanted to be
PP v. ADOR – On the day of the killing, the accused was assisted by a counsel. In other words, nakakuha ng
brought to the police HQ. Now, on the way to the laboratory, uncounselled confession ang mga police which was used
one of the accused told the police escort that he has a gun against him in trial. On appeal, Bonola questioned the
in his house. Of course, after acquiring such information, the admission of such evidence.
police lost no time in retrieving the gun. He also told the
police that he fired the gun on the night of the incident and RULING: SC said that In line with PP v. Morales and PP v.
ballistic examination shows that the bullet found in the Galit, we held in People vs. Decierdo that uncounselled
victim’s head was fired from the recovered gun. confessions are inadmissible in evidence. We elucidated
that when the accused is not assisted by counsel, his
HELD: The confession is INADMISSIBLE with respect to the statement, in contemplation of the law, becomes
gun and to the fact of firing the gun. At the time the accused "involuntary" even if it were otherwise voluntary, in a
made the confession, he was a suspect. Consequently, his technical sense.
right under custodial investigation pursuant to Section 12,
Art. III of the Constitution that is attached, and any waiver

From the Discussions of Atty. Enrique Jr. Bonocan


University of Mindanao College of Law
REVIEWER ON CONSTITUTIONAL LAW II | CYNDI SATORRE-BICERA | 2nd Sem | S.Y. 2021-2022 45

(b) Nature and circumstances of the offense;


In People vs. Dacoycoy and People vs. Pecardal, we
(c) Penalty for the offense charged;
categorically ruled that a waiver of the constitutional right to
(d) Character and reputation of the accused;
counsel shall not be valid when the waiver is made without
(e) Age and health of the accused;
the presence and assistance of counsel.
(f) Weight of the evidence against the accused;
(g) Probability of the accused appearing at the trial; (abscond)
(h) Forfeiture of other bail;
SECTION 13. BAIL
(i) The fact that accused was a Fugitive from justice when
arrested; and
Section 13. All persons, except those charged with offenses (j) Pendency of other cases where the accused is on bail.
punishable by reclusion perpetua when evidence of guilt is
strong, shall, before conviction, be bailable by sufficient Excessive bail shall not be required.
sureties, or be released on recognizance as may be provided
by law. The right to bail shall not be impaired even when the NOTE: Money in Bail can be returned unless the accused
privilege of the writ of habeas corpus is suspended. Excessive violates the terms of his bail, it will be forfeited. Bail is only a
bail shall not be required. security for the accused’s appearance in Court.

BAIL – Bail is the security given for the release of a person in Instances where the presence of the accused is
custody of the law, furnished by him or a bondsman, to indispensable and necessary during trial:
guarantee his appearance before any court as required under
the conditions hereinafter specified. Bail may be given in the 1. When there is a need for the prosecution to establish the
form of corporate surety, property bond, cash deposit, or identity of the accused;
recognizance. (Sec. 1, Rule 114 of the Rules of Court) 2. Whether the accused is a fugitive of justice.

NOTE: The right to bail is recognized by our Constitution. This Q: May a person who is not yet arrested, post bail?
is connected with the right to be presumed innocent. Each one GR: You need to post bail because you are under detention;
of us is presumed to be innocent. The purpose of allowing the freedom of movement is being restricted or withheld from you.
accused to post bail is to secure his attendance. XPN: To abbreviate the process, if the person is still not being
arrested but he knows he will eventually be arrested, he can
Q: When is bail a matter of right? When is it a matter of post bail, consequently he has to surrender, then the court can
court discretion? acquire jurisdiction over his person.

1. Bail as a matter of RIGHT – All persons in custody shall be Even if the person is in the hospital, he still needs to surrender.
admitted to bail as a matter of right, with sufficient sureties, or But not necessarily go to the police. The police can go to the
released on recognize as prescribed by law or this Rule (a) hospital to acquire jurisdiction over his person and read his
before or after conviction by the MeTC, MTC, MCTC and (b) Miranda rights. After that, the counsel can file the bail on behalf
before conviction by the RTC of an offense not punishable by of the accused.
death, reclusion perpetua, or life imprisonment. (Sec. 4, Rule
114 ROC). [NOTE: As of the moment, the capital penalty is Q: Is bail available in extradition proceedings (process of
reclusion perpetua (20Y, 1D - 40 Y used in RPC as opposed to sending a person within the jurisdiction of the requested state
Life Imprisonment which means imprisonment for life, found in to the territory of the requesting state because the person is
some SPL). facing criminal charges of the requesting state. It is only
possible if there exists an extradition treaty between the two
2. Bail as a matter of COURT DISCRETION – if the offense countries or an agreement to that effect.)
charged is a CAPITAL OFFENSE and the EVIDENCE OF
GUILT IS NOT STRONG. This means, notwithstanding that the
GOVERNMENT OF US v. PURGANAN (2002)
punishment is charged with capital offense, if the evidence of
guilt is not strong, the court in its discretion may still grant bail.
FACTS: Pursuant to the existing RP-US Extradition Treaty,
Remember that when the bail is a matter of court discretion, it
the US Government, through diplomatic channels, sent to
is incumbent upon the defense to prove that the guilt of the
the PH Government Note Verbale No. 0522 requesting the
accused is not that strong. The court cannot just grant this
extradition of Mark B. Jimenez, also known as Mario
without giving the prosecution the chance to be heard. Itong
Batacan Crespo. Upon learning of the request, Jimenez
hearing ng bail need not necessarily prove guilt beyond
sought and was granted a TRO by the RTC of Manila to
reasonable doubt. What is needed is that the evidence of guilt
prohibit the DOJ from filing with the RTC a petition for his
is NOT STRONG. [See also Sec. 5, Rule 114 ROC]
extradition. The validity of the TRO was, however, assailed
by the SOJ. Initially, the Court – by a vote of 9-6 dismissed
FACTORS IN FIXING BAIL:
the Petition. The SOJ was ordered to furnish private
respondent copies of the extradition request and its
Section 9, Rule 114 ROC: Amount of bail; guidelines. — The
supporting papers and to grant him a reasonable period
judge who issued the warrant or granted the application shall
within which to file a comment and supporting evidence. The
fix a reasonable amount of bail considering primarily, but not
RTC granted the Motion of Jimenez and set the case for
limited to, the following factors:
hearing in June. In that hearing, petitioner manifested its
reservations on the procedure adopted by the RTC allowing
(a) Financial ability of the accused to give bail;

From the Discussions of Atty. Enrique Jr. Bonocan


University of Mindanao College of Law
REVIEWER ON CONSTITUTIONAL LAW II | CYNDI SATORRE-BICERA | 2nd Sem | S.Y. 2021-2022 46

the accused in an extradition case to be heard prior to the 1. Yes. If bail can be granted in deportation cases, we see
issuance of a WOA. After the hearing, the court a quo no justification why it should not also be allowed in
required the parties to submit their respective memoranda. extradition cases. Likewise, considering that the Universal
As for Jimenez, he sought an alternative prayer: that in case Declaration of Human Rights applies to deportation cases,
a warrant should be issued, he be allowed to post bail in the there is no reason why it cannot be invoked in extradition
amount of P100,000. Thereafter, the RTC issued an Order, cases. After all, both are administrative proceedings where
directing the issuance of a warrant for his arrest and fixing the innocence or guilt of the person detained is not in issue.
bail for his temporary liberty at P1m in cash. After he had Clearly, the right of a prospective extraditee to apply for bail
surrendered his passport and posted the required cash in this jurisdiction must be viewed in the light of the various
bond, Jimenez was granted provisional liberty via the treaty obligations of the Philippines concerning respect for
challenged Order. the promotion and protection of human rights. Under these
treaties, the presumption lies in favor of human liberty. Thus,
ISSUE: WON the right to bail is available in extradition the Philippines should see to it that the right to liberty of
proceedings. every individual is not impaired.

RULING: NO. The court agrees with the petitioner. As 2. An extradition proceeding being sui generis, the standard
suggested by the use of the word “conviction,” the of proof required in granting or denying bail can neither be
constitutional provision on bail quoted above, as well as the proof beyond reasonable doubt in criminal cases nor the
Section 4 of Rule 114 of the ROC, applies only when a standard of proof of preponderance of evidence in civil
person has been arrested and detained for violation of cases. While administrative in character, the standard of
Philippine criminal laws. It does not apply to extradition substantial evidence used in administrative cases cannot
proceedings because extradition courts do not render likewise apply given the object of extradition law which is to
judgments of conviction or acquittal. It is also worth noting prevent the prospective extraditee from fleeing our
that before the US government requested the extradition of jurisdiction.
the respondent, proceedings had already been conducted in
that country. But because he left the jurisdiction of the In his Separate Opinion in Purganan, then Associate
requesting state before those proceedings could be Justice, now Chief Justice Reynato S. Puno, proposed that
completed, it was hindered from continuing with the due a new standard which he termed “clear and convincing
processes prescribed under its laws. His invocation of due evidence” should be used in granting bail in extradition
process now has thus become hollow. He already had that cases. According to him, this standard should be lower than
opportunity in the requesting state; yet, instead of taking it, proof beyond reasonable doubt but higher than
he ran away. preponderance of evidence. The potential extraditee must
prove by “clear and convincing evidence” that he is not a
flight risk and will abide by all the orders and processes of
Now, after 5 years, comes this case of…
the extradition court.

GOVERNMENT OF HONGKONG v. OLALIA (2007)


NB: SC abandoned the ruling in PURGANAN because they
(SC abandoned the ruling in PURGANAN)
realized that it is an extraditee’s constitutional right even if it
was not a criminal proceeding because nothing is more
FACTS: Private respondent Muñoz was charged (in HK) 3
important than the LIBERTY of a person. Invoking the universal
counts of Accepting Advantage as Agent and 7 counts of
covenant of human rights and the international covenant of civil
conspiracy to defraud. Warrants were issued and Hong
and political rights (ICCPR). Furthermore, SC said that “we
Kong filed with the RTC of Manila a petition for the
adhere to the generally accepted principles of international law
extradition of Muñoz, he later filed, in the same case, a
under the Doctrine of Incorporation. Hence, a person subject to
petition for bail which was opposed by petitioner. The
extradition proceedings CAN POST BAIL.”
petition for bail was denied by Judge Bernardo, Jr. (who
eventually inhibited himself from further hearing the case
Q: If the accused is convicted by the trial court, can he still
and raffled off to J. Olalia). Muñoz filed a motion for
post bail on appeal?
reconsideration of the Order denying his application for bail.
This was granted by Olalia. Petitioner filed an urgent motion
A: NOT ANYMORE. In the case of LEVISTE v. CA (Leviste
to vacate the above Order, but it was denied by Olalia.
was convicted by the RTC for the lesser crime of homicide and
Hence, the instant petition. Petitioner alleged that the trial
sentenced to suffer an indeterminate penalty of six years and
court committed GADALEJ in admitting private respondent
one day of prision mayor as minimum to 12 years and one day
to bail; that there is nothing in the Constitution or statutory
of reclusion temporal as maximum.) SC gave emphasis on
law providing that a potential extraditee has a right to bail,
Sec. 13, Art. 3 of the Constitution, after conviction by the trial
the right being limited solely to criminal proceedings.
court, the presumption of innocence terminates or is
overturned and accordingly, the constitutional right to bail ends.
ISSUES:
From then on, the grant of bail is subject to judicial
1. WON a potential extraditee is entitled to post bail.
discretion. At the risk of being repetitious, such discretion
2. What should be the quantum of evidence needed to grant
must be exercised with grave caution and only for strong
such bail to a potential extraditee.
reasons. Considering that the accused was in fact convicted by
the trial court, allowance of bail pending appeal should be
RULING:
guided by a stringent-standards approach. This judicial

From the Discussions of Atty. Enrique Jr. Bonocan


University of Mindanao College of Law
REVIEWER ON CONSTITUTIONAL LAW II | CYNDI SATORRE-BICERA | 2nd Sem | S.Y. 2021-2022 47

disposition finds strong support in the history and evolution of Section 14. (1) No person shall be held to answer for a
the rules on bail and the language of Section 5, Rule 114 of the criminal offense without due process of law.
Rules of Court (5 bail-negating circumstances). It is likewise
consistent with the trial court’s initial determination that the (2) In all criminal prosecutions, the accused shall be
1
accused should be in prison. Furthermore, letting the accused presumed innocent until the contrary is proved, and shall
out on bail despite his conviction may destroy the deterrent enjoy the 2right to be heard by himself and counsel, 3to be
effect of our criminal laws. This is especially germane to bail informed of the nature and cause of the accusation against
pending appeal because long delays often separate him, 4to have a speedy, impartial, and public trial, 5to meet the
sentencing in the trial court and appellate review. In addition, at witnesses face to face, and 6to have compulsory process to
the post-conviction stage, the accused faces a certain prison secure the attendance of witnesses and the production of
sentence and thus may be more likely to flee regardless of bail evidence in his behalf. However, after arraignment, trial may
bonds or other release conditions. Finally, permitting bail too proceed notwithstanding the absence of the accused provided
freely in spite of conviction invites frivolous and time-wasting that he has been duly notified and his failure to appear is
appeals which will make a mockery of our criminal justice unjustifiable.
system and court processes.
Due Process of Law – implies the right of the person affected
Also, in QUI v. PEOPLE – the Supreme Court said the thereby to be present before the tribunal which pronounces
contention of the petitioner is spurious. Certainly, after one is judgment upon the question of life, liberty and property in its
convicted by the trial court, the presumption of innocence, and most comprehensive cases to be heard by testimony or
with it, the constitutional right to bail, ends. otherwise and to have to have the right of controverting by
proof of any material fact. (Medenilla vs. CSC). Due process of
PP VS FITZGERALD – If the penalty imposed by the trial court law is a proceeding which hears before it condemns, the
is imprisonment exceeding six (6) years, the accused shall be punishment of the guilty only proceeds after inquiry and
denied bail, or his bail shall be canceled upon a showing by the renders judgment only after trial.
prosecution, with notice to the accused, of the following or [2 aspects: substantive and procedural due process]
other similar circumstances:
Section 14 in relation to RULE 115: Rights of Accused
(a) That he is a recidivist, quasi-recidivist, or habitual
delinquent, or has committed the crime aggravated by the Section 1. Rights of accused at the trial. — In all criminal
circumstance of reiteration; prosecutions, the accused shall be entitled to the following rights:
(b) That he has previously escaped from legal confinement,
(a) To be presumed innocent until the contrary is proved beyond
evaded sentence, or violated the conditions of his bail reasonable doubt.
without valid justification;
(c) That he committed the offense while under probation, (b) To be informed of the nature and cause of the accusation against
parole, or conditional pardon; him.
(d) That the circumstances of his case indicate the probability
of flight if released on bail; or (c) To be present and defend in person and by counsel at every stage
(e) That there is undue risk that he may commit another of the proceedings, from arraignment to promulgation of the judgment.
The accused may, however, waive his presence at the trial pursuant to
crime during the pendency of the appeal.
the stipulations set forth in his bail, unless his presence is specifically
ordered by the court for purposes of identification. The absence of the
DIPATUAN v. MANGOTARA (charged with a Capital Offense) accused without justifiable cause at the trial of which he had notice
– The rule is very explicit as to when admission to bail is shall be considered a waiver of his right to be present thereat. When
discretionary on the part of the respondent Judge. It is an accused under custody escapes, he shall be deemed to have
imperative that judges be conversant with basic legal principles waived his right to be present on all subsequent trial dates until
and possess sufficient proficiency in the law. In offenses custody over him is regained. Upon motion, the accused may be
punishable by reclusion perpetua or death, the accused has no allowed to defend himself in person when it sufficiently appears to the
court that he can properly protect his right without the assistance of
right to bail when the evidence of guilt is strong. Thus, as the
counsel.
accused in this criminal case had been sentenced to reclusion
perpetua, the bail should have been canceled, instead of (d) To testify as a witness on his own behalf but subject to
increasing it as the respondent Judge did. cross-examination on matters covered by direct examination. His
silence shall not in any manner prejudice him.
According to Fr. Bernas, the better doctrine is that after
conviction of the trial court when the presumption of innocence (e) To be exempt from being compelled to be a witness against
terminates, the constitutional right to bail also terminates. himself.

(f) To confront and cross-examine the witnesses against him at the


NOTE: Life imprisonment is treated as the same as reclusion trial. Either party may utilize as part of its evidence the testimony of a
perpetua. If the charge is murder and the evidence only witness who is deceased, out of or cannot with due diligence be found
indicates homicide (not a capital offense), the right to bail is not in the Philippines, unavailable or otherwise unable to testify, given in
lost. another case or proceeding, judicial or administrative, involving the
same parties and subject matter, the adverse party having the
opportunity to cross-examine him.
SECTION 14. THE RIGHTS OF THE ACCUSED
(g) To have a compulsory process issued to secure the attendance of
witnesses and production of other evidence on his behalf.

From the Discussions of Atty. Enrique Jr. Bonocan


University of Mindanao College of Law
REVIEWER ON CONSTITUTIONAL LAW II | CYNDI SATORRE-BICERA | 2nd Sem | S.Y. 2021-2022 48

(h) To have speedy, impartial and public trials.


4. RIGHT TO SPEEDY, IMPARTIAL, and PUBLIC TRIAL.
(i) To appeal in all cases allowed and in the manner prescribed by law.
Impartial Trial – The judge must be the one disinterested in
First paragraph of Sec. 14 is what we called the right to due the prosecution of the case, so that if the judge happens to be
process. One good essence of criminal procedure is the right the blood relative or a good friend to the accused, then he
of the person to be informed as to why he is criminally charged must inhibit himself and let someone else try the case.
and what crime he is charged with. Stated otherwise, the
following must be observed by the court in affording the Public Trial – This is also one of the recognized Constitutional
accused to his right to due process: rights of the accused wherein the public should not be
a. There must be adherence to the rules in criminal excluded as a general rule in the trial of the criminal case.
procedure.
Speedy – This constitutional right is actually a two-edged
b. He must be tried by a competent court – It means the sword, to paraphrase the late Rev. Fr. Bernas, because it can
issue was tried by the court who has jurisdiction over the case. work for or against the advantage of the accused. By
observing this Constitutional tenet, two may happen: It’s either
c. He must be tried by an impartial judge – meaning, the the CONVICTION or ACQUITTAL is expedited. It works both
judge should be someone not interested in prosecuting the ways.
case. The judge should defer it if it happens in case he has
interest or inhibits himself or there’s automatic disqualification Why do you think this provision exists in the Constitution?
(Rule 137, Rules of Court). When a person is being charged with a criminal offense, it
causes anxiety, stress, etc. because the people (peers) are
Q: Do military courts have jurisdiction over civilians? going to judge. The witnesses may fabricate stories maybe for
GR: NO. Because civilian authorities are always supreme over vengeance and the judges may commit errors, too. This is just
the military. one of the sacrosanct protection afforded by the Constitution
XPN: When civilian courts are not functioning like during upon a person charged with a criminal offense.
Martial Laws.
Among others, the Constitution mandates that the accused is
REMEMBER: due process is not necessarily judicial process entitled to his right to speedy trial. Here, whenever the
and that when it is absolutely necessary for public safety, the prosecution or the court causes undue delay as to the conduct
legal process can be superseded and the exercise of such of the proceedings and eventually, the conclusion of the trial,
must be authorized with the jurisdiction normally vested in the right of the accused to speedy trial is violated. The
court. Possible when civilian courts are not functioning. accused’s right to speedy trial, once violated by the court or
prosecution, may amount to his acquittal. Corollarily, while the
Second paragraph talks about the accused RIGHTS: accused has the right to speedy trial, the Constitution would
neither tolerate delays caused by him. Simply put, when the
1. PRESUMPTION OF INNOCENCE – the GUILT of the accused himself causes unjustified delays in the criminal case,
accused must be proven BEYOND REASONABLE DOUBT. If then he can no longer invoke this Constitutional right. He will
the evidence were with just an iota of doubt, a mere strand of be considered ESTOPPED because he himself caused the
ambiguity, this could be a good reason to dismiss the case. delay. SC emphasized that while it is the duty of the
(Sec. 1 (a), Rule 115 ROC) prosecution to see to it that criminal cases are tried without
2. TO BE HEARD – the right to be notified and the right to be unfounded delays, the accused himself cannot sleep on the
heard; a proceeding which hears before it condemns. Also, it said right but must see to it that the case is tried at an earlier
is the defense who has the onus probandi (burden of proof) to date.
prove that he is not guilty. meaning, he must be able to present
his own evidence. NOTE: The dismissal of the criminal case for a violation of the
right to speedy trial is equivalent to acquittal and is therefore, a
3. TO BE INFORMED OF THE NATURE AND CAUSE OF BAR to subsequent prosecution for the same offense. This is
ACCUSATION AGAINST HIM – this is the during always with prejudice – meaning the case is already
ARRAIGNMENT – initial stage of trial. Where the accused terminated for good. You can no longer file the very same
must be present in court and the charges are being read to case for the second time. Because dismissal due to violation of
him. If the accused is not sure of his answer (as to his guilt), a the right to speedy trial is equivalent to acquittal and therefore
“not guilty” plea will be entered. If the accused said he is NOT a bar to prosecution for the same – principle of double
GUILTY, trial will proceed. If the accused says GUILTY, no full jeopardy.
blown trial will ensue.
In criminal procedure, the difference between a dismissal and
It is the formal mode of implementing the constitutional right of acquittal is: ACQUITTAL is always based on the merits, that is,
the accused to be informed of the nature of the accusation the defendant is acquitted because the evidence does not
against him. It is made before the court where the complaint or show that defendant's guilt is beyond reasonable doubt; but
information has been filed or assigned for trial; and in open DISMISSAL does not decide the case on the merits or that the
court, by the judge or clerk by furnishing the accused a copy of defendant is not guilty.
the complaint or information with the list of the witnesses,
reading it in a language or dialect known to him and asking him A criminal case could be terminated by the court either:
of his plea;

From the Discussions of Atty. Enrique Jr. Bonocan


University of Mindanao College of Law
REVIEWER ON CONSTITUTIONAL LAW II | CYNDI SATORRE-BICERA | 2nd Sem | S.Y. 2021-2022 49

1. Trial on the merits – meaning, both parties have presented but his/her presence for trial cannot be obtained by due diligence or
their evidence and then based on both sets of evidence, the he/she resists appearing at or being returned for trial.
court renders judgment. Either conviction or acquittal of the
(c) Any period of delay resulting from the fact that the accused is
accused; or
mentally incompetent or physically unable to stand trial.

2. A case is terminated – without going into the merits of the (d) If the information is dismissed upon motion of the prosecution and
case, meaning it is “dismissed” without the parties being able thereafter a charge is filed against the accused for the same offense,
to present their evidence. Mostly due to technicalities. Among or any offense required to be joined with that offense, any period of
other things, the dismissal of the criminal case without going delay from the date the charge was dismissed to the date the time
through its merits, may be caused by violation of the right of limitation would commence to run as to the subsequent charge had
the accused to speedy trial. there been no previous charge.
(e) A reasonable period of delay when the accused is joined for trial
with a co-accused over whom the court has not acquired jurisdiction, or
In relation to RA 8493 or “Speedy Trial Act of 1998.” – This as to whom the time for trial has not run and no motion for severance
law requires both the prosecution and the court for the has been granted.
observance:
(f) Any period of delay resulting from a continuance granted by any
(1) observe the prescribed time limits for trial (Sec. 6); justice or judge motu propio or on motion of the accused or his/her
(2) contains the rules in terms of the exclusions or exceptions counsel or at the request of the public prosecutor, if the justice or judge
for the punishability of delays that the State can invoke granted such continuance on the basis of his/her findings that the ends
of justice served by taking such action outweigh the best interest of the
(Sec.10); and
public and the defendant in a speedy trial. No such period of delay
(3) provides for the criminal penalties for the violation thereof. resulting from a continuance granted by the court in accordance with
(Sec. 14) this subparagraph shall be excludable under this section unless the
court sets forth, in the record of the case, either orally or in writing, its
Section 6. Time Limit for Trial. – In criminal cases involving persons reasons for finding that the ends of justice served by the granting of
charged of a crime, except those subject to the Rules on Summary such continuance outweigh the best interests of the public and the
Procedure, or where the penalty prescribed by law does not exceed six accused in a speedy trial.
(6) months imprisonment, or a fine of One thousand pesos (P1,000.00)
or both, irrespective of other imposable penalties, the justice or judge Section 14. Sanctions. - In any case in which counsel for the
shall, after consultation with the public prosecutor and the counsel for accused, the public prosecution or public attorney:
the accused, set the case for continuous trial on a weekly or other
short-term trial calendar at the earliest possible time so as to ensure (a) knowingly allows the case to be set for trial without disclosing the
speedy trial. In no case shall the entire trial period exceed one hundred fact that a necessary witness would be unavailable for trial;
eighty (180) days from the first day of trial, except as otherwise
authorized by the Chief Justice of the Supreme Court pursuant to (b) files a motion solely for the purpose of delay which he/she knows is
Section 3, Rule 22 of the Rules of Court. totally frivolous and without merit;

Sec. 7: Arraignment = 30D from filing of info; After Plea is entered = (c) makes a statement for the purpose of obtaining continuance which
15D to prep for Trial; Trial = commence in 30D after arraignment. he/she knows to be false and which is material to the granting of a
continuance; or
Section 10. Exclusions. – The following periods of delay shall be
excluded in computing the time within which trial must commence: (d) otherwise willfully fails to proceed to trial without justification
consistent with the provisions of this Act, the court may, without
(a) Any period of delay resulting from other proceedings concerning the prejudice to any appropriate criminal and/or administrative charges to
accused, including but not limited to the following: be instituted by the proper party against the erring counsel if and when
(1) delay resulting from an examination of the accused, and hearing on warranted, punish any such counsel or attorney, as follows:
his/her mental competency, or physical
incapacity; (1) in the case of a counsel privately retained in connection with the
(2) delay resulting from trials with respect to charges against the defense of an accused, by imposing a fine not exceeding; fifty percent
accused; (50%) of the compensation to which he/she is entitled in connection
(3) delay resulting from interlocutory appeals; with his/her defense of the accused;
(4) delay resulting from hearings on pre-trial motions: Provided, that
the delay does not exceed thirty (30) days, (2) by imposing on any appointed counsel de officio or public
(5) delay resulting from orders of inhibition, or proceedings relating to prosecutor a fine not exceeding Ten thousand pesos (10,000.00); and
change of venue of cases or transfer from other courts;
(6) delay resulting from a finding of the existence of a valid prejudicial (3) by denying any defense counsel or public prosecutor the right to
question; and practice before the court considering the case for a period not
(7) delay reasonably attributable to any period, not to exceed thirty (30) exceeding thirty (30) days.
days, during which any proceeding concerning the accused is actually
under advisement. The authority to punish provided for by this section shall be in addition
to any other authority or power available to the court. The court shall
(b) Any period of delay resulting from the absence or unavailability of follow the procedures established in the Rules of Court in punishing
the accused or an essential witness. For purposes of this any counsel or public prosecutor pursuant to this section.
subparagraph, an accused or an essential witness shall be considered
absent when his/her whereabouts are unknown and, in addition, 5. RIGHT TO CROSS-EXAMINE – The right of the accused to
he/she is attempting to avoid apprehension or prosecution or his/her
confront the witnesses against him face-to-face. This is
whereabouts cannot be
determined by due diligence. An accused or an essential witness shall
likewise re echoed in Rule 115, Sec. 1(f) of the Rules on
be considered unavailable whenever his/her whereabouts are known Criminal Procedure:

From the Discussions of Atty. Enrique Jr. Bonocan


University of Mindanao College of Law
REVIEWER ON CONSTITUTIONAL LAW II | CYNDI SATORRE-BICERA | 2nd Sem | S.Y. 2021-2022 50

According to Fr. Bernas, this rule has Two-Fold Purposes: (2) When there is a need for the prosecution to identify the
1. To afford the accused the opportunity to test the testimony of accused.
the witness by way of cross-examination; and
2. To allow the judge to observe the deportment and the THREE (3) CONDITIONS FOR TRIAL IN ABSENTIA:
demeanor of the witness while testifying on the witness stand. 1. Accused was already been arraigned;
During Cross-examination, the demeanor of the witness/es can 2. Notice of the trial was duly served to the accused and
be observed especially if one is put on the hot seat. The Judge properly returned; and
will be able to observe this every time he utters the words that 3. Failure of the accused to appear during the trial was
would consist of his statement either for or against the unjustified.
accused.
NB: From the Book of Fr. Bernas: Trial in absentia can also
6. RIGHT TO COMPULSORY PROCESSES – It is recognized take place when the accused voluntarily waives his right to be
by the Constitution for the accused to be able to raise his present. There are, however, restrictive conditions for allowing
defense intelligently, after all, he is presumed innocent. In fact, waiver. The right may be waived "provided that after
I have already emphasized this before, that it is incumbent arraignment he may be compelled to appear for the purpose of
upon the prosecution to prove the guilt of the accused beyond identification by the witnesses of the prosecution, or provided
reasonable doubt and it is not the duty of the accused to prove he unqualifiedly admits in open court after his arraignment that
his innocence. When there is a need for the accused to he is the person named as the defendant in the case on trial.
present defenses, he is always entitled to the production of Reason for requiring the presence of the accused, despite his
evidence for and on his behalf. waiver, is, if allowed to be absent in all the stages of the
proceeding without giving the People's witnesses the
Brady Doctrine (Brady vs. Maryland 1963 case) – The act of opportunity to identify him in court, he may in his defense say
the prosecution in withholding or suppressing evidence that he was never identified as the person charged in the
material for the establishment of the guilt or innocence of the information and, therefore, is entitled to acquittal." Thus, for an
accused is fatal to the prosecution’s case and is a violation of accused to be excused from attending trial, it is not enough
due process. That is the Brady Doctrine in the U.S. that he vaguely agrees to be identified by witnesses in his
jurisprudence, and the same is true in the jurisdiction of the absence. He must unqualifiedly admit that every time a witness
Philippines. mentions a name by which he is known, the witness is to be
understood as referring to him.
To establish the right to continuance by reason of absence
of witnesses, the accused must:
SEC 15. PRIVILEGE OF THE WRIT OF HABEAS CORPUS
1. Show that the witness is really material;
2. That he is guilty of no neglect in previously obtaining
attendance of said witness; Section 15. The privilege of the writ of habeas corpus shall not
3. That the witness will be available at the time desired; be suspended except in cases of invasion or rebellion when
4. That no similar evidence could be obtained. the public safety requires it.

These are the minute rules cited by Fr. Bernas in justifying the It must be read in harmony with Section 18 of Art. VII viz:
existence of this Right to Compulsory Process for the
production of evidence, for and on his behalf. To aid the Section 18. The President shall be the Commander-in-Chief of all
accused to prove his innocence. At least allowed yan ng rules, armed forces of the Philippines and whenever it becomes necessary,
1
he may call out such armed forces to prevent or suppress
kunyari may material witness living in a far away city such as
lawless violence, invasion or rebellion. In case of invasion or
Cebu, it is incumbent upon the prosecution to do everything to
rebellion, when the public safety requires it, 2he may, for a period not
compel and secure the attendance of that witness at the trial if exceeding sixty days, suspend the privilege of the writ of habeas
his testimony is material to prove his innocence. In fact, under corpus or place the Philippines or any part thereof under martial
the Rules of Court, may mga certain rules pertaining to this law. Within forty-eight hours from the proclamation of martial law or the
one, like the witness meron dapat compensation and libreng suspension of the privilege of the writ of habeas corpus, the President
pamasahe. Syempre it would be unfair na gagastos sya ng shall submit a report in person or in writing to the Congress. 3The
sariling pera to give his testimony. Congress, voting jointly, by a vote of at least a majority of all its
Members in regular or special session, may revoke such
proclamation or suspension, which revocation shall not be set
Sec. 14 likewise allows the conduct of TRIAL IN ABSENTIA
aside by the President. 4Upon the initiative of the President, the
– This is where the trial will still proceed even without the Congress may, in the same manner, extend such proclamation or
presence of the accused provided he is being notified. What is suspension for a period to be determined by the Congress, if the
important is that the ARRAIGNMENT AND HIS PRESENCE IS invasion or rebellion shall persist and public safety requires it.
NECESSARY TO BE IDENTIFIED BY THE WITNESSES. This
is in relation to the right of the accused to be present at the The Congress, if not in session, shall, within twenty-four hours
trial. This particular right of the accused may be waived. In following such proclamation or suspension, convene in accordance
with its rules without need of a call. 5The Supreme Court may review,
other words, the accused may choose not to be present in the
in an appropriate proceeding filed by any citizen, the sufficiency
trial. Notwithstanding his absence, the Constitution allows the
of the factual basis of the proclamation of martial law or the
continuation of the trial. However, this rule is not absolute suspension of the privilege of the writ of habeas corpus or the
because there are two very important instances when the extension thereof, and must promulgate its decision thereon
accused’s presence is indispensable during trial that is: within thirty days from its filing.

(1) During arraignment; and

From the Discussions of Atty. Enrique Jr. Bonocan


University of Mindanao College of Law
REVIEWER ON CONSTITUTIONAL LAW II | CYNDI SATORRE-BICERA | 2nd Sem | S.Y. 2021-2022 51

6
A state of martial law does not suspend the operation of the 4. The President may request Congress to extend martial law,
Constitution, nor supplant the functioning of the civil courts or but this is always largely dependent on the determination of
legislative assemblies, 7nor authorize the conferment of jurisdiction Congress WON rebellion or invasion persists and public safety
on military courts and agencies over civilians where civil courts are
requires it.
able to function, 9nor automatically suspend the privilege of the writ of
habeas corpus.
5. This power of the president is still subject to the Supreme
10
The suspension of the privilege of the writ of habeas corpus shall Court’s Power of Judicial Review. It is not a pure political
apply only to persons judicially charged for rebellion or offenses question.
inherent in, or directly connected with invasion.
6, 7, and 8 – stated in Section 18, Article VII.
11
During the suspension of the privilege of the writ of habeas corpus,
any person thus arrested or detained shall be judicially charged within
9. If the President intends to suspend the privilege of the writ of
three days, otherwise he shall be released.
Habeas Corpus during martial law, he must expressly include it
Sec. 18 of Art VII of the Constitution, is otherwise known in his declaration.
as the Commander-in-Chief Clause of the Constitution.
This is indeed, pursuant to Sec. 3 of Art. III that civilian 10 and 11 – stated in Section 18, Article VII.
authority is at all times, supreme to the Military.
The concept behind the writ of habeas corpus – may be
The president is a civilian. As a matter of fact, there is no prayed before the court by any interested party to request the
requirement that the president should have experience in the court to order a particular government official or private
military but the Constitution treats him as the person to bring in the living body of another who is under his
commander-in-chief of the military. or her custody or detention. The office of this writ is to inquire
into all manner of involuntary restraint and to relieve such a
In relation to this, Sec. 18 actually gives us the Constitutional person from such restraint, especially if such restraint is illegal.
principle as well as the limitations on the declaration of either:
(1) martial law; or Ex: During martial law, a person was arrested for probably
(2) the suspension of the privilege of the writ of habeas corpus; committing rebellion or invasion, or any offense relative thereto
(3) BOTH. and then after 3 days, no criminal case has been filed against
that person, there was already the constitutional violation of his
Under the present Constitution, martial law may mean military right because according to Sec. 18, all persons arrested must
government over domestic territory. Martial law, in this sense, be judicially charged within 3 days, otherwise, they must be
when we talk about Sec. 18 of Art. VII, in relation to Sec. 15 of released. If the 3-day period the person is still under detention,
Art. III, may be properly treated as martial rule. And this takes the family can now file for a WRIT OF HABEAS CORPUS –
the place of certain governmental agencies, when for the time that is the best legal remedy. Go to court, and ask the court to
being, they are unable to cope with existing conditions in a issue an order directing the prison officials to bring before the
locality which makes it subject to sovereignty. There really is no court the living body of that person under their custody and
military government, only martial rule, where the latter merely explain and establish the legality of such detention. If the
assists certain government agencies who for the time being, detaining officials fail to justify the restraint they may be
are not able to cope with the existing conditions. And this ordered by the court to release the person detained.
normally happens during war or during a period of belligerency.
Martial law is an exercise of police power because the object is Ex: In private cases such as the father who has visitation rights
public safety – a concern of police power. failed to return the child. The mother or the family can file for a
writ of habeas corpus and not file a criminal case for
Sec. 18, Art. VII provides for the Limitations on the Power kidnapping because a father cannot kidnap his own child (legal
to Declare Martial Law and suspension of the Privilege of impossibility). Here, where the court issues the writ, it actually
the Writ of Habeas Corpus: orders the father to bring in the living body of the child and
explain why he must not be directed to return the custody over
1. The president cannot declare either martial law or suspend the child to the mother.
the privilege of the WHC or both when there is no rebellion,
invasion or public safety does not require it. As a matter of fact, Writ of Habeas Corpus – A writ issued by a court directed to a
you must not forget that these 2 requirements must concur. person detaining another, commanding him to produce the
Rebellion or invasion must not only exist, but public safety body of the prisoner at a designated time and place, with the
must also require it. day and cause of his caption and detention, to do, to submit to,
and to receive whatever the court or judge awarding the writ
2. It cannot exceed the period of 60 days and within 48 hours shall consider in his behalf [Sombong v. CA, G.R. No. 111876
of such proclamation, the president shall submit a report either (1990)].
in person or in writing to Congress.
Q: “Is the president’s power to declare martial law and
3. The President does not have the all-encompassing military suspend the privilege of the WOHC, a political question?”
powers. He may declare martial law or suspend the privilege of
writ of habeas corpus but it is still subject to the concurrence of It is stated in Sec. 18 of Art VII that ANY CITIZEN may
Congress. If Congress disagrees, it has the Constitutional question the validity of the declaration before the Supreme
power to revoke such a declaration. Court. But take note of the discussion in the following cases:

From the Discussions of Atty. Enrique Jr. Bonocan


University of Mindanao College of Law
REVIEWER ON CONSTITUTIONAL LAW II | CYNDI SATORRE-BICERA | 2nd Sem | S.Y. 2021-2022 52

• Barcelon vs. Baker – The Supreme Court held that such


SECTION 17. RIGHT AGAINST SELF-INCRIMINATION
executive power is a political question, meaning that the court
will not arrogate itself to exercise jurisdiction over the matter
because it is not for the court to answer. Section 17. No person shall be compelled to be a witness
against himself.
• Lansang vs. Garcia – Barcelon was abandoned, and the
court said that the power of the president to declare martial law NOTE: Section 17 is also known as the Right to
is not a political question, meaning, that the court may inquire self-incrimination. In the US, they called this the 5th
into its validity. amendment right.

• Padilla Garcia vs. Enrile – The court reverted to the ruling in Rationale: Generally, this was included on grounds of public
Barcelon. policy and humanity. After all, part of the constitutional rights of
a person accused of a crime is the right to be presumed
• Lagman vs. Medialdea (recent case) – Here among other innocent. A person may invoke this constitutional right even
things, it was alleged that the basis of the president in prior to the filing of charges against him or even during the
declaring martial law is the report from the military, which were pendency of the criminal trial or proceedings.
not all factually correct. The Supreme Court said that no, the
president is not required to establish the factual correctness of In some instances, in certain criminal cases, different counsels
the basis in declaring martial law is not required, because only would opt not to place the accused on the witness stand
probable cause is required of the president in validly declaring because he would be subjected to cross-examination by the
martial law and suspension of the PWHC. – textual approach. prosecution. In the event that the accused refuses to take the
witness stand, his refusal can never be used against him.
In the book of Fr. Bernas, SC, in reviewing the validity of Q: Which of the following situations can we say that there was
the Pres.’s declaration may use 3 kinds of approaches: violation of the person’s right against self-incrimination?

1. Textual Approach – The court will base its judgment upon A: For example, when the accused is subjected to ultraviolet
the black letter of the law. It asks the question, “What does the examination in buy-bust operations. This is likewise being used
letter of the constitution say?” When the law grants in some other entrapment operations like estafa or extortion. In
discretionary authority to a person to be exercised upon his order to prove that the person accused of that particular
opinion of certain facts, he alone is the judge of the existence offense had really received or touched the money, the money
of those facts. is considered the corpus delicti or the body of the crime. It will
be subject to UV rays. If the powder is found on the accused’s
2. Functional Approach – which asks the question, “Are we hands, the conclusion would be that indeed, he had touched
capable of resolving the problem posed?” The executive and the money.
legislative departments have the machinery for verifying the
existence of those facts whereas the courts do not. This is also Q: May the person sought to be subject to UV rays invoke
used in Sec. 18, because the declaration is always subject to his right against self-incrimination?
the concurrence of Congress.
3. Prudential or Political Approach – which asks “Whether A: The answer is No. Because that is not the type of
there are overriding considerations which prevent the court compulsion being protected by Section 17. The scope of
from entering thicket?” Interference by the courts in the Section 17 is testimonial compulsion, necessarily you have to
decision can result in tying the hands of those charged with utter some words. These words would tend to incriminate you
maintaining but these can be reduced to three, each order. or work against your favor which can be considered as your
admission to the offense. In UV rays, there is no testimonial
NOTE: The court here will just use Prudence in its review on compulsion.
the validity of the President’s declaration of martial law. This is
where the balancing of interest will come in. WON the Court ILLUSTRATIONS:
will preclude itself. The Court is faced with a dilemma where if
(a) it will preclude itself – it might endanger the lives of the • When subject to a paraffin test for the purpose of determining
people; (b) if the court arrogates – the Court will tie the hands whether or not he fired the gun, the accused CANNOT invoke
of the President who is incharge of maintaining public order. violation of his right against self-incrimination. The very same
reason that there is no testimonial compulsion in this particular
situation.
SECTION 16. RIGHT TO SPEEDY TRIAL
• When a judge “orders” in forcing out from the mouth of the
Section 16. All persons shall have the right to a speedy accused morphine, given that the accused is charged with a
disposition of their cases before all judicial, quasi-judicial, or violation of the Drugs Law, the accused CANNOT say that his
administrative bodies. right against self-incrimination was violated. That could not be
tenable because there is no testimonial compulsion.
Do not confuse this with the right of the accused to speedy trial
in Sec. 14. (for criminal cases). Under Sec. 16, this is • In one case where the accused is charged with rape, the
applicable to both criminal and civil cases. judge ordered him to put on a pair of pants which was
recovered from the crime scene. When the accused put on the
pants, it fit him. An inference could be made that those pants

From the Discussions of Atty. Enrique Jr. Bonocan


University of Mindanao College of Law
REVIEWER ON CONSTITUTIONAL LAW II | CYNDI SATORRE-BICERA | 2nd Sem | S.Y. 2021-2022 53

belong to the accused, and it could be concluded that he was the proof to convict him. The prosecution cannot compel the
at the crime scene. This situation DOES NOT depict a violation accused to provide the evidence that would show or prove his
of Section 17 because there is no testimonial compulsion. guilt. However, in documents, this can only be invoked by
natural persons. Corporations are not entitled to invoke this
• When a person is subjected to a police line-up, where the right. It is a personal right. The privilege against
complainant is tasked to identify the suspect (initial self-incrimination is a personal one and would only apply to
establishment of the identity of the suspect). Section 17 is natural persons or individuals. Another thing is that privilege
concerned, it CANNOT be invoked by the accused. which exists as to private papers cannot be maintained in
relation to records required by law to be kept in order that there
In the case of Villaflor v. Summers, the issue is WON a may be suitable information of transactions which are the
woman who is charged of adultery could be validly compelled appropriate subjects of governmental regulation and
to undergo a pregnancy test. The Court said in the conduct of enforcement of restrictions validly established. This is because
a pregnancy test, there is no testimonial compulsion. However, of the regulatory powers of the government, if the rule is
when it comes to a person’s handwriting, there is one case: otherwise, the regulatory powers of the government would be
rendered nugatory.
Beltran vs Samson (GR No. 32025, September 23, 1929).
Example, in labor laws, part of the powers conferred by law
upon the Secretary of Labor and Employment is the so-called
FACTS: Petitioner complained that the respondent judge
“visitorial powers.” This means that from time to time, the
ordered him to appear before the provincial fiscal to take
DOLE may check upon the records of a particular employer to
dictation in his own handwriting from the latter. The order
see to it that such employer is compliant with the labor
was given upon petition of said fiscal for the purpose of
standards. Another example is BIR, when the BIR would want
comparing the petitioner's handwriting and determining
to look at your financial records. Also in local business taxes,
whether or not it is he who wrote certain documents
the City Treasurer conducts an examination of books for the
supposed to be falsified. Prosecution did not have solid
purpose of determining whether or not the concerned
evidence as to the handwriting of the accused, it was later
taxpayers have been paying correct taxes.
offered in Court hich was made basis of the court’s
conviction of the accused.
Social Justice Society (SJS) v. Dangerous Drugs Board
HELD: The Supreme Court invalidated the act of the court.
According to it, it was a clear violation of a person’s right FACTS: Petitioners assail the constitutionality of Section 36
against self-incrimination. Writing is something more than of R.A. 9165, otherwise known as the Comprehensive
moving the body, or the hands, or the fingers; writing is not a Dangerous Drugs Act of 2002, insofar as it requires
purely mechanical act, because it requires the application of mandatory drug testing of candidates for public office,
intelligence and attention. For the purposes of constitutional students of secondary and tertiary schools, officers and
privilege, there is a similarity between one who is compelled employees of public and private offices, and persons
to produce a document, and one who is compelled to furnish charged before the prosecutor's office with certain offenses,
a specimen of his handwriting, for in both cases, the witness among other personalities. They invoked the right against
is required to furnish evidence against himself. self-incrimination.

ISSUE: Is the provision violative of the right against


Q: If a person invokes his Fifth Amendment (assuming we
self-incrimination?
are in the US), and as a result, keeps silent. Will his
silence be considered as a tacit admission?
HELD: YES but only with respect to persons charged before
A: No. This right against self-incrimination, as protected under
the prosecutor’s office. Unlike the situation covered by Sec.
Section 17, would be rendered meaningless if the silence of
36(c) and (d) of RA 9165, the Court finds no valid
the witness may be used against him. The refusal of the
justification for mandatory drug testing for persons accused
accused to be a witness or to answer should in no manner be
of crimes. In the case of students, the constitutional viability
used against him.
of the mandatory, random, and suspicionless drug testing for
Q: If a person is compelled to produce documents in his
students emanates primarily from the waiver by the students
possession, which are in themselves incriminating, may
of their right to privacy when they seek entry to the school,
he invoke his right against self-incrimination?
and from their voluntarily submitting their persons to the
A: Yes. The documents that could incriminate the person in
parental authority of school authorities. In the case of private
whose possession those papers could be found may invoke
and public employees, the constitutional soundness of the
the right against self-incrimination. This was established in the
mandatory, random, and suspicionless drug testing
old case of Boyd v. United States and Stonehill v. Diokno.
proceeds from the reasonableness of the drug test policy
and requirement.
In both cases, the Supreme Court held that compulsory
production of the private books and papers of the owner is
The situation is entirely different in the case of persons
compelling him to be a witness against himself. As established
charged before the public prosecutor's office with criminal
in the case of Boyd, the right against self-incrimination likewise
offenses punishable with six (6) years and one (1) day
covers documentary evidence. It is because an accused is
imprisonment. The operative concepts in the mandatory
presumed innocent and in relation to this right, it is incumbent
drug testing are "randomness" and "suspicionless." In the
upon the prosecution to prove the accused’s guilt beyond
case of persons charged with a crime before the
reasonable doubt, so we cannot ask from the accused himself

From the Discussions of Atty. Enrique Jr. Bonocan


University of Mindanao College of Law
REVIEWER ON CONSTITUTIONAL LAW II | CYNDI SATORRE-BICERA | 2nd Sem | S.Y. 2021-2022 54

Once the President/Secretary of Labor assumes jurisdiction


prosecutor's office, a mandatory drug testing can never be
over the dispute, there is an automatic twin effect:
random or suspicionless. The ideas of randomness and
being suspicionless are antithetical to their being made
(1) For the striking workers to return to work immediately and
defendants in a criminal complaint. They are not randomly
corollary to that, the management must admit all the returning
picked; neither are they beyond suspicion. When persons
striking workers; and
suspected of committing a crime are charged, they are
(2) The dispute is automatically referred to compulsory
singled out and are impleaded against their will. The
arbitration.
persons thus charged, by the bare fact of being haled before
the prosecutor's office and peaceably submitting themselves
SECTION 19. RIGHT AGAINST CRUEL PUNISHMENT &
to drug testing, if that be the case, do not necessarily
EXCESSIVE FINES
consent to the procedure, let alone waive their right to
privacy. To impose mandatory drug testing on the accused is
a blatant attempt to harness a medical test as a tool for Section 19. (1) Excessive fines shall not be imposed, nor
criminal prosecution, contrary to the stated objectives of RA cruel, degrading or inhuman punishment inflicted. Neither shall
9165. Drug testing in this case would violate a persons' right the death penalty be imposed, unless, for compelling reasons
to privacy guaranteed under Sec. 2, Art. III of the involving heinous crimes, the Congress hereafter provides for
Constitution. Worse still, the accused persons are veritably it. Any death penalty already imposed shall be reduced to
forced to incriminate themselves. reclusion perpetua.

(2) The employment of physical, psychological, or degrading


SECTION 18. FREEDOM FROM POLITICAL BELIEF AND punishment against any prisoner or detainee or the use of
INVOLUNTARY SERVITUDE substandard or inadequate penal facilities under subhuman
conditions shall be dealt with by law.
Section 18. (1) No person shall be detained solely by reason
of his political beliefs and aspirations. SECTION 20. NON-PAYMENT OF DEBT OR POLL TAX

(2) No involuntary servitude in any form shall exist except as a


Section 20. No person shall be imprisoned for debt or
punishment for a crime whereof the party shall have been duly
non-payment of a poll tax.
convicted.
Comment: No person shall be imprisoned for a debt, but only
Q: If you know that I believe in Communism, can you go to
when it comes to monetary obligations purely civil in nature
the police and ask the police to arrest me and file a case
(pure contract of loan).
against me because I am a rebel?
Exceptions:
A: No. That is a mere belief and that is protected under
1. B.P. 22 – a criminal offense for issuance of post-dated
Section 18, paragraph 1. In this particular instance, provided
checks that bounced or dishonored by the bank.
that this belief or aspiration remain in thought, so that as soon
2. Estafa – criminal offense of swindling; fraudulent debt.
as this belief is turned into overt acts, and these acts constitute
3. Malversation – committed by a public officer.
elements of a criminal offense, then I can no longer invoke the
protection in par. 1 of Section 18.
Question: What is poll tax? It is a Cedula.

Q: What is the concept of involuntary servitude?


A: No person can compel another to render services in his SECTION 21. DOUBLE JEOPARDY
favor against his will.
Section 21. No person shall be twice put in jeopardy of
There are many exceptions to this rule: punishment for the same offense. If an act is punished by a law
1. Parental Authority – parents can ask the children to work. and an ordinance, conviction or acquittal under either shall
constitute a bar to another prosecution for the same act.
2. Posse comitatus – the power of the State to call on all able
bodied citizens of the Philippines to render civil or military There are two species of Double Jeopardy under Sec. 21.
service especially in times of war or other national 1. Prosecution for the second time for the very same offense.
emergencies.
2. Acquittal or conviction of a person of an act both punishable
3. Labor as part of criminal penalty – e.g. community service by a national law or a local ordinance.
for offenses punishable under arresto menor (1 - 30 Days). Or
those CICL (Children in Conflict with the Law), they will be NOTE: Double Jeopardy is not a limitation on the enactment of
punished by diversion. penal laws but a limitation on the application or enforcement of
penal laws (criminal procedure, more on the judiciary function
4. Doctrine of Assumption of Jurisdiction – the power of the not legislative).
President thru the Secretary of Labor to automatically assume
jurisdiction over labor disputes in industries whose business is First Kind of Double Jeopardy: Prosecution for the second
so impressed with public interest. time for the very same offense. Either he is already acquitted
or convicted.

From the Discussions of Atty. Enrique Jr. Bonocan


University of Mindanao College of Law
REVIEWER ON CONSTITUTIONAL LAW II | CYNDI SATORRE-BICERA | 2nd Sem | S.Y. 2021-2022 55

ACQUITTAL or CONVICTION – There had been trial on the


Requisites for the attachment of Double Jeopardy: merits, meaning both parties are given their day in court, both
the prosecution and defense presented their evidence in court.
(1) First jeopardy must have been attached prior to the second.
(2) The first double jeopardy must have been terminated; and PEOPLE v. SALICO – it was ruled that there was dismissal in
(3) The second double jeopardy must be for the same offense the proper sense of the rule on double jeopardy because the
as that in the first. termination was not based on the merits but on a jurisdictional
issue. By the appeal, therefore, the accused was not placed in
As regards to the first requirement, the attachment of the first second jeopardy. The only issue raised on appeal was one of
jeopardy must have taken place prior to the attachment of the jurisdiction. The most that the appellate court could do was to
second jeopardy, meaning there was already either acquittal or remand the case to the court of origin. In such an event, there
conviction as regards to the first criminal case filed against the would be no new trial but merely a continuation of the first. The
accused and the second criminal action only came in later. dismissal, obtained with the express consent of the accused,
indeed, upon his instance, "constituted a waiver of his
First Requisite: This have sub-requisites: constitutional right or privilege, for the reason that he thereby
prevented the court from proceeding to the trial on the merits
When do we say that the first requisite has already taken and rendering a judgment of conviction against him." The
place? There are 5 requisites: heart of the teaching in Salico, therefore was: Dismissal that is
1. Upon a good or valid Indictment; not on the merits and without the consent of the accused is a
2. Must be before a Competent Court (Jurisdiction); bar to subsequent prosecution. Implicitly, too, Salico taught that
3. Must be after Arraignment; termination based on the merits, even when improperly called
4. After the accused enters his Plea; dismissal, and with or without the consent of the accused, is a
5. After the trial of the case has actually commenced by bar to further prosecution, because such termination is, in fact,
the questioning of at least one witness (this is an an acquittal.
obiter dictum – Fr. Bernas)
GR: A judgment of dismissal of the criminal case which would
NOTE: In the Book of Riano and cited in the case of Braza v. also include acquittal is also a BAR to another prosecution for
SandiganBayan and many other jurisprudence, the 5th the same offense, meaning there is DOUBLE JEOPARDY.
requisite is the acquittal or conviction of the accused, or the
dismissal or termination of the case against him without his NOTE: This will only apply if the dismissal or the termination of
express consent. the criminal case by either way (acquittal or conviction or
dismissal) this presupposes that the court that rendered the
UPON A GOOD OR VALID INDICTMENT – Means the judgment of dismissal had jurisdiction over the case or the
information filed before the court is valid; it satisfies all the legal court is competent. Thus, APPEAL tantamounts to Double
as well as the procedural requirements. Section 7, Rule 117, Jeopardy. Judgment of Dismissal/Acquittal must be FINAL (No
New Rules of Court, says that there must be a "valid Motion for Reconsideration) except if there is GADALEJ.
complaint or information or other formal charge sufficient
in form and substance to sustain a conviction." Under such XPNs:
rule, it has been uniformly held that if the charge is fatally 1. ERROR OF JURISDICTION (court has no jurisdiction) = NO
defective in form, as in a charge of rape where the complaint is Double Jeopardy; Dismissal is VOID.
signed not by the complainant but by the police chief, jeopardy
does not attach." Thus, too, when the substance of the 2. Dismissal with expressed CONSENT (moved for the
information does not allege sufficient facts to constitute an dismissal of the case on his own instance even prior to the
offense, jeopardy does not attach." Under this and similarly inception of the prosecution’s evidence in chief) = NO Double
defective information, the accused can never be convicted and, Jeopardy because it tantamounts to waiver of his constitutional
hence, he cannot be said to have been in jeopardy of right against Double Jeopardy.
conviction."
3. GADALEJ (Grave Abuse of Discretion Amounting to Lack or
SECOND REQUISITE: Termination of the First Jeopardy Excess of Jurisdiction) = When it comes to dismissal/acquittal
Here, there can be no Double Jeopardy if there is no judgment handed by trial courts out of GADALEJ, then this judgment is
of conviction, or acquittal, or no dismissal or termination in a NULL and VOID under the law which will preclude the
former case for the same offense. attachment of the constitutional protection of Double Jeopardy.

Example: Whenever a criminal case is dismissed by the Court In one case, the Supreme Court allowed the attachment of first
prior to the conduct on the trial on the merits on the grounds Double Jeopardy in a criminal case when the Prosecution for
that the right of the accused to speedy trial has been violated so many times failed to appear to present its evidence in chief
by the prosecution, then the EFFECT OF THAT DISMISSAL is (always postponing or the case is being dragged at the
tantamounts to ACQUITTAL. Hence, this is a BAR to another expense of the accused).
prosecution to the same offense.
If the accused should deem such conditional or provisional
DISMISSAL – Termination of the case without going to the trial dismissal to be unjust and prejudicial to him because he has
on the merits. Or in some other cases, the accused filed a been deprived of his right to speedy trial as for instance, his
DEMURRER to evidence and the court granted it. case has been dragged for an unreasonably long time, he

From the Discussions of Atty. Enrique Jr. Bonocan


University of Mindanao College of Law
REVIEWER ON CONSTITUTIONAL LAW II | CYNDI SATORRE-BICERA | 2nd Sem | S.Y. 2021-2022 56

could object to such dismissal and insist that he be decided on and/or (ii) where there is a denial of a party’s due process
the merits. rights. A judgment of acquittal sought to be reviewed on the
basis of grave abuse of discretion amounting to lack or excess
NOTE: Verbal Dismissal is not final unless it is written and of jurisdiction or on the ground of denial of due process implies
signed by the judge. There must be an order or decision. an invalid or otherwise void judgment. If either or both grounds
are established, the judgment of acquittal is considered void;
Second Kind of Double Jeopardy: Acquittal or conviction as a void judgment, it is legally inexistent and does not have
of a person of an act both punishable by a national law or the effect of an acquittal. Thus, the defense of double jeopardy
a local ordinance. will not lie in such a case.

Ex. Smoking in public places – Anti-Smoking Ordinance of


LITO BAUTISTA and JIMMY ALCANTARA v. SHARON G.
Davao City and RA 9211 (National Law). The accused pleaded
CUNETA-PANGILINAN
guilty or not guilty and there was a conduct of trial and later on
convicted of the ordinance, then the Prosecution wanted to file
FACTS: Sharon filed a criminal case of libel against Lito
another case for RA 9211, that is NOT ALLOWED because
Bautista, Editor-Publisher of Bandera. During trial, after the
Double Jeopardy has already been attached, viz-a-viz.
prosecution rested its case, Lito Bautista filed a demurrer of
evidence on the ground that the Prosecution failed to
Q: What about the happening of a supervening event, may
establish the identity of the accused. Apparently, during the
the accused invoke double jeopardy?
trial, when the Prosecution presented its evidence, when
Sharon’s private lawyer conducted a trial, he forgot to let the
Ex. A mauled B and B was hospitalized, and A was charged
witness identify the accused (very fatal) and the trial court
criminally for serious physical injuries. While serving his
granted the demurrer (amounts to acquittal). Because of the
sentence, A died on account of supervening effect, may the
dismissal, the Prosecution filed an appeal. Bautista
Prosecution filed for homicide for a heavier penalty?
interposed his right against double jeopardy.
A: There are two (2) rules here:
HELD: It should be stressed that the granting of petitioners’
1. Old rule – (People v. Tarok) the death of the injured person
Demurrer to Evidence already amounted to a dismissal of
was the principal element of homicide but it was not a part of
the case on the merits and a review of the order granting the
the assault of battery. At the time of the trial, death had not yet
demurrer to evidence will place the accused in double
ensued and not until it did ensue was the homicide committed.
jeopardy.
Here, there is Double Jeopardy because the second
offense is necessarily included in the first offense or
Corollarily, after the prosecution rests its case, and the
viz-a-viz.
accused files a Demurrer to Evidence, the trial court is
required to evaluate whether the evidence presented by the
2. New rule – enunciated in the case of Melo v. People
prosecution is sufficient enough to warrant the conviction of
(1950), the rule for the determination of identity of offenses "did
the accused beyond reasonable doubt.
not apply when the second offense was not in existence at the
time of the first prosecution, for the simple reason that in such
In criminal cases, the grant of demurrer is tantamount to
a case there is no possibility for the accused, during the first
acquittal and the dismissal order may not be appealed
prosecution, to be convicted for an offense that was then
because this would place the accused in double jeopardy.
inexistent." The rule is that: "Where after the first prosecution a
Although the dismissal order, is not subject to appeal, it may
new fact supervenes for which the defendant is responsible,
be reviewed only through certiorari under Rule 65 (special
which changes the character of the offense and together with
civil action not a mode of appeal which contains a prayer
the facts existing at the time, constitutes a new and distinct
that the court review the validity of the act of the court in
offense, the accused cannot be said to be in second jeopardy if
dismissing the case with the allegation of GADALEJ such as
indicted for the new offense." {trial proceed with amended info}
where the prosecution is denied the opportunity to present
its case or when the trial is a sham, thus rendering the
People v. Buling (1960) – ISSUE: whether the prosecution
assailed judgment void).
and conviction of Balaba for less serious physical injuries is a
bar to the second prosecution for serious physical injuries.
The burden on the petitioner is to clearly demonstrate that
HELD: The Court does not believe that a new fact supervened,
the trial court abused its authority to a point so grave as to
or that a new fact has come into existence. Added the
the privy of its very power to dispense justice. In this case,
qualification that where the exact nature of the injury could
no such circumstance exists to warrant a departure from the
have been discovered, but was not, because of the
general rule and reverse the findings of SB.
incompetence of the physician, the subsequent discovery of
the real extent of the injury would not be a supervening fact
which could warrant the application of the Melo doctrine. NOTE: In a criminal case, it must only be appealed by the
SolGen, not the private prosecutor which Sharon’s party did.
ON APPEALS – In PEOPLE v. SANDIGANBAYAN, the
proscription against double jeopardy only envisages appeals
JASON IVLER v. MARIA MODESTO-SAN PEDRO
based on errors of judgment, but not errors of jurisdiction.
Jurisprudence recognizes two grounds where double jeopardy FACTS: Jason Ivler was charged with a criminal case of (1)
will not attach, these are: (i) on the ground of grave abuse of reckless imprudence resulting in slight physical injury and
discretion amounting to lack or excess of jurisdiction; (2) reckless imprudence resulting in homicide and damage

From the Discussions of Atty. Enrique Jr. Bonocan


University of Mindanao College of Law
REVIEWER ON CONSTITUTIONAL LAW II | CYNDI SATORRE-BICERA | 2nd Sem | S.Y. 2021-2022 57

1. Criminalizes an act and punishes such an act which was


to property in the other court. Ivler pleaded guilty to the first innocent when done before the passage of the law;
case and was meted out a penalty of public censure (did not 2. Aggravates a crime, makes it greater than it was committed;
go to prison). He then moved for the dismissal for the other 3. Inflicts a greater Punishment than that annexed to the crime
criminal action invoking double jeopardy. He asserted that when committed;
since his first case has already been dismissed, his second 4. Alters the legal rules of Evidence, authorizes conviction
case should also be dismissed. The Prosecution disagreed. upon less or different testimony than the law required at the
time of the commission of the offense in order to convict the
ISSUE: Will double jeopardy bar the prosecution of Ivler in defendant;
the other criminal case considering he was already 5. Assumes to Regulate civil rights and remedies only, but in
convicted of reckless imprudence resulting in slight physical effect imposes penalty or deprivation of a right for something
injury? which when done was lawful; ie. Art. 34 Civil interdiction.
6. Which deprives a person accused of a crime of some lawful
HELD: Yes. Reckless Imprudence is a Single Crime, its
Protection i.e. acquittal, amnesty, Presidential clemency (Art.
consequences on persons and property are material
19, Sec 7 of the 1987 Constitution)
only to determine the penalty prior conviction or
7. Every law which, in relation to the offense or its
acquittal of Reckless Imprudence Bars Subsequent
consequences, alters the Situation of a person to his
Prosecution for the Same Quasi-Offense. The two charges
disadvantage.
against petitioner, arising from the same facts, were
prosecuted under the same provision of the Revised Penal
Code, as amended, namely, Article 365 defining and Illustration/Examples of the Prohibition:
penalizing quasi-offenses.
1. One which makes an action done before the passing of the
Structurally, the acts as defined in Art. 365 are collapsible law and which was innocent when done, CRIMINAL and
into four sub-groupings relating to (1) the penalties attached punishes such action.
to the quasi-offenses of "imprudence" and "negligence" (par.
1-2); (2) a modified penalty scheme for either or both
quasi-offenses (par. 3,4,6 and 9); (3) a generic rule for trial Ex: Wearing of masks – Both the Bayanihan Laws and Local
courts in imposing penalties (par. 5); and (4) the definition of ordinances criminalizes the non wearing of facial masks
"reckless imprudence" and "simple imprudence" (par. 7 & 8). because of the pandemic. Say the provision will state that
Conceptually, quasi-offenses penalize "the mental attitude or non-wearing of the facial mask is considered CRIMINAL and
condition behind the act, the dangerous recklessness, lack those who failed to wear their face mask in public 90 days prior
of care or foresight, the imprudencia punible," unlike willful to the effectivity of this act shall also be criminally liable. - This
offenses which punish the intentional criminal act. These
is an example of an ex-post facto law. Here, the law is given a
structural and conceptual features of quasi-offenses set
them apart from the mass of intentional crimes under the RETROACTIVE effect as regards to the applicability of criminal
first 13 Titles of Book II of the RPC, as amended. sanctions. This is prohibited.

In other words, there was error on the part of the 2. Those which AGGRAVATES a crime or makes it greater
Prosecution to file two separate criminal cases when the than when it was committed.
accused here sought to be penalized for ONLY ONE
SINGLE ACT – reckless imprudence.
Ex: Murder and Homicide. Murder is considered a heinous
There must be one case filed but all the allegations such as crime, in fact it is considered as a capital offense as opposed
the slight physical injury, death of the husband and damage to homicide where the penalty is only reclusion temporal. If
to the property and these three material facts will only be Congress will pass a law that all killings regardless of the
material to the determination to determine the severity of the attendance or non attendance of the qualifying circumstance
penalty to be imposed. will be considered as murder that is an ex-post facto law.

3. That which changes the PUNISHMENT and inflicts a greater


SECTION 22. EX POST FACTO LAW & B.O.A punishment than the law annexed to the crime when it was
committed.
Section 22. No ex post facto law or bill of attainder shall be
enacted. Ex: In Slight Physical Injuries. This is punishable by arresto
menor. Now, under the ISLAW (Indeterminate Sentence Law)
The Congress is prohibited by the Constitution to enact ex-post the minimum penalty is only 1-10 days. If Congress will pass a
facto laws and bills of attainder (bills of pain). law that the penalty of the same will be changed from arresto
menor to arresto mayor. And further, all slight physical injuries
BILL OF ATTAINDER – Kinds of law that provides for committed within 1 year before the effectivity of this act shall be
automatic determination of a crime depriving the accused of punished under this law. This is an ex-post facto law.
due process, trial, remedies.
4. That which alters the legal rules of EVIDENCE and receive
EX-POST FACTO LAW – Retroactive application of laws; Fait less or different testimony than the law required at the time of
Accompli; Adheres to the elementary rule of nullum crimen the commission of the offense in order to convict the
nulla poena sine lege. It is an act by the Congress to impose a defendant.
penalty to an act which is not criminal or penalized at the time
of commission.
Ex: In criminal laws, the prosecution is constitutionally required
to prove the guilt of the accused beyond reasonable doubt.
Six types of ex post facto law: [CAPERPS] Lacson v.
ExSec That is the quantum of evidence when it comes to criminal
cases. In civil cases, it is mere preponderance of evidence. In

From the Discussions of Atty. Enrique Jr. Bonocan


University of Mindanao College of Law
REVIEWER ON CONSTITUTIONAL LAW II | CYNDI SATORRE-BICERA | 2nd Sem | S.Y. 2021-2022 58

administrative proceedings, it is substantial evidence. If Simply put, Congress by way of Republic Act can just convict a
Congress will pass a criminal law that will only require person for a particular crime without judicial trial, the act is
preponderance of evidence to acquire a conviction.. Of course, properly termed as the BILLS OF PAINS AND PENALTY.
that cannot be because that is an ex-post facto law.
Historical background of this Particular Principle: It
NOTE: The prohibition against ex-post facto laws may only be originated from Thomas Wenworth when he was still the legal
applicable to criminal legislation which affects the substantial advisor to King Charles I. Because he was so brilliant he was
rights of the accused. Also applies to criminal procedural law brought to the impeachment court for attempting to suburbed
prejudicial to the accused. This would mean that any the liberties of England. People created false accusations
amendment to an existing criminal law or rules of criminal against him so he’d be abdicated. However, he was able to
procedures that will not have the effect of negatively affecting defend himself so ably and so brilliantly that his enemies
the substantial rights of the accused but rather would benefit fearing for his acquittal withdrew the impeachment case and
him CANNOT BE CONSIDERED AN EX-POST FACTO LAW. passed a bill ordering for the beheading of Thomas without
going through the process of impeachment proceedings. The
GR: Criminal laws should always be prospective. law itself already pronounces the guilt of the person and
XPN: When the retroactive effect would benefit the accused. provides for his punishment.

Ex: Robin Padilla was convicted in a criminal case for illegal


LUMANOG v. PEOPLE G.R. No. 182555
possession of firearm (9mm pistol) during the effectivity of PD
1866 where all firearms regardless of the caliber have the
FACTS: Sec. 3 of RA 9346 abolished the Death Penalty –
same penalty. A new law which amended PD 1866 and
all death sentences even including those which became final
categorized the offense under this law as to the penalty was
and executory were reduced to Reclusion Perpetua and in
enacted. Under the new law, those who were convicted of this
addition to that the death convicts were precluded to avail
particular offense by way of having in their possession firearms
Propbation. This was questioned to be an ex post facto law.
of low caliber entails a lower penalty. So when this law took
effect, it turned out that Robin Padilla had already served the
RULING: No, this is not an ex-post facto law. The SC said
minimum sentence for those who have been convicted for
No constitutional sanctities will be offended if persons
possession of lower caliber firearms. He was given a
previously sentenced to death, or persons sentenced to
retroactive effect because it is favorable to the accused.
reclusion perpetua, are denied the benefit of parole. As to
persons previously sentenced to death, it should be
Q: WHAT IS NOT EX-POST FACTO LAW?
remembered that at the time of the commission of the crime,
the penalty attached to the crime was death. To their benefit,
A: The statutory changes in the mode of trial or Rules of
Rep. Act No. 9346 reduced the penalty attached to the
Evidence which do not deprive the accused of defense
crime to reclusion perpetua. Yet such persons cannot claim
and which occur only in limited and substantial manner to
the benefit of parole on the basis of the ex-post facto clause
his disadvantage are not prohibited. Rules of Procedure
of the Constitution, since an ex-post facto law is one which,
which have nothing to do with the substantial rights of the
among others, "changes punishment, and inflicts a greater
accused will not amount to ex-post facto law.
punishment than the law annexed to the crime when
committed." RA 9346 had the effect of "inflicting" a lighter
Ex: Under the Summary Procedure (criminal cases under the
punishment, not a greater punishment, than what the law
jurisdiction of MTCs; penalty is less than one year), this
annexed to the crime when committed.
particular kind of statutory change is not tantamount to ex-post
facto law because it does not negatively affect the statutory
rights of the accused. The amendatory rule will only be
PEOPLE v. JOSEPH ESTRADA G.R. No. 164368-69
pertaining to the Rules that should be followed during the
proceedings.
FACTS: One of the issues raised here as a defense of
Estrada in the alleged use of the name of Jose Velarde in
Ex: The Judicial Affidavit Rule – here, direct testimony of a
violation of the Anti Money Laundering Law. However RA
witness is given by way of a written affidavit. All you have to do
9160 or AMLA was not yet in effect when Estrada used such
is to let the witness Identify the document and afterwards, the
alias. Despite that, he was indicted.
opposite party will examine it. This particular instance is NOT
an ex post facto law.
HELD: The enactment of R.A. 9160, is a significant
development only because it clearly manifests that prior to
BILL OF ATTAINDER – PJ-BRIM
its enactment, numbered accounts or anonymous accounts
•Legislative act which inflicts Punishment without Judicial trial
were permitted banking transactions, whether they be
•Imposition of a Burden on it
allowed by law or by a mere banking regulation. To be sure,
•Retroactive application to past conduct suffice to
stigmatize an indictment against Estrada using this relatively recent law
•A legislative Intent cannot be maintained without violating the constitutional
•Singling out a definite Minority prohibition on the enactment and use of ex post facto laws.
•Bill of Attainder is objectionable because of its ex post Finally, R.A. No. 9160, as a law of recent vintage in relation
facto features. Accordingly, if a statute is a Bill of Attainder, to the indictment against Estrada, cannot be a source or an
it is also an ex post facto law. influencing factor in his indictment.

From the Discussions of Atty. Enrique Jr. Bonocan


University of Mindanao College of Law
REVIEWER ON CONSTITUTIONAL LAW II | CYNDI SATORRE-BICERA | 2nd Sem | S.Y. 2021-2022 59

BILLS OF ATTAINDER; DEFINED:


termination of a BIR or BOC official or employee and
provides for the consequences thereof. The democratic
EXECUTIVE SECRETARY v. CA – Bills of attainder are
processes are still followed and the constitutional rights of
legislative acts which inflict punishment on individuals or
the concerned employee are amply protected.
members of a particular group without a judicial trial. Essential
to a bill of attainder are a specification of certain individuals or
NOTE: The CSC Laws must be adhered also.
a group of individuals, the imposition of a punishment, penal or
otherwise, and the lack of judicial trial.
An act declaring all the members of the CPP-NPA guilty of the
Bills of attainder are an ancient instrument of tyranny. In more crime of rebellion and hereby provides for their penalty to suffer
modem terms, a bill of attainder is essentially a usurpation the penalty of reclusion temporal, etc. – IS A BOA
of judicial power by a legislative body. It envisages and
effects the imposition of a penalty — the deprivation of life NOTE: Prohibition against the bill of attainder cannot be
or liberty or property — not by the ordinary processes of invoked in non-criminal issues. [Bosea case]
judicial trial, but by legislative fiat. While cast in the form of
special legislation, a bill of attainder (or bill of pains and —- END OF FINALS COVERAGE —
penalties, if it prescribed a penalty other than death) is in intent
and effect a penal judgment visited upon an identified person
or group of persons (and not upon the general community)
Without a prior charge or demand, without notice and hearing,
without an opportunity to defend, without any of the civilized
forms and safeguards of the judicial process as we know it.
Such is the archetypal bill of attainder wielded as a means of
legislative oppression. [TUAZON v. ROD]

BOSEA VS. MARGARITO TEVES G.R. No. 181704

FACTS: RA 9335 intends to encourage BIR and BOC


officials and employees to exceed their revenue targets by
providing a system of rewards and sanctions through the
creation of a Rewards and Incentives Fund and a Revenue
Performance Evaluation Board. It covers all officials and
employees of the BIR and the BOC with at least six months
of service, regardless of employment status. The provision
states that:

a. That he/she will meet the allocated Revenue Collection


Target and thereby undertakes and binds himself/herself
that in the event the revenue collection falls short of the
target with due consideration of all relevant factors affecting
the level of collection as provided in the rules and
regulations promulgated under the Act and its IRR, he/she
will voluntarily submit to the provisions of Sec. 25 (b) of the
IRR and Sec. 7 of the Act; and

b. That he/she will cascade and/or allocate to respective


Appraisers/Examiners or Employees under his/her section
the said Revenue Collection Target and require them to
execute a Performance Contract, and direct them to accept
their individual target. The Performance Contract executed
by the respective Examiners/Appraisers/Employees shall be
submitted to the Office of the Commissioner through the
LAIC on or before March 31, 2008.

HELD: No, RA 9335 is NOT a bill of attainder. A bill of


attainder is a legislative act which inflicts punishment on
individuals or members of a particular group without a
judicial trial. Essential to a bill of attainder are a specification
of certain individuals or a group of individuals, the imposition
of a punishment, penal or otherwise, and the lack of judicial
trial. RA 9335 does not possess the elements of a bill of
attainder. It does not seek to inflict punishment without a
judicial trial. It merely lays down the grounds for the

From the Discussions of Atty. Enrique Jr. Bonocan


University of Mindanao College of Law

You might also like